SlideShare a Scribd company logo
1 of 113
Download to read offline
Problemas y ejercicios
resueltos de
Termodinámica I

Julián Moreno Mestre
Estudiante de Ciencias Físicas
Universidad Complutense de Madrid

"Lo que hacemos por nosotros mismos, muere con nosotros. Lo que hacemos por los
demás y por el mundo, permanece y es inmortal." Albert Payne
Julián Moreno Mestre

1
Ejercicios y problemas de Termodinámica I

Índice:
PRÓLOGO ……………………………………………………………………….

4

BIBLIOGRAFÍA CONSULTADA ………………………………………………

5

CAPÍTULO 1º. Principio cero de la termodinámica y temperatura. Ecuaciones
de estado. Coeficientes termodinámicos importantes. Relaciones diferenciales en
termodinámica. …………………………………………………………….……..

6

CAPÍTULO 2º. Trabajo en termodinámica. Relaciones entre las derivadas
parciales. Primer principio de la termodinámica. Coeficientes calorimétricos ….

22

CAPÍTULO 3º. Segundo principio de la termodinámica. Temperatura
termodinámica y entropía. Principio de aumento de entropía. Ecuación
fundamental de la termodinámica. Ecuaciones TdS. ……………………………..

42

CAPITULO 4º. Potenciales termodinámicos. Relaciones de Maxwell…………...

78

CAPÍTULO 5º. Transiciones de fase. Regla de las fases. ………………………..

102

2
Julián Moreno Mestre

3
Ejercicios y problemas de Termodinámica I

PRÓLOGO:
Tras muchos años cursando como alumno la asignatura de Termodinámica I en la
facultad de ciencias físicas de la Universidad Complutense de Madrid, decidí hacer un
recopilatorio de parte de los problemas que he estudiado y resuelto en la asignatura.
Mi experiencia positiva en las asignaturas de Mecánica y Ondas II y Electromagnetismo
I, en las que contribuí a escribir librillos de problemas con la academia INTEC con
Ramón Fernández Villegas, me ha llevado a hacer lo mismo con Termodinámica I. El
resultado en termodinámica fue un notable y conocer mejor la termodinámica, y fruto
del trabajo en la asignatura ha quedado además esta colección de problemas resueltos
que espero les sirva de ayuda.
Muchos de los problemas son ejercicios de clase de la asignatura. La mayoría de los
problemas provienen de libros de termodinámica. En cada uno he procurado contrastar
la solución final que daban los libros o en clase. No obstante, prefiero advertir que al ser
una publicación sin revisión puede ser posible encontrar no pocos errores. Si cualquiera
que lea este documento encuentra un error comuníquemelo al mail para su posterior
corrección y mejora de este texto.
No es un texto completo en problemas para la asignatura. La ausencia de colecciones de
problemas de sistemas abiertos, así como la ausencia de problemas y numerosos casos
prácticos en lo referente al equilibrio termodinámico y al tercer principio, dejan
incompleto este texto. No obstante estará abierta la posibilidad de un sexto capitulo o
ampliación de algunos que incluya y corrija la ausencia de estos temas que se imparten
en Termodinámica I.
Ideal sería dotar a este texto de un Capítulo 0 que trate sobre los métodos matemáticos
utilizados en Termodinámica I, como es la integración de ecuaciones diferenciales en
una y varias variables.
Quiero expresar mi agradecimiento a la profesora del departamento de Física Aplicada I
Vicenta Maria Barragán por la gran cantidad de dudas que me resolvió con los
problemas. Sin duda sus orientaciones me han servido para resolver ciertos problemas
que puedo presentar en el presente texto.
También mi agradecimiento al fallecido profesor José Aguilar Peris, del cual es
imposible no considerarse alumno suyo cuando he aprendido mucho de su libro Curso
de Termodinámica. No son pocos los problemas que he sacado de su libro.
Es mi deseo dedicarle este texto a todos los compañeros de la facultad de ciencias
físicas de la complutense, espero que les sirva de mucha ayuda, y espero me disculpen
por algún posible error que puedan encontrar.

Julián Moreno Mestre
Madrid, 25 de Marzo de 2008
julianmorenomestre@hotmail.com

4
Julián Moreno Mestre

BIBLIOGRAFÍA:
Para la elaboración de esta colección de problemas así de cómo los resúmenes de teoría
se ha consultado varios libros.
Curso de termodinámica. José Aguilar Péris. Ed. Alhambra Longman .
Termo I y II. Manuel Zamora Carranza. Ed. Universidad de Sevilla.
Físicoquímica I y II. Levine, Ed. Mac Graw Hil.
Problemas de Física Vol. III Termología. E. Gullón de Senespleda y M. López
Rodríguez. Ed. Librería internacional de Romo SL.
100 Problemas de Termodinámica. J. A. Manzanares y J. Pellicer. Ed. Alianza.
Calor y Termodinámica. Zemansky y Dittman. Ed. Mac Graw Hill.

5
Ejercicios y problemas de Termodinámica I

CAPÍTULO 1º
Principio cero de la termodinámica y temperatura. Ecuaciones de estado.
Coeficientes termodinámicos importantes. Relaciones diferenciales en
termodinámica.

Resumen de teoría:
Principio cero de la termodinámica:
- Dos sistemas aislados A y B puestos en contacto prolongado alcanzan el
equilibrio térmico.
- Si A y B separadamente están en equilibro térmico con C, están también en
equilibrio térmico entre si. (Propiedad transitiva)
Temperatura empírica:

θ ( x) = 273.16
Ecuaciones de estado importantes:
Gas ideal:
Sólido paramagnético (Ley de Curie):

x
x PT

pV = nRT
TM = HC

Gas de Van der Waals:
⎛
n2a ⎞
⎜ p + 2 ⎟(V − nb) = nRT
⎜
V ⎟
⎠
⎝

Coeficientes termodinámicos importantes:
Dilatación cúbica
Compresibilidad isoterma
1 ⎛ ∂V ⎞
1 ⎛ ∂V ⎞
β= ⎜
κT = − ⎜
⎟
⎟
V ⎝ ∂T ⎠ p
V ⎝ ∂p ⎠T

Piezotérmico
1 ⎛ ∂p ⎞
α= ⎜ ⎟
p ⎝ ∂T ⎠V

6
Julián Moreno Mestre

Problemas:
1º

Los sistemas A y B son sales paramagnéticas con coordenadas (H, M) y (H’, M’)
respectivamente, mientras que el sistema C es un gas con coordenadas (p, V). Cuando
A y C están en equilibrio térmico se cumple: nRcH − MpV = 0 , y cuando lo están B y
C se cumple: EMBED Equation.3 M ' pV − nR(c' H '+ aM ' ) = 0 , siendo los símbolos
n, R, a, c y c’ constantes:
a) ¿Cuáles son las funciones, del par de variables de cada sistema, iguales entre si
en el equilibrio térmico?
b) ¿Cuál es la relación que expresa el equilibrio térmico entre los sistemas A y B?
Solución:
a) Partiendo de las relaciones entre los sistemas en el equilibrio, las funciones del par
de variables iguales entre si en el equilibrio son:
nRcH
⎫
nRcH − MpV = 0 → pV =
⎪
nRcH nR (c ' H '+ aM ')
⎪
M
=
⎬ pV =
M
M'
nR(c ' H '+ aM ') ⎪
M ' pV − nR(c ' H '+ aM ') = 0 → pV =
⎪
⎭
M'

b) La relación en el equilibrio entre los sistemas A y B la extraemos a partir de la
relación obtenida en el apartado a):
RcH R(c ' H '+ aM ')
nRcH nR(c ' H '+ aM ')
=
=
→
M
M'
M
M'
2º

Los sistemas A y B son gases ideales con coordenadas (p,V) y (p,V), respectivamente,
y el sistema C es una sustancia elástica de coordenadas (F,L). Cuando A y C están en
equilibrio térmico se cumple:
⎛ L L2 ⎞
0
kpV ⎜ − 2 ⎟ − FR = 0
L0 L ⎠
⎝
Cuando están en equilibrio térmico A y B se cumple:
pV − p '(V '− b) = 0
siendo b, R, k y L0 constantes. ¿Cuáles son las funciones del par de variables de cada
sistema, iguales entre sí en el equilibrio térmico? ¿Cuál es la relación que expresa el
equilibrio térmico entre los sistemas B y C?
Solución: Partiendo de las relaciones de equilibrio:
⎫
⎛ L L2 ⎞
FR
0
kpV ⎜ − 2 ⎟ − FR = 0 → pV =
−1
⎪
⎛ L L2 ⎞ ⎪
FR ⎛ L L2 ⎞
⎝ L0 L ⎠
0
0
k ⎜ − 2 ⎟ ⎬ ⇒ pV = p '(V '− b) =
⎜ − ⎟
k ⎝ L0 L2 ⎠
⎝ L0 L ⎠ ⎪
⎪
pV − p '(V '− b) = 0 → pV = p '(V '− b)
⎭
Partiendo de la relación entre el par de variables de cada sistema en el equilibrio,
podemos deducir la relación que expresa el equilibrio térmico entre B y C:

FR ⎛ L L2 ⎞
0
p '(V '− b) =
⎜ − ⎟
k ⎝ L0 L2 ⎠

7

−1
Ejercicios y problemas de Termodinámica I
3º

Un sólido dieléctrico tiene como variables la polarización , P, y el campo eléctrico E,
una sal paramagnética la imanación, I, y el campo magnético H y un gas tiene las
coordenadas presión, p, y la densidad ρ. Cuando están en equilibrio mutuo, el primero
y el segundo:
χ d EH = β IP
y cuando están en equilibrio el primero y el tercero cumplen la condición:
α pP = ρχ d E
donde α y β son constantes y χ d es la susceptibilidad dieléctrica del material no
conductor. ¿Cuál es la ecuación que expresa el equilibrio térmico entre el segundo y el
tercer sistema? Tomando como temperatura empírica la del gas ideal: T = MpV / mR ,
donde M es la masa molecular del gas, determinar las ecuaciones de estado de los tres
sistemas. (Carranza)
Solución: Despejamos χ d E en las ecuaciones del equilibrio antes citadas:

χ d EH = β IP → χ d E=

β IP ⎫

β IP α pP
H ⎪
⎪
=
⎬ → χ d E=
α pP ⎪
H
ρ
α pP = ρχ d E → χ d E=
ρ ⎪
⎭
Por tanto el equilibrio entre el segundo y el tercero es:
βI α p
=
H
ρ
En el problema la temperatura empírica del gas es:
αp
T=

ρ

Partiendo que la temperatura empírica del gas ideal es:
MpV Mp V Mp M p
=
=
=
T=
mR
R m Rρ R ρ
Esto significa que la constante α es:
M
α=
R
Por tanto la temperatura empírica del gas es:
M p
T=
R ρ
Por la relación de equilibrio de la sal y el gas sabemos que la temperatura de la sal es:
I
T =β
H
Y por ello también sabemos que la temperatura empírica del sólido dieléctrico será:
χ E
β IP
χ d E=
= PT → χ d E = PT → T = d
P
H

8
Julián Moreno Mestre
4º

Tres sistemas, que llamaremos 1, 2 y 3 y que tienen el volumen y la presión como
variables mecánicas, se ponen en contacto térmico por parejas. Cuando el primero y el
tercero están en equilibrio térmico se cumplen:
p1V1 − p2V2 = bp1
y cuando lo están el segundo y el tercero:
V3 ( p2V2 − p3V3 ) = a
donde a y b son constantes. ¿Cumplen estos tres gases el principio cero? Si lo hacen,
¿Cuáles son las funciones que se igualan en el equilibrio térmico? (Carranza)
Solución: Si lo cumplen, ya que existen dos ecuaciones que relacionan las variables de
estado de dos sistemas con uno de ellos, y esto implicará que todos los tres sistemas
van a estar ligados en el equilibrio por tres ecuaciones. Por tanto las funciones que se
igualan en el equilibro térmico son:
p2V2 = p1V1 − bp1 ⎫
p1V1 − p2V2 = bp1 ⎫
a
⎪
a
⎬→
⎬ → p2V2 = p1V1 − bp1 = + p3V3
p2V2 = + p3V3 ⎪
V3 ( p2V2 − p3V3 ) = a ⎭
V3
V3
⎭

5º

Tres gases 1, 2 y 3, cumplen las siguientes ecuaciones cuando se encuentran en
equilibrio térmico mutuo entre parejas:
α p2 + βV2
k p2 + hV2
ln V1 =
+ ln α
ln p3 =
+ ln β
hV3
α p1
donde α , β , k y h son constantes. Determinar las relaciones necesarias entre las
constantes para que se cumpla el principio cero y las funciones que definen la
temperatura empírica en cada sistema.
Solución: Realizamos un par de operaciones algebraicas:
k p + hV2
α p2 + βV2
ln p3 = 2
+ ln β
+ ln α
ln V1 =
hV3
α p1

ln

V1

α

=

α p1 ln

α p2 + β V2
α p1
V1

α

ln

= α p2 + β V2

p3

β

=

hV3 ln

k p2 + hV2
hV3

p3

β

= k p2 + hV2

Si las constantes k y h son proporcionales a α y β respectivamente, se cumplirá el
principio cero:
k = mα
h = mβ
Donde m es una constante de proporcionalidad. Finalmente por igualación y
simplificación obtenemos:
p
V
α p2 + βV2 = α p1 ln 1 = αV3 ln 3

α

β

Se define la temperatura empírica como θ = θ ( X , Y ) , en nuestro caso θ = θ ( p, V ) . Por
tanto las funciones que describen la temperatura empírica en los sistemas descritos son:
p
V
θ1 ( p1 , V1 ) = α p1 ln 1
θ 2 ( p2 ,V2 ) = α p2 + β V2
θ3 ( p3 ,V3 ) = αV3 ln 3

α

9

β
Ejercicios y problemas de Termodinámica I
6º

Los valores límites a p → 0 del producto pV en un termómetro de gas a volumen
constante, en contacto térmico con dos sistemas, θ 1 y θ 2, son 45.423 y 53.394 Pa·m3,
respectivamente. Calcúlese la relación de temperaturas T (θ 1)/T(θ 2), de los dos
sistemas. Si el sistema θ 1 es una célula de punto triple del agua, ¿cuál es la temperatura
T(θ 2)?
Solución: Partiendo de la definición de temperatura en nuestro caso:
θ = θ ( X , Y ) → θ = θ ( p,V ) = pV
Por tanto la relación de temperaturas:
T (θ1 ) p1V
p 45.423
=
= 1 =
= 0.851
T (θ 2 ) p2V p2 53.394

7º

Supóngase que se define una escala absoluta de temperaturas, T’, tal que la diferencia
entre el punto triple del agua y el cero absoluto son 300 grados. ¿Cuál sería la
temperatura del punto de ebullición del agua? (La temperatura del punto de ebullición
del agua en la escala absoluta habitual es 373.15K).
Solución: Considerando la antigua temperatura en Kelvin, la aplicamos para conocer
las temperaturas en la nueva escala absoluta. Utilizamos la siguiente expresión:
x
T ( x) = 273.16
x3
Colocamos la temperatura del punto triple x3 de la nueva escala absoluta de
temperaturas y consideraremos x como la temperatura de ebullición del agua en la
nueva escala, y T(x) , resultando la ecuación:
x
373.15
→ x = 300
= 409.81
373.15 = 273.16
300
273.16

8º

Un termómetro de gas, a volumen constante, se utiliza para determinar la temperatura
absoluta de un sistema. Las lecturas corregidas de la presión en el bulbo cuando, lleno
con diversas cantidades de gas, se pone en contacto con agua en el punto triple (p0) y
con el sistema (p) son:
p0 (mmHg)
1000.00
800.00
500.00
200.00
100.00
p (mmHg)
1296.02
1036.86
648.11
259.30
129.66
Calcúlese la temperatura del sistema en la escala de los gases ideales.
Solución: Partiendo de la temperatura del agua en el punto triple y de la expresión:
p
p
T = T0
= 273.16
p0
p0
Determinamos cual es la temperatura calculada del sistema para cada muestra de gas:
p0 (mmHg)
1000.00
800.00
500.00
200.00
100.00
p (mmHg)
1296.02
1036.86
648.11
259.30
129.66
T (K)
354.02
354.04
354.08
354.15
354.18
Haciendo una media aritmética de las temperaturas nos da como resultado aproximado
de la temperatura del sistema 354.09 K.

10
Julián Moreno Mestre
9º

Un termistor obedece la ley:
R (T ) = R 0 e B / T
donde R 0 y B son constantes. Cuando se calibra dicho termistor en el punto triple del
agua se encuentra una resistencia R 3 = 938.7 Ω y en el punto de vapor Rv = 1055.2 Ω.
Hallar la temperatura que mide ese termistor cuando su lectura es R = 1004,5 Ω.
Solución: Planteemos el sistema de ecuaciones exponenciales que se nos plantea para
calcular la temperatura que nos piden que mediría el termistor:
938.7 = R 0 e B / 273.16 ⎫ B = −119.26 K
⎬
1055.2 = R 0 e B / 373.16 ⎭ R0 = 1452.6 Ω
Y ahora deducimos la temperatura del termistor sustituyendo las constantes:
1004.5 = 1452.6e −119.26 / T → T = 324.24 K

10º Un termopar se calibra manteniendo una de sus dos soldaduras en el punto de hielo,
th = 0.00 ºC, y se obtiene:
ε (t ) = α t + β t 2
donde α = 0.04 mV/ºC y β = 4.00·10-6 mV/ºC2. Determinar la fuerza electromotriz que
produce dicho termopar cuando una soldadura se encuentra en el punto de vapor,
tv = 100.00 ºC y la otra en el punto de fusión del cinc, tZn = 419.53 ºC. (Carranza)
Solución: La fuerza electromotriz producida por el termopar entre las dos soldaduras
será debida a la diferencia de potencial generada en las dos soldaduras a distintas
temperaturas:
ε T = ε (tZn ) − ε (tv ) = 17.48 − 4.04 = 13.44 mV
11º La resistencia de un termómetro de platino es de 18.56 Ω en el punto de fusión del
estaño y de 9.83 Ω en el punto triple del agua. En estos mismos puntos las presiones de
un termómetro de H2 a volumen constante son 17.2 atm y 6.80 atm. Para otra cantidad
de gas en el bulbo se obtiene 1.85 atm y 1.00 atm. Además, las fuerzas electromotrices
de un termómetro cobre-niquel en los mismos puntos son 9.02 mV y 2.98 mV.
Determínese la temperatura de fusión del estaño en las distintas escalas termométricas.
Solución: Utilizaremos en los cálculos la expresión:
X
TSn = 273.16 Sn
X3
Calcularemos la temperatura ahora en cada termómetro:
18.56
Termómetro de platino: TSn = 273.16
= 515.76 K
9.83
12.7
= 510.17 K
Termómetro de gas H2: TSn = 273.16
6.8
1.85
= 505.35 K
Termómetro de otro gas: TSn = 273.16
1
9.02
Termómetro de Cu-Ni: TSn = 273.16
= 826.81 K
2.98

11
Ejercicios y problemas de Termodinámica I
12º La ecuación de estado de un gas perfecto es:
pV = nRT
Calcular el coeficiciente de compresibilidad isoterma y de dilatación cúbica.
Solución: Partiendo de las expresiones diferenciales de dichos coeficientes, derivando
determinaremos los mismos:
1
1
1 ⎛ ∂V ⎞
nR 1
1 ⎛ ∂V ⎞
nRT 1
= → β=
β= ⎜
κT = − ⎜
= → κT =
⎟ =
⎟ =
2
p
T
V ⎝ ∂T ⎠ p Vp T
V ⎝ ∂p ⎠T Vp
p
13º Un mol de gas real, a presiones moderadas, cumple con la ecuación:
p (V − b ) = RT

Donde R y b son constantes. Calcular el coeficiciente de compresibilidad isoterma y de
dilatación cúbica.
Solución: Partiendo de las expresiones diferenciales de dichos coeficientes, derivando
determinaremos los mismos:
R
RT
1 ⎛ ∂V ⎞
R
1 ⎛ ∂V ⎞
RT
β= ⎜
κT = − ⎜
→β=
⎟ = 2 → κ T = Vp 2
⎟ =
Vp
V ⎝ ∂T ⎠ p Vp
V ⎝ ∂p ⎠T Vp
14º Hallar el volumen final de 200 g de agua que sufre una compresión isoterma a 0 ºC
desde 1 bar hasta 300 bar.
κT = 0.50885 Gpas–1
Datos: ρ(0 ºC, 1 bar) = 999.84 kg/m3
Solución: Sabiendo que:
1 ⎛ ∂V ⎞
dV
κT = − ⎜
= −κ T dp → ln V = −κ T p + f (T )
⎟ →
V ⎝ ∂p ⎠T
V
Como es a temperatura constante, entonces f (T ) ≡ cte :
ln V = −κ T p + k
Partiendo de los datos del problema, determinamos el volumen inicial ocupado por los
200 g de agua:
m
200
= 0.200032 L
Vi = =
ρ 999.84
Y conocido el volumen inicial, convirtiendo las presiones y el coeficiente de
compresibilidad a pascales, obtendremos el volumen final:
ln Vi = −κ T pi + k ⎫
⎬ ln V f − ln Vi = −κ T ( p f − pi ) → V f = exp ln Vi − κ T ( p f − pi )
ln V f = −κ T p f + k ⎭

(

)

V f = 0.197012 L

12
Julián Moreno Mestre
15º Una vasija contiene 8.450 g de agua a 0ºC y el resto de la misma se llena con parafina.
Cuando el agua se congela a 0ºC, se expulsan 0.620g de parafina. La densidad de la
parafina a 20ºC es 0.800 g/cm3 y su coeficiente de dilatación 9.0·10-4K-1. Calcúlese la
densidad del hielo. Considérese la densidad del agua igual a 1g/cm3.
Solución: Integramos la ecuación diferencial del coeficiente de dilatación lineal de la
parafina:
1 ⎛ ∂V ⎞
∂V
= β ∂T → ln V = β T + f ( p ) → V (T ) = f ( p)·e β T
⎜
⎟ =β →
V ⎝ ∂T ⎠ p
V
A partir de la expresión de la densidad:
m
m − βT
e
ρ= =
V
f ( p)
pensemos que al ser un proceso isobárico entonces f (p) ≡ cte , y que la masa de la
parafina es también constante, por tanto la expresión de la densidad de la parafina es:
ρ (T ) = ke− β T
Conociendo a 20 ºC = 293 K la densidad de la parafina, averiguamos el valor de la
constante k:
−4 · 293

ρ (293) = ke−9·10

= 0.8 → k = 1.041 kg/L

La densidad de la parafina a 273 K es:

ρ (273) = 1.041e−9·10

−4 · 273

= 0.814 g/cm3

El volumen de parafina desalojado es:
m 0.620
V= =
= 0.763 cm3
ρ 0.814
La vasija a 0 ºC tenía inicialmente agua líquida y parafina, pero a 0 ºC el agua se
congela y cambia su densidad cambiando por tanto su volumen, el volumen de parafina
expulsado de la vasija es equivalente a la ganancia de volumen del agua al convertirse
en hielo. Por tanto el volumen del hielo será:
VH2O(s) = VH2O(l) + V = 8.450 + 0.763 = 9.212 cm3
Teniendo en cuenta que toda la masa de agua líquida se convierte en hielo, la densidad
del hielo es por tanto:
mH2O 8.450
ρ H2O(s) =
=
= 0.917 g/cm3
VH2O(s) 9.212

13
Ejercicios y problemas de Termodinámica I
16º Un hilo metálico de 0.0085 cm2 de sección, sometido a una fuerza de 20 N y a la
temperatura de 20 ºC, está situado entre dos soportes rígidos separados 1.2 m.
a) ¿Cuál es la fuerza recuperadora final si la temperatura se reduce a 8ºC?
b) Si además de la anterior disminución de temperatura los soportes se acercan
0.012 cm, ¿cuál será la fuerza recuperadora final?
Supóngase que en todo momento el hilo se mantiene rectilíneo y que el coeficiente de
dilatación lineal y el módulo de Young isotermo tienen valores constantes e iguales a
1.5·10-5 K-1 y 2·1011 N/m2, respectivamente.
Solución: A partir de las correspondientes relaciones diferenciales:
1 ⎛ ∂L ⎞
L ⎛ ∂F ⎞
βL = ⎜ ⎟
ϕ= ⎜ ⎟
L ⎝ ∂T ⎠ F
A ⎝ ∂L ⎠T
Procedemos a integrar las ecuaciones diferenciales:
1 ⎛ ∂L ⎞
∂L
L ⎛ ∂F ⎞
∂L ∂F
β L = ⎜ ⎟ → β L ∂T =
ϕ= ⎜ ⎟ →
=
L ⎝ ∂T ⎠ F
L
A ⎝ ∂L ⎠T
L Aϕ
F
F
β LT = ln L + g ( F )
ln L =
+ f (T ) → f (T ) = ln L −
Aϕ
Aϕ
Tras este proceso podemos distinguir la ecuación de estado más un término constante:
F
β LT = ln L −
+ cte
Aϕ
Con un valor inicial, determinamos el valor del término constante.
20
1.5·10−5 ·293 = ln1.2 −
+ cte → cte = −30227.1
0.85·10−6 ·2·1011
Por tanto:
F
β LT = ln L −
− 0.177808
Aϕ
Ahora respondemos a los apartados del ejercicio:
a) Tenemos que calcular la fuerza cuando la temperatura es 8 ºC.
F
1.5·10−5 ·281 = ln1.2 −
− 0.177808 → F = 50.6 N
0.85·10−6 ·2·1011
b) No solo la temperatura es de 8º C, sino que además cambia la longitud:
F
1.5·10−5 ·281 = ln(1.19988) −
− 0.177808 → F = 33.8 N
0.85·10−6 ·2·1011

14
Julián Moreno Mestre
17º Los coeficientes térmicos de dilatación isobárica β y compresibilidad isoterma κ T de
cierto gas real vienen dados por las expresiones:
Rv 2 (v − b)
v 2 (v − b) 2
β=
κT =
RTv 3 − 2a (v − b) 2
RTv 3 − 2a (v − b) 2
siendo a y b dos constantes características del gas y v el volumen molar.
a) Hállese la ecuación de estado de dicho gas, teniendo en cuenta que para
volúmenes molares altos el gas se comporta idealmente.
b) ¿Qué volumen ocupará un mol de gas ideal a 300 K y 14 atm suponiendo que a
= 3.61 atm·L2/mol2 y b = 0.0428 L/mol.
Solución: a) Integramos el coeficiente de compresibilidad isoterma:
v 2 (v − b ) 2
1 ⎛ ∂v ⎞
RTv 3 − 2a(v − b) 2
⎛ ∂p ⎞
=− ⎜ ⎟ →⎜ ⎟ =−
κT =
RTv 3 − 2a(v − b) 2
v ⎝ ∂p ⎠T
v 3 (v − b) 2
⎝ ∂v ⎠T

⎛ RT
2a ⎞
RT
a
dp = − ⎜
− 3 ⎟ dv → p =
+ 2 + f (T )
2
v ⎠
v−b v
⎝ (v − b)
Pero el coeficiente de dilatación isobárica presenta dificultades:
Rv 2 (v − b)
1 ⎛ ∂v ⎞
RTv 3 − 2a (v − b) 2
⎛ ∂T ⎞
= ⎜
→⎜
=
β=
⎟
⎟
RTv 3 − 2a (v − b) 2 v ⎝ ∂T ⎠ p
Rv 3 (v − b)
⎝ ∂v ⎠ p
Hay problemas ya que no es una ecuación de variables separadas, buscaremos con la
relaciones diferenciales un recurso que nos permita su integración:
⎛ ∂T ⎞ ⎛ ∂p ⎞
1 RTv 3 − 2a (v − b) 2 v − b
⎛ ∂T ⎞
⎛ ∂p ⎞ v − b
= −⎜
=
·
= −⎜ ⎟
⎟ ⎜ ⎟
⎜
⎟
3
v (v − b)
v−b
⎝ ∂v ⎠ p
⎝ ∂v ⎠T R
⎝ ∂p ⎠v ⎝ ∂v ⎠T R
⎛ ∂T ⎞ ⎛ ∂p ⎞
R
RT
⎛ ∂p ⎞ v − b ⎛ ∂T ⎞ v − b
−⎜
→⎜
→ dp =
dT =
+ g (v )
⎟ ⎜ ⎟ = −⎜ ⎟
⎟ =
R
v−b
v−b
⎝ ∂v ⎠T R
⎝ ∂p ⎠v ⎝ ∂v ⎠T
⎝ ∂p ⎠v
Por tanto la función de estado es:
RT
a
p=
+ 2 +k
v −b v
Según las condiciones del problema, si el volumen molar es muy grande, entonces la
ecuación de estado debe desembocar en la de un gas ideal, eso significa que:
RT
a
+ 2 + k ⎧v −b ≈ v
p=
RT
RT
k =0
→⎨
⇒ p=
+ k ⎯⎯⎯⎯ p =
→
v −b v
2
gas ideal
v
v
⎩a / v ≈ 0
v ≡ grande
Por tanto la ecuación de estado buscada es esta:
RT
a
p=
+ 2
v −b v
b) Si n = 1 entonces v = V , por tanto la ecuación de estado:
RT
a
p=
+ 2
V −b V
Sustituyendo llegamos a la ecuación (que desemboca en ecuación de tercer grado):
0.082·300 3.61
14 =
+
→ V = 1.65 L.
V − 0.0428 V 2
Para resolver la ecuación anterior necesitamos ayuda informática o métodos numéricos.

15
Ejercicios y problemas de Termodinámica I
18º Los coeficientes de dilatación cúbica y de compresibilidad isoterma de cierta sustancia
vienen dados por:
3aT 3
b
β=
κT =
V
V
siendo a y b constantes. Determínese la ecuación de estado que relaciona p, V y T.
Solución: Procedemos a integrar las ecuaciones diferenciales de los coeficientes de
dilatación cúbica y de compresibilidad isoterma:
3aT 3 1 ⎛ ∂V ⎞
b
1 ⎛ ∂V ⎞
= ⎜
β=
κT = = − ⎜
⎟
⎟
V
V ⎝ ∂T ⎠ p
V
V ⎝ ∂p ⎠T

⎛ ∂V ⎞
3aT 3 = ⎜
⎟
⎝ ∂T ⎠ p

⎛ ∂V ⎞
b = −⎜
⎟
⎝ ∂p ⎠T
b∂p = −∂V

3aT 3∂T = ∂V
3aT 4
= V + f ( p)
bp = −V + g (T )
4
3aT 4
V = g (T ) − bp
V=
+ f ( p)
4
Tras este proceso de integración, es ahora fácil determinar la ecuación de estado
resultante más un término constante fruto del proceso de integración:
3aT 4
V=
− bp + cte
4
19º El coeficiente de dilatación de un gas vale:

4T 3
T 4 −θ 4
donde θ es una constante, y el coeficiente de compresibilidad κ T = p −1 . Hallar la
ecuación de estado del gas.

β=

Solución: Solución: Procedemos a integrar las ecuaciones diferenciales de los
coeficientes de dilatación cúbica y de compresibilidad isoterma:
4T 3
1 ⎛ ∂V ⎞
1
1 ⎛ ∂V ⎞
β= 4 4= ⎜
κT = = − ⎜
⎟
⎟
T −θ
V ⎝ ∂T ⎠ p
p
V ⎝ ∂p ⎠T
∂p
∂V
4T 3
∂V
=−
∂T =
4
4
p
V
T −θ
V
4
4
ln p = − ln V + g (T )
ln(T − θ ) = ln V + f ( p)
ln V = − ln p + g (T )
ln V = ln(T 4 − θ 4 ) + f ( p)
Tras este proceso de integración, es ahora fácil determinar la ecuación de estado
resultante más un término constante fruto del proceso de integración:
ln V = ln(T 4 − θ 4 ) − ln p + cte
Para simplificarla, reagrupamos los logaritmos en un lado de la igualdad y nos
valdremos de sus propiedades:
Vp
⎛ Vp ⎞
= cte
= cte → 4
ln V − ln(T 4 − θ 4 ) + ln p = cte → ln ⎜ 4
4 ⎟
T −θ 4
⎝ T −θ ⎠

16
Julián Moreno Mestre
20º La ecuación de estado de una sustancia elástica ideal es:
⎛ L L2 ⎞
0
F = KT ⎜ − 2 ⎟
⎝ L0 L ⎠
donde F es la fuerza recuperadora, K es una constante y L0 (valor de la longitud a
fuerza recuperadora nula) es función solo de la temperatura. Determínese:
a) El coeficiente de dilatación lineal.
b) El módulo de Young de la sustancia, así como el valor de este último a fuerza
recuperadora nula.
Solución: a) Transformamos la ecuación de estado para facilitarnos la labor del cálculo
del coeficiente de dilatación lineal.
⎛ L L2 ⎞
⎛ L3 − L3 ⎞
0
0
F = KT ⎜ − 2 ⎟ = KT ⎜
→ FL0 L2 = KT ( L3 − L3 )
0
2 ⎟
⎝ L0 L ⎠
⎝ L0 L ⎠
Procedemos a derivar considerando F como constante:
∂
∂L
∂L
( FL0 L2 ) = ∂∂ KT ( L3 − L30 ) → 2 FL0 L ⎛ ∂T ⎞ = K ( L3 − L30 ) + 3L2 KT ⎛ ∂T ⎞
⎜
⎟
⎜
⎟
T
∂T
⎝
⎠F
⎝
⎠F
Y sabiendo que el coeficiente de dilatación lineal es:
1 ⎛ ∂L ⎞
βL = ⎜ ⎟
L ⎝ ∂T ⎠ F
lo sacaremos de la ecuación:
1 ⎛ ∂L ⎞
1 ⎛ ∂L ⎞
3
3
3
2 FL0 L2 ⎜
⎟ = K ( L − L0 ) + 3L KT ⎜
⎟
L ⎝ ∂T ⎠ F
L ⎝ ∂T ⎠ F

(

)

2 FL0 L2 β L = K ( L3 − L3 ) + 3L3 KT β L → β L L2 ( 2 FL0 − 3LKT ) = K ( L3 − L3 )
0
0

βL =

K ( L3 − L3 )
0

2 FL0 L2 − 3L3 KT
Mediante la ecuación de estado, eliminamos F:
K ( L3 − L3 )
( L3 − L30 )
0
→ βL =
βL =
2TL3 − L3T
⎛ L3 − L3 ⎞
0
2
3
0
L0 L − 3L KT
2 KT ⎜
2 ⎟
⎝ L0 L ⎠
b) Derivamos la ecuación inicial de nuestro problema:
⎛ 2 L2 1 ⎞
L ⎛ ∂F ⎞
L
ϕ = ⎜ ⎟ = KT ⎜ 30 + ⎟
A ⎝ ∂L ⎠T A
L0 ⎠
⎝ L
Y ahora para fuerza recuperadora nula L = L0:
⎛ 2 L2 1 ⎞
3KT
L0
ϕ = KT ⎜ 30 + ⎟ → ϕ =
A
A
⎝ L0 L0 ⎠

17
Ejercicios y problemas de Termodinámica I
21º El coeficiente de dilatación térmica del mercurio a 273 K es 1.8·10–4·K–1 . El
coeficiente de compresibilidad isotermo es 5.3·10–6 bar–1. Si una ampolla
completamente llena de mercurio, sellada y sin espacio vapor, se calienta desde 273 a
274 K, ¿cuál sería el aumento de presión desarrollado en el interior de la ampolla?
Solución: Partiendo de las relaciones diferenciales de los coeficientes:
1 ⎛ ∂V ⎞
1 ⎛ ∂V ⎞
β= ⎜
κT = − ⎜
⎟
⎟
V ⎝ ∂T ⎠ p
V ⎝ ∂p ⎠T
Procedemos integrando a buscar la ecuación de estado del mercurio:
1 ⎛ ∂V ⎞
∂V
1 ⎛ ∂V ⎞
∂V
β= ⎜
κT = − ⎜
⎟ → β∂T =
⎟ → −κ T ∂p =
V ⎝ ∂T ⎠ p
V
V ⎝ ∂p ⎠T
V
ln V = β T + f ( p)
ln V = −κ T p + g (T )

Podemos deducir ahora la ecuación de estado más un término constante:
ln V = β T − κ T p + k
Para calcular el incremento de presión despejamos la presión en la ecuación de estado:
k + β T − ln V
p=

κT

El incremento de presión es:
k + β T f − ln V k + β Ti − ln V β (T f − Ti )
∆p = p f − pi =
−
=
= 33.96 bar

κT

κT

κT

22º Demuestre que la ecuación:
⎛ ∂p ⎞ ⎛ ∂T ⎞ ⎛ ∂V ⎞
⎟ ⎜
⎟ = −1
⎜
⎟ ⎜
⎟
⎜
⎝ ∂V ⎠T ⎝ ∂p ⎠V ⎝ ∂T ⎠ p
se cumple en un gas ideal y en un gas de Clausius de ecuación p(V − b) = RT .

Solución: Sea la ecuación de estado del gas ideal:
pV = nRT
Sean sus derivadas parciales de la ecuación de arriba:
⎛ ∂T ⎞
V
nR
nRT
⎛ ∂p ⎞
⎛ ∂V ⎞
⎜
⎟ =
⎜
⎟ =
⎜
⎟ =− 2
V
p
⎝ ∂V ⎠T
⎝ ∂T ⎠ p
⎝ ∂p ⎠V nR
Por tanto, sustituyendo, simplificando y teniendo en cuenta la ecuación del gas ideal:
nRT
⎛ nRT ⎞ ⎛ V ⎞ ⎛ nR ⎞
= −1
⎟=−
⎜ − 2 ⎟⎜
⎟⎜
pV
⎝ V ⎠ ⎝ nR ⎠ ⎝ p ⎠
Demostrado

En el caso de la ecuación de un gas de Clausius:
⎛ ∂T ⎞
RT
V −b
R
⎛ ∂p ⎞
⎛ ∂V ⎞
⎜
⎟ =
⎜
⎟ =
⎜
⎟ =−
2
(V − b)
R
⎝ ∂V ⎠T
⎝ ∂T ⎠ p p
⎝ ∂p ⎠V
Ahora sustituyendo, simplificando y considerando la ecuación del gas de Clausius:
⎛
RT ⎞ ⎛ V − b ⎞ ⎛ R ⎞
RT
= −1
⎜−
⎟⎜ ⎟ = −
2 ⎟⎜
p (V − b)
⎝ (V − b) ⎠ ⎝ R ⎠ ⎝ p ⎠
Demostrado.

18
Julián Moreno Mestre
23º Un metal, cuyos coeficientes de dilatación cúbica y de compresibilidad isotérmica son
5·10–5 K–1 y 1.2·10–11 Pa–1 respectivamente, está a una presión de 105 Pa y a una
temperatura de 20 ºC. Si se le recubre con una capa gruesa y muy ajustada de una
sustancia de dilatación y compresibilidad despreciables:
a) ¿Cuál será su presión final al elevar su temperatura hasta 32 ºC?
b) ¿Cuál es la máxima temperatura que puede alcanzar el sistema si la presión más
alta que puede resistir la envoltura es 1.2·1018 Pa?
Solución: Partiendo de las relaciones diferenciales de los coeficientes:
1 ⎛ ∂V ⎞
1 ⎛ ∂V ⎞
β= ⎜
κT = − ⎜
⎟
⎟
V ⎝ ∂T ⎠ p
V ⎝ ∂p ⎠T
Procedemos integrando a buscar la ecuación de estado del mercurio:
1 ⎛ ∂V ⎞
∂V
1 ⎛ ∂V ⎞
∂V
β= ⎜
κT = − ⎜
⎟ → β∂T =
⎟ → −κ T ∂p =
V ⎝ ∂T ⎠ p
V
V ⎝ ∂p ⎠T
V
ln V = β T + f ( p)
ln V = −κ T p + g (T )

Podemos deducir ahora la ecuación de estado más un término constante:
ln V = β T − κ T p + k
Al recubrirse de una capa gruesa que le impide dilatarse, esto supone que V es
constante y por tanto rescribimos la ecuación de estado como:
k = β T − κT p
Determinamos ahora el valor de la constante considerando los valores iniciales del
problema:
k = 5·10−5 ·293 − 1.2·10−11 ·105 = 1.465·10−2
a) Su presión final al subir la temperatura hasta 32 ºC = 305 K será:
1.465·10−2 = 5·10−5 ·305 − 1.2·10−11 p → p = 5.01·107 Pas
b) La máxima temperatura que puede alcanzar el sistema con la presión más alta
soportable será:
1.465·10−2 = 5·10−5 ·T − 1.2·10−111.2·108 → T = 321.8 K
24º La constante dieléctrica relativa del agua varía con la temperatura de la forma:
ε r = a − bT
donde a y b son constantes. Sabiendo que:
⎛ ∂P ⎞
χ e = ⎜ ⎟ = ε 0 (ε r − 1)
⎝ ∂E ⎠T
Establecer una ecuación de estado para el agua líquida sometida al campo eléctrico.
Solución: A partir de la ecuación diferencial que cumple la susceptibilidad dieléctrica:
⎛ ∂P ⎞
χ e = ⎜ ⎟ = ε 0 (ε r − 1)
⎝ ∂E ⎠T
Buscaremos integrando la ecuación de estado que nos piden para el agua líquida:
⎛ ∂P ⎞
⎜
⎟ = ε 0 (a − bT − 1) → ∂ P = ε 0 (a − bT − 1)∂E → P = ε 0 (a − bT − 1)E + cte
⎝ ∂E ⎠T
Tengamos presente que si el campo es cero, entonces también es nula la polarización,
luego el término cte es nulo.
P = ε 0 (a − bT − 1)E + cte

19
Ejercicios y problemas de Termodinámica I
25º Un paramagnético ideal cumple la ley de Curie con la susceptibilidad dependiente solo
de la temperatura:
1 ⎛ ∂M ⎞
C
χm = ⎜
⎟ =
V ⎝ ∂H ⎠T T
donde C es una constante. Suponiendo que se conoce su coeficiente de dilatación, α,
que es una función exclusiva de la temperatura, determinar la influencia del campo
magnético sobre el coeficiente magnetotérmico que se define como:
⎛ ∂M ⎞
⎜
⎟
⎝ ∂T ⎠ H
Solución: Nos valdremos para obtener la relación de las segundas derivadas parciales
cruzadas, y nos valdremos además que el momento magnético del material M es una
variable de estado:
⎛ ∂ ⎛ ∂M ⎞ ⎞ ⎛ ∂ ⎛ ∂M ⎞ ⎞
∂χ
∂V
⎛ ∂
( χ mV ) ⎞ = ⎛ m ⎞ V + χ m ⎛ ⎞
⎜
⎜
⎟ ⎟ =⎜
⎜
⎟ ⎟ =⎜
⎟
⎜
⎟
⎜
⎟
⎠ H ⎝ ∂T ⎠ H
⎝ ∂T ⎠ H
⎝ ∂H ⎝ ∂T ⎠ H ⎠T ⎝ ∂T ⎝ ∂H ⎠T ⎠ H ⎝ ∂T
⎛ ∂ ⎛ ∂M ⎞ ⎞ ⎛ ∂χ m ⎞
χm
1⎞
⎛
V + V χmβ = V χm ⎜ β − ⎟
⎜
⎜
⎟ ⎟ =⎜
⎟ V + V χmβ = −
T
T⎠
⎝
⎝ ∂H ⎝ ∂T ⎠ H ⎠T ⎝ ∂T ⎠ H
Integramos una vez con respecto a H:
H
⎛ ∂ ⎛ ∂M ⎞ ⎞
⎛
1 ⎞⎞
⎛
dH = ∫ ⎜ V χ m ⎜ β − ⎟ ⎟ dH
∫ ⎜ ∂H ⎜ ∂T ⎟H ⎟
T ⎠⎠
⎝
⎠ ⎠T
⎝
⎝
0⎝
Ningún elemento del integrando del segundo miembro depende del campo magnético,
luego:
1⎞
⎛ ∂M ⎞
⎛ ∂M ⎞
⎛
⎜
⎟ =⎜
⎟ + V χm ⎜ β − ⎟
T⎠
⎝ ∂T ⎠ H ⎝ ∂T ⎠ H =0
⎝

20
Julián Moreno Mestre

21
Ejercicios y problemas de Termodinámica I

CAPÍTULO 2º
Trabajo en termodinámica. Relaciones entre las derivadas parciales. Primer
principio de la termodinámica. Coeficientes calorimétricos.

Resumen de teoría:
Trabajo:
Alambre metálico: δW = − fdl
Cambio de volumen: δ W = pdV
Torsión de un alambre: δ W = − µ dθ
Pila eléctrica reversible: δW = −ε dq
Polarización dieléctrico: δW = − EdP
Lámina superficial de un líquido: δ W = −σ ·dA
Sistema magnético: δW = − µ0 VHdH − µ0 VHdm
− µ0 VHdm Aumento del campo en el vacío.
− µ0 VHdH Aumento del momento magnético del material.
Relaciones entre las derivadas parciales:
Dada una función implícita f ( x, y, z ) = 0 entre tres variables termodinámicas, de las
cuales dos pueden seleccionarse como independientes, pudiéndose escribir esto de tres
formas alternativas:
x = x( y, z )
y = y ( x, z )
z = z ( x, y )
Según el teorema fundamental de la diferenciación:
⎛ ∂x ⎞
⎛ ∂x ⎞
⎛ ∂y ⎞
⎛ ∂y ⎞
dx = ⎜ ⎟ dy + ⎜ ⎟ dz
dy = ⎜ ⎟ dx + ⎜ ⎟ dz
⎝ ∂x ⎠ z
⎝ ∂z ⎠ x
⎝ ∂z ⎠ y
⎝ ∂y ⎠ z
⎛ ∂z ⎞
⎛ ∂z ⎞
dz = ⎜ ⎟ dy + ⎜ ⎟ dx
⎝ ∂x ⎠ y
⎝ ∂y ⎠ x
De estas tres ecuaciones, despejando, es posible deducir expresiones diferenciales del
tipo:
−1

⎛ ∂x ⎞ ⎛ ∂y ⎞
⎜ ⎟ =⎜ ⎟
⎝ ∂y ⎠ z ⎝ ∂x ⎠ z

⎛ ∂x ⎞ ⎛ ∂y ⎞
⎜ ⎟ ⎜ ⎟ =1
⎝ ∂y ⎠ z ⎝ ∂x ⎠ z

⎛ ∂x ⎞
⎛ ∂x ⎞ ⎛ ∂z ⎞ ⎛ ∂y ⎞
⎛ ∂x ⎞ ⎛ ∂z ⎞
⎜ ⎟ ⎜ ⎟ ⎜ ⎟ = −1
⎜ ⎟ = −⎜ ⎟ ⎜ ⎟
⎝ ∂z ⎠ y ⎝ ∂y ⎠ x
⎝ ∂y ⎠ z ⎝ ∂x ⎠ y ⎝ ∂z ⎠ x
⎝ ∂y ⎠ z
Imaginemos que dividimos por dw las tres relaciones diferenciales:
dx ⎛ ∂x ⎞ dy ⎛ ∂x ⎞ dz
dy ⎛ ∂y ⎞ dx ⎛ ∂y ⎞ dz
=⎜ ⎟
+⎜ ⎟
=⎜ ⎟
+⎜ ⎟
dw ⎝ ∂x ⎠ z dw ⎝ ∂z ⎠ x dw
dw ⎝ ∂y ⎠ z dw ⎝ ∂z ⎠ y dw
dz ⎛ ∂z ⎞ dy ⎛ ∂z ⎞ dx
=⎜ ⎟
+⎜ ⎟
dw ⎝ ∂y ⎠ x dw ⎝ ∂x ⎠ y dw
Y considerásemos como constante una de variables x, y, z, entonces obtendríamos
relaciones del tipo:
⎛ ∂x ⎞ ⎛ ∂x ⎞ ⎛ ∂w ⎞
⎛ ∂x ⎞ ⎛ ∂x ⎞ ⎛ ∂x ⎞ ⎛ ∂w ⎞
⎜ ⎟ =⎜
⎟
⎜ ⎟ =⎜ ⎟ +⎜
⎟
⎟ ⎜
⎟ ⎜
⎝ ∂y ⎠ z ⎝ ∂w ⎠ z ⎝ ∂y ⎠ z
⎝ ∂y ⎠ z ⎝ ∂y ⎠ w ⎝ ∂w ⎠ y ⎝ ∂y ⎠ z

22
Julián Moreno Mestre

Primer principio de la termodinámica:
dU = δ Q − δ W
Capacidades caloríficas en sistemas hidrostáticos y sus valores:
⎛ ∂H ⎞
⎛ ∂U ⎞
Cp = ⎜
CV = ⎜
⎟
⎟
⎝ ∂T ⎠V
⎝ ∂T ⎠ p
- Monoatómicos: CV = 1.5nR y C P = 2.5nR
- Biatómicos: CV = 2.5nR y C P = 3.5nR
Relación de Mayer:

α 2TV
⎛ ∂p ⎞ ⎛ ∂V ⎞
C p − CV = T ⎜
=
= nR
⎟ ⎜
⎟
κT
⎝ ∂T ⎠V ⎝ ∂T ⎠ p
Coeficiente adiabático y trabajo adiabático, transformaciones adiabáticas:
C
piVi − p f V f
γ= p
Wad =
γ −1
CV
TV γ −1 = cte

pV γ = cte

Coeficientes calorimétricos de un sistema p, V, T:
δ Q = C p dT + hdp
δ Q = CV dT + ldV

p1−γ T γ = cte

δ Q = λ dV + µ dp

C p − CV
⎛ ∂T ⎞
l = ( C p − CV ) ⎜
⎟ =
βV
⎝ ∂V ⎠ p

Cp
⎛ ∂T ⎞
⎟ =
⎝ ∂V ⎠ p βV

λ = Cp ⎜

⎛ ∂T ⎞
CV
κ
= CV T
⎟ =
β
⎝ ∂p ⎠V α p

µ = CV ⎜

CV − C p
⎛ ∂T ⎞
⎛ ∂V ⎞
⎛ ∂V ⎞
κ
= ( CV − C p ) T = l ⎜
h = ( CV − C p ) ⎜
⎟ =
⎟ = λ⎜
⎟ +µ
αp
β
⎝ ∂p ⎠V
⎝ ∂p ⎠T
⎝ ∂p ⎠T

23
Ejercicios y problemas de Termodinámica I

Problemas:
1º

Determine el trabajo realizado en la expansión isoterma de un mol de gas ideal a la
temperatura de 300 K cuando:
a) El gas se expansiona en una etapa, desde 10 atm a 1 atm, contra una presión
exterior constante de 1 atm.
b) La expansión se realiza en dos etapas. En la primera el gas se expansiona desde
10 a 5 atm, contra una presión exterior constante de 5 atm. En la segunda el gas
se expansiona desde 5 a 1 atm, contra una presión exterior constante de 5 atm. En
la segunda el gas se expansiona desde 5 atm a 1 atm, contra una presión exterior
constante de 1 atm.
c) La expansión se realiza en tres etapas: 1) desde 10 a 5 atm a presión exterior
constante de 5atm, 2) desde 5 a 2 atm a presión exterior constante de 2 atm, 3)
desde 2 a 1 atm a presión exterior constante de 1 atm.
d) La expansión se realiza en 9 etapas, desde 10 a 1 atm, reduciendo
progresivamente presión exterior en incrementos de 1 atm.
e) La expansión se realiza en un número infinito de etapas, haciendo que la presión
externa sea un infinitésimo inferior de la presión interna en cada etapa sucesiva.
Compárese entre sí los resultados obtenidos en cada uno de los apartados anteriores.
Solución: Dado que trabajamos con un solo mol de gas ideal. Resumimos la ecuación de
estado en:
pV = RT
Así calcularemos los volúmenes finales e iniciales en las etapas sugeridas por los
apartados según:
RT
V=
p
Siendo los volúmenes ocupados por el gas en función de la presión:
p(atm)
10
9
8
7
6
5
4
3
2
1
V (L)
2.46 2.73 3.07 3.51
4.1
4.92 6.15 8.20 12.3 24.6
Ahora realizaremos fácilmente los cálculos para los cuatro primeros apartados del
problema, observando como tomando diferentes caminos para llegar desde el estado
inicial al estado final los trabajos son distintos, lo cual es conforme con la teoría que el
trabajo en termodinámica no es función de estado.
a) Expansión en una etapa a presión exterior constante de 1
atm:

W=

V2

∫ p dV = p (V
2

2

2

− V1 ) = 22.14 atm·L = 2243.7 J

V1

b) Expansión en dos etapas:
Etapa 1: W1 =

V2

∫ p dV = p (V
2

2

2

− V1 ) = 1246.5 J

V1

Etapa 2: W2 =

V3

∫ p dV = p (V
3

3

3

− V2 ) = 1994.4 J

V2

El trabajo total es: W = W1 + W2 = 3240.9 J

24
Julián Moreno Mestre

c) Expansión en tres etapas:
Etapa 1: W1 =

V2

∫ p dV = p (V
2

2

2

− V1 ) = 1246.5 J

V1

V3

∫ p dV = p (V

Etapa 2: W2 =

3

3

3

− V2 ) = 1495.8 J

V2

Etapa 3: W3 =

V4

∫ p dV = p (V
4

4

4

− V3 ) = 1246.5 J

V3

El trabajo total es: W = W1 + W2 + W3 = 3988.8 J
d) Expansión en 9 etapas, calculamos los trabajos como en los
procedimientos anteriores:
W1 = p2 (V2 − V1 ) = 246.2 J
W2 = p3 (V3 − V2 ) = 275.5 J
W3 = p4 (V4 − V3 ) = 312.0 J
W4 = p5 (V5 − V4 ) = 358.6 J
W5 = p6 (V6 − V5 ) = 415.3 J
W6 = p7 (V7 − V6 ) = 498.4 J
W7 = p8 (V8 − V7 ) = 623.0 J
W8 = p9 (V9 − V8 ) = 830.7 J
W9 = p10 (V10 − V9 ) = 1246.0 J
9

El trabajo total es por tanto: W = ∑ Wi =4805.7 J
i =1

e) En este caso se van a ir produciendo cambios infinitesimales
de volumen y de presión. Hay que hacer por tanto el cálculo
del trabajo como un proceso isotermo y reversible:
V2
V2
⎡V ⎤
dV
W = ∫ pdV = RT ∫
= RT ln ⎢ 2 ⎥ = 5738.0 J
V
⎣ V1 ⎦
V1
V1
2º

Un mol de gas perfecto se expande isotérmica e irreversiblemente desde la presión
inicial de 10 atm contra una presión exterior de 6 atm, y una vez alcanzado el equilibrio
vuelve a expandirse bruscamente de modo isotérmico contra la presión exterior
constante de 3 atm hasta alcanzar de nuevo el equilibrio. Calcúlese en julios el trabajo
total realizado por el gas si la temperatura es en todo momento de 300 K.
Solución: Se trata de un proceso en dos etapas. Comenzamos aplicando la ecuación de
estado del gas ideal (para n = 1):
pV = RT
Para determinar los volúmenes de los estados inicial, intermedio y final
p(atm)
10
6
1
V (L)
2.46
4.1
24.6
Calculamos los trabajos para cada etapa:

Etapa 1: W1 =

V2

∫ p dV = p (V
2

2

2

− V1 ) = 996.8 J

V1

Etapa 2: W2 =

V3

∫ p dV = p (V
3

V2

25

3

3

− V2 ) = 2076.6 J
Ejercicios y problemas de Termodinámica I

El trabajo total es: W = W1 + W2 = 3073.4 J
3º

Considérense tres estados de equilibrio; A(3 bar, 1 L), B(1 bar, 3 L) y C(1 bar, 1 L), de
cierto sistema hidrostático. Calcule el trabajo realizado por el sistema cuando pasa del
estado A al B a través de cada una de las siguientes trayectorias del plano pV:
a) arco de circunferencia, con centro en C, que une A con B.
b) línea recta uniendo A con B.
c) Proceso isócoro (AC), seguido de un proceso isóbaro (CB).
Solución: En los tres apartados emplearemos la expresión integral del trabajo:
Vf

W=

∫ pdV

Vi

imponiendo los respectivos caminos de cada apartado:
a) Se trata de un camino de arco de circunferencia, en especial
medio arco de una circunferencia tal y como podemos
observar en el diagrama pV. El trabajo es el área sombreada.
Para calcular la relación entre la presión y el volumen
siguiendo el arco de circunferencia utilizamos la expresión
matemática de la circunferencia de radio R centrada en un
punto de coordenadas (xc, yc):
( x − xc ) 2 + ( y − yc ) 2 = R 2
Sustituimos las variables x e y por p y V, y además podemos comprobar que se trata de
una circunferencia de R = 2. Consideraremos además que esta ecuación que describe la
trayectoria pV de arco de circunferencia es además adimensional.
2
2
(V − Vc ) + ( p − pc ) = R 2

Despejando la presión p hasta describir un semiarco del primer cuadrante:
p = R 2 − (V − Vc ) + pc
2

y sustituyendo en la integral:
W=

∫(

VB

)

R 2 − (V − Vc ) + pc dV
2

VA

El cálculo de esta integral requiere de utilizar bastantes herramientas de cálculo, por ello
recurriendo a un libro de tablas de integrales podemos ver que las integrales del tipo:
x R 2 − x2 R 2
⎛x⎞
arcsin ⎜ ⎟
+
∫
2
2
⎝R⎠
y así realizar el cálculo. O bien recurrimos a Derive planteándole la integral:
R 2 − x 2 dx =

3

W =∫
1

(

)

22 − (V − 1) + 1 dV = π + 2 bar·L = 520.2 J
2

b) La trayectoria la determinamos con la expresión de la
ecuación de la recta que pasa por dos puntos:
x − x0
y − y0
=
x1 − x0 y1 − y0
Sustituyendo por nuestras variables x e y por nuestras variables
p y V:

26
Julián Moreno Mestre

V − VA
p − pA
V − VA
V −1
=
→ p=
( pB − pA ) + pA =
(1 − 3) + 3 = −V + 4
VB − VA pB − pA
VB − VA
3 −1
Integrando ahora:
3

VA

W=

VB

1

∫ pdV = ∫ −V + 4dV = 4 bar·L = 400 J

c) Viendo la gráfica pV planteada, obviamente solo hay que
calcular el trabajo en el proceso isóbaro (CB) y olvidarse de
calcularlo en el isócoro (AC), ya que en el es evidente que no
se produce trabajo.
W=

VB

∫ pdV = p (V

B

− VC ) = 1(3 − 1) = 2 bar·L = 200 J

VC

4º

Calcular el trabajo efectuado por un mol de gas que obedece a la ecuación de Van der
Waals al expansionarse desde el volumen V1 al volumen V2 en los siguientes casos:
a) Por vía reversible e isoterma.
b) Por vía irreversible a presión constante.
Determínese las expresiones del trabajo en ambos casos si el gas fuese ideal.
Solución: La ecuación de Van der Waals es:
a ⎞
RT
a
⎛
− 2
⎜ p + 2 ⎟ (V − b) = RT → p =
V ⎠
V −b V
⎝
a) Expansión isoterma reversible, es decir, que no es brusca, luego:
W=

V2

∫

V1

V

V2

2
a ⎞
V
⎛ RT
⎡a⎤
pdV = ∫ ⎜
− 2 ⎟ dV = RT [ ln(V − b) ]V2 + ⎢ ⎥
1
V −b V ⎠
⎣ V ⎦V1
V1 ⎝

⎛V −b ⎞ a a
W = RT ln ⎜ 1
⎟+ −
⎝ V2 − b ⎠ V1 V2

b) Se trata de una expansión a presión exterior constante:
W=

V2

∫ pdV → W = p(V

2

− V1 )

V1

Si el gas hubiese sido ideal, entonces la ecuación de estado sería:
RT
pV = RT → V =
p
y ambos trabajos serían (recontestando a los apartados):
a) Expansión isoterma reversible:
W=

V2

∫

V1

pdV =

V2

∫

V1

⎛V ⎞
RT
V
dV = RT [ ln(V )]V2 → W = RT ln ⎜ 2 ⎟
1
V
⎝ V1 ⎠

b) Expansión isobara:
W=

V2

∫ pdV → W = p(V

2

V1

27

− V1 )
Ejercicios y problemas de Termodinámica I

5º

Un mol de gas ideal evoluciona cuasiestáticamente desde
el estado A hasta el estado C. Determínese el trabajo
realizado cuando la evolución tiene lugar a lo largo de
los caminos I y II indicados en el diagrama pT de la
figura.
Solución: Empezaremos por el camino I. En este camino determinaremos la ecuación de
la recta que pasa por los puntos que marca A y C:
p − p1 T − T1
T −T
=
→ T = ( p − p1 ) 2 1 + T1
p2 − p1 T2 − T1
p2 − p1
Sustituimos en la ecuación de estado del gas ideal (para n = 1):
T −T
R
T − T RT
pV = RT → pV = R( p − p1 ) 2 1 + RT1 → V = ( p − p1 ) 2 1 + 1
p2 − p1
p
p2 − p1
p
Determinamos el diferencial de volumen:
⎡
⎤ dp
T −T
dV = ⎢ Rp1 2 1 − RT1 ⎥ 2
p2 − p1
⎣
⎦p
y procedemos a calcular ahora el trabajo realizado por el camino I:
V2
p2
⎡
⎤ dp ⎡
⎤ ⎛p ⎞
T −T
T −T
WI = ∫ p2 dV = ∫ p ⎢ Rp1 2 1 − RT1 ⎥ 2 = ⎢ Rp1 2 1 − RT1 ⎥ ln ⎜ 2 ⎟
p2 − p1
p2 − p1
⎣
⎦p
⎣
⎦ ⎝ p1 ⎠
V1
p1
⎡
⎤ ⎛p ⎞
T −T
WI = ⎢ Rp1 2 1 − RT1 ⎥ ln ⎜ 2 ⎟
p2 − p1
⎣
⎦ ⎝ p1 ⎠

Por el camino 2 el trabajo pasa por un proceso isotermo primero y por un proceso
isóbaro después. El trabajo realizado en el proceso isotermo (de A a B) es:
VB
VB
⎛V ⎞
dV
WIIIsotermo = − ∫ pdV = − RT1 ∫
= RT1 ln ⎜ 1 ⎟
V
⎝ VB ⎠
V1
V1
En el proceso isóbaro es:
V2

WIIIsóbaro = − ∫ pdV = − p (V2 − VB )
VB

El trabajo total en el camino 2 es:
⎛V ⎞
WII = WIIIsotermo + WIIIsóbaro = RT1 ln ⎜ 1 ⎟ − p (V2 − VB )
⎝ VB ⎠
Cambiando los volúmenes por los datos dados por el problema:
RT
RT
RT
V1 = 1
VB = 1
V2 = 2
p1
p2
p2
⎛p ⎞
WII = RT1 ln ⎜ 2 ⎟ − R(T 2 −T1 )
⎝ p1 ⎠

28
Julián Moreno Mestre
6º

La presión de un gas en un cilindro dotado de un pistón desplazable varia con el
volumen según la expresión p = C/V 2. Donde C es una constante. Si la presión inicial es
500 kPa, el volumen inicial es 0.05 m3, y la presión final, 200 kPa, determínese el
trabajo realizado en el sistema.
Solución: Pasamos a atmósferas las presiones y los volúmenes a litros:
pi = 4.936 atm
Vi = 50 L
p f = 1.974 atm

A partir de la ecuación de estado determinamos el volumen final:
C
p = 2 → C = piVi 2 = p f V f2 → V f = 79.057 L
V
Siendo la constante de la ecuación de estado:
C = piVi 2 = 12339.6 atm·L2
Calculamos ahora el trabajo:
Vf

W=

∫

Vf

pdV =

Vi

7º

V

f
C
⎡C ⎤
dV = − ⎢ ⎥ = 9188.6 J
∫ V2
⎣ V ⎦Vi
Vi

Un mol de agua se comprime reversiblemente en una prensa hidráulica, a la temperatura
constante de 20 ºC, desde la presión inicial de una atmósfera hasta la presión final de
cien atmósferas. Calcúlese el trabajo realizado en el proceso sabiendo que, a esta
temperatura, el coeficiente de compresibilidad isotermo y la densidad valen 45.30·10-6
atm-1 y 0.9982 g·cm-3, respectivamente.
Soluciones: Lo primero es determinar la ecuación de estado que rige este proceso:
1 ⎛ ∂V ⎞
∂V
κT = − ⎜
→ κ T p = − ln V + K → V = Ke−κT p → K = VeκT p
⎟ → κ T ∂p = −
V ⎝ ∂p ⎠T
V
Determinamos el volumen de agua a 1 atm de presión, con la densidad y partiendo del
número de moles y la masa molar del agua (18 g/mol):
m 18·n
V= =
= 1.80324·10−2 L

ρ

ρ

Determinamos ahora el volumen final con la ecuación de estado:

K = 1.80324·10−2 ·e 45.3·10

−6

= V f ·e45.3·10

−4

→ V f = 1.79517·10−2 L

Determinamos también el valor de la constante de la ecuación de estado:

K = 1.80324·10−2 ·e45.3·10
Diferenciamos la ecuación de estado:

−6

= 1.80332·10−2 L

K = VeκT p → 0 = eκT p dV + V κ T eκT p dp → dV = −V κ T dp =

K
κT p

e

κ T dp

Y con la expresión del trabajo:
V2

100

V1

1

W = − ∫ pdV = − ∫

29

K
p κ p κ T dp = − K κ T
e
T

100

∫ eκ
1

100

p
T

⎡ e −κ p (κ T p + 1) ⎤
dp = − Kκ T ⎢ −
⎥ = 0.412 J
κ T2
⎣
⎦1
T

p
Ejercicios y problemas de Termodinámica I
8º

El nitrógeno líquido es un material dieléctrico. En un cierto intervalo de temperaturas, la
polarización eléctrica o momento bipolar por unidad de volumen, P, está relacionado
con el campo E por:
P = ε 0 (23.8 − 4.16·10−4 E)E
donde P está expresado C·m–2, E en V·m–1 y ε 0 es la permitividad del vacío igual a
8.85·10–12 C2·N–1·m–2. Calcúlese el trabajo realizado por cm3 de nitrobenceno cuando el
campo eléctrico aumenta de 0 a 104 V·m-1 e indíquese si se da o se recibe trabajo del
sistema.
Solución: A partir de la expresión del trabajo para este tipo de sistemas dieléctricos:
P2

W = − ∫ EdP
P1

Determinamos el diferencial de polarización con la expresión del ejercicio:
P = ε 0 (23.8 − 4.16·10−4 E)E → P = ε 0 (23.8 − 8.32·10−4 E)dE
Por tanto el trabajo:
104
⎛
108
1012 ⎞
−9
W = − ∫ Eε 0 (23.8 − 8.32·10−4 E)dE = ε 0 ⎜ −23.8
+ 8.32·10−4
⎟ = 8.08·10 J
2
2 ⎠
⎝
0
9º

Una muestra de bismuto de masa 27.3 g, cuya susceptibilidad magnética a la
temperatura ambiente es χm = – 1.32·10– 5, se introduce en un solenoide que crea un
campo de inducción magnética uniforme de diez teslas. Determinar la imanación final de
la muestra y el trabajo que realiza.
Datos: µ0 = 4π·10– 7 T·m·A–1; densidad del bismuto ρ = 9.80 g·cm–3.
Solución: Sabiendo que la susceptibilidad magnética se define como:
M
χm =
H
Donde M es la imanación, y H el campo magnético en el material. Sabiendo que:
B
B
H= =
µ µ0 (1 + χ m )
Determinamos la imanación final:
χmB
M=
= −105.04 A·m −1
µ0 (1 + χ m )
El volumen de bismuto que tenemos es:
m
V = = 2.78·10−6 m3

ρ

La expresión del trabajo es:
M2

M2

V µ0 ⎡ M ⎤ 2
−3
W = − ∫ µ0 HVd M = −V ∫ µ0
dM =
⎢ 2 ⎥ = 1.46·10 J
χm
χm ⎣ ⎦M 1
M1
M1
M

M

30
Julián Moreno Mestre
10º Un sólido paramagnético ideal de volumen V adquiere, cuasiestáticamente, una
imanación m en presencia de un campo magnético externo variable. La temperatura del
sólido varía durante la imanación según la ley:
2

T = T0 e K m
siendo K una constante. Hállese el trabajo realizado en la imanación del sólido en
función de la temperatura.
Solución: En sistemas magnéticos la expresión del trabajo es:
m2

W = − ∫ µ0V Hdm
m1

A partir de la ecuación dada por el problema, determinamos el dm:
2
⎛T ⎞
dT
1 dT
T = T0 e K m → ln ⎜ ⎟ = K m 2 → 2Kmdm =
→ 2mdm =
T
K T
⎝ T0 ⎠
Y con la ley de Curie:
H
m=C
T
Procedemos a aplicarla a nuestro diferencial:
1 dT
H
1 dT
dT
2mdm =
→ 2C dm =
→ dm =
K T
K T
2CKH
T
Sustituimos en la expresión del trabajo:
m2
T
µV
µV T
dT
W = − ∫ µ0V Hdm = − ∫ µ0V H
= − 0 ∫ dT → W = − 0 (T − T0 )
2CK
2CKH
2CK T
m
T
1

0

0

11º Un mol de gas ideal está en equilibrio a 6 atm de presión y volumen 10 L. Se le enfría
isócoramente hasta alcanzar una presión igual a la mitad de su presión inicial. A
continuación se calienta a presión constante hasta que alcanza un volumen, Vf, tal que en
una compresión isoterma final regresa a su estado inicial.
a) Dibuje el proceso en un diagrama de Clapeyron (p,V)
b) Calcule el trabajo neto realizado en el ciclo.
Solución: Exponemos la información que tenemos en el
problema:
pV
TA = TC =
= 731.7 K
pA = 6 atm
VA = VB = 10 L
R
RT
pV
TB = B B = 365.85 K
VC = C = 20 L
pB = pC = 3 atm
R
pC
Solo realizan trabajo la etapa isoterma y la etapa isóbara. Por tanto:

W = Wisóbara + Wisócora = pB (VC − VB ) +

VA

∫

VC

⎛V
W = pB (VC − VB ) + RTC ln ⎜ A
⎝ VC

31

VA

pdV = pB (VC − VB ) + RTC ∫

⎞
⎟ = −11.59 atm/L = −1174 J
⎠

VC

dV
V
Ejercicios y problemas de Termodinámica I
12º Para comprimir adiabáticamente 2 moles de un gas ideal biatómico cuya temperatura
inicial es de 300 K, ha sido necesario suministrarle 800 J en forma de trabajo. ¿Cuál es
la temperatura final del gas?
Solución: En un proceso adiabático el incremento de energía interna es igual al trabajo
suministrado. Por tanto la temperatura final la determinamos mediante el primer
principio:
5
∆U = −W → nCV (T f − Ti ) = 2 8.31(T f − 300 ) = 800 → T f = 319.25 K
2
13º Un mol de gas ideal monoatómico se encuentra en un cilindro provisto de un pistón
móvil. Si la presión externa sobre el pistón se mantiene constante en 1 atm, ¿Qué
cantidad de calor debe aportarse al gas para aumentar su volumen de 20 a 50 litros?
Solución: El gas realiza una expansión isobara. La temperatura inicial y final será de:
pV f
pV
Tf =
= 609.8 K
Ti = i = 243.9 K
nR
nR
Determinamos el calor que es necesario aportarle mediante el primer principio:
3
Q = ∆U + W = CV ∆T + p∆V = R∆T + p∆V = 7600 J
2
14º Un recipiente de 15 m3 contiene radiación electromagnética en equilibrio con sus
paredes a la temperatura de 300 K. Dicha radiación se comporta como un sistema (gas
de fotones) con ecuaciones térmica de estado y energética dadas por las expresiones:
aT 4
p=
U = aVT 4
3
donde a = 7.56·10-16 J·m-3·K-4.
a) Calcúlese el calor absorbido por el sistema de radiación en un proceso isotermo
reversible en el que su volumen se duplica.
b) Obténgase, por otra parte, la ecuación que rige, en coordenadas (p,V ), los
procesos adiabáticos reversibles experimentados por dicho sistema.
Solución: a) Partiendo del primer principio de la termodinámica:
∆U = Q − W → Q = ∆U + W
Determinamos ahora el calor transferido a partir del cambio de energía interna y el
trabajo. Comenzamos por el trabajo, el cual es realizado por el sistema sobre los
alrededores, luego es positivo:
Vf

Vi

W=

Vf

Vi

∫ pdV = ∫

4aT 4
4aT 4
4aT 4
4aT 4
(V f − Vi ) =
(2Vi − Vi ) =
dV =
Vi
3
3
3
3

El cambio de energía interna depende en este caso del volumen:
∆U = U f − U i = aV f T 4 − aViT 4 = 2aViT 4 − aViT 4 = aViT 4
Por tanto el calor:
aV T 4 4aViT 4
Q = ∆U + W = aViT 4 + i =
= 1.225·10−4 J
3
3
b) Partiendo del primer principio con Q = 0 y diferenciando:

aT 4
dU = − dW → aT dV + 4aVT dT = − pdV → aT dV + 4aVT dT = −
dV
3
4

3

4

3

32
Julián Moreno Mestre

T
4T
dT
4dV
dT
4 dV
dV → 4VdT = −
dV → 4
=−
→ 4∫
=− ∫
3
3
3V
3 V
T
T
4
4 ln T = − ln V + cte → cte = ln (T 4V 4 / 3 ) → cte = T 4V 4 / 3
3
Y ahora a partir de la ecuación de estado que relaciona presión y temperatura:
aT 4
3p
3 p 4/3
p=
→T4 =
cte = T 4V 1/ 3 → cte =
V → cte = pV 4 / 3
3
a
a
15º Un mol de agua a 24 ºC y 1.013 bar se calienta a presión constante hasta 100 ºC. El
coeficiente de dilatación cúbica del agua vale 4·10–4 K–1. Calcúlese el cambio de
energía interna experimentado por el agua, considerando que la densidad media del
líquido en dicho intervalo de temperaturas es de 1 g·cm–3 y el calor específico medio a
presión constante vale 4.18·103 J·kg–1 ·K–1.
TdV + 4VdT = −

Solución: En el presente problema tenemos un exceso de datos para calcular el cambio
de energía interna. Nos basta con conocer el calos específico del agua, el incremento
de temperatura y la masa de agua (1 mol de agua = 18 g de agua) que tenemos:
∆U = mce ∆T = 5718.24 J
16º Un mol de gas biatómico experimenta cambios reversibles desde pi = 10 atm y
V1 = 10 L a pf = 1 atm, de acuerdo con los siguientes procesos:
a) V = cte
b) T = cte,
c) Adiabáticamente
Calcúlense W, Q, ∆U y ∆H en cada proceso. Represéntese los procesos en un
diagrama pV.
Solución: Determinamos primero la temperatura inicial con la ecuación de estado del
gas ideal:
pV
pV = RT → T =
= 1219.5 K
R
Como se trata de un proceso de expansión o contracción de un gas ideal biatómico,
entonces las funciones de estado entalpía y energía interna obedecerán a las
expresiones (con n = 1):
∆U = CV ∆T
∆H = C p ∆T

Donde:
5
7
R
Cp = R
2
2
a) Como es a V = cte, determinaremos la temperatura final:
pV
p f V f = RTi = piVi → V f = i i = 100 L
pf
como es un proceso isócoro, el trabajo es nulo. La entalpía y
la energía interna es:
∆U = CV ∆T = −22792.3 J
∆H = C p ∆T = −31909.5 J

CV =

El calor es igual al incremento de energía interna según el primer principio.

33
Ejercicios y problemas de Termodinámica I

b) Se trata de un proceso isotermo. Dado que no existen
transiciones de fase en el gas, el cambio de energía interna y
el de entalpía son nulos. Por el primer principio, el calor es
igual al trabajo. Determinaremos primero el volumen final:
pV
pV = RT → T = f i = 121.95 K
R
y ahora ya podemos calcular el calor y el trabajo:
Vf
Vf
⎛V ⎞
RT
Q = W = − ∫ pdV = − ∫
dV = − RT ln ⎜ f ⎟ = −23325.0 J
V
⎝ Vi ⎠
Vi
Vi
c) Al tratarse de un proceso adiabático, el gas se va regir (en
el proceso) por la ecuación de estado siguiente:
p1−γ T γ = cte
al tratarse de un gas ideal biatómico (cumple con la ecuación
de estado del gas ideal), el coeficiente adiabático γ = 1.4 .
Por tanto determinamos la temperatura final y el volumen final del gas:
cte = pi1−γ Ti1−γ = p1f−γ T f1−γ → T f = 631.6 K
p f V f = RT f → V f = 51.79 L
Determinamos ahora el cambio de energía interna y la entalpía del proceso:
∆U = CV ∆T = −12208.6 J
∆H = C p ∆T = −17092.1 J
El trabajo adiabático obedece a la expresión:
p V − p2V2
W= 1 1
= −12209.2 J
γ −1
Como es un proceso adiabático, el calor es cero.
17º Un cilindro vertical de paredes adiabáticas y 100 cm de altura está dividido en dos
partes por una membrana impermeable que se encuentra a 50 cm de la base. La parte
superior del cilindro está cerrada por un pistón adiabático sobre el que se ejerce una
presión exterior constante. Inicialmente la parte inferior está vacía, mientras que la
parte superior contiene un mol de gas ideal monoatómico a 300 K, encontrándose el
pistón a 100 cm de altura. En un momento determinado se rompe la membrana y,
consecuencia, el pistón desciende. Determínese la altura a la que se detiene el pistón
una vez que se ha alcanzado el equilibrio y el trabajo realizado sobre el gas.
Solución: Antes de la rotura de la membrana, las variables de
estado del sistema valen:
RT
V1 = 1 = 24.6 L
T1 = 300 K
p1 = 1 atm
p1
Después de la rotura de la membrana, la energía interna no
cambia, la temperatura es la misma, el volumen se duplica y la
presión es por tanto la mitad:
T2 = 300 K
p2 = 0.5 atm
V2 = 29.2 L
Pero al poco de romperse la membrana el pistón comienza a contraer el gas, y lo hace
de forma adiabática. Dado que es un gas monoatómico, el coeficiente adiabático del
gas es γ = 1.67 . El gas va a tener una presión final igual a la atmosférica (p3 = 1 atm).
Siguiendo las relaciones entre dos estados unidos por una adiabática determinaremos
el volumen final del gas:
p3V3γ = p2V2γ → V3 = 32.5 L
34
Julián Moreno Mestre

La altura que adquiere el pistón la calculamos mediante una regla de tres:
V1 h1
24.6 L 60 cm
= →
=
→ h3 = 66 cm
V3 h3
h3
32.5 L
El trabajo (adiabático) realizado sobre el gas es:
p V − p3V3
W= 2 2
= −11.77 atm·L= −1192.9 J
1− γ
18º Un gas ideal biatómico, encerrado en un cilindro de paredes adiabáticas, se calienta a
la presión constante de 2 bar mediante una resistencia de capacidad calorífica
despreciable. El volumen ocupado por el gas aumenta de 25 a 42 L en 6 minutos.
Calcúlese:
a) El cambio de energía interna experimentado por el gas.
b) El trabajo eléctrico suministrado a la resistencia.
c) La intensidad que circula por la resistencia de valor 100 Ω.
Solución: No es posible calcular ni el número de moles ni la temperatura de los
diferentes de estados. No obstante como es un proceso isóbaro:
pV1 = nRT1
pV2 = nRT2
Pasamos a atmósferas los 2 bares de presión, por tanto la presión es de 1.9738 atm.
Ahora respondemos a los apartados:

a) Utilizaremos en este apartado las relaciones anteriores de las variables de estado
para calcular el cambio de energía interna:
5
5
5
∆U = CV ∆T = nR(T2 − T1 ) = nRT2 − nRT1 = 8498 J
2
2
2
b) El trabajo realizado es:

W=

V2

∫ pdV = p(V

2

− V1 ) = 3399 J

V1

c) Calculamos el trabajo eléctrico realizado sobre el sistema por la resistencia:
∆U = We − W → We = ∆U + W = 11897 J
Por la Ley de Joule del calentamiento eléctrico de las resistencias:
We
We = I 2 Rt → I =
= 0.575 A
Rt
19º Partiendo de la ecuación calorimétrica δ Q = C p dT + hdp demuéstrese que en un

proceso adiabático se cumple:

γ ⎛ ∂p ⎞
⎛ ∂p ⎞
⎜
⎟ =
⎜
⎟
⎝ ∂T ⎠Q γ − 1 ⎝ ∂T ⎠V
Solución: Al ser un proceso adiabático δ Q = 0 , luego:
Cp
⎛ ∂p ⎞
0 = C p dT + hdp → C p dT = −hdp → ⎜
⎟ =
⎝ ∂T ⎠Q − h
Dado que el coeficiente calorimétrico es:
⎛ ∂T ⎞
h = −(C p − CV ) ⎜
⎟
⎝ ∂p ⎠V

35
Ejercicios y problemas de Termodinámica I

por tanto:

Cp
⎛ ∂p ⎞
⎜
⎟ =
⎝ ∂T ⎠Q

C p ⎛ ∂p ⎞
CV
⎛ ∂p ⎞
⎛ ∂p ⎞
⎛ ∂p ⎞
→⎜
⎟ =
⎜
⎟ →⎜
⎟ =C
⎟
C ⎜ ∂T ⎠V
⎛ ∂T ⎞
p
⎝ ∂T ⎠Q C p − CV ⎝ ∂T ⎠V
⎝ ∂T ⎠Q
− V ⎝
(C p − CV ) ⎜
⎟
CV CV
⎝ ∂p ⎠V
γ ⎛ ∂p ⎞
⎛ ∂p ⎞
⎜
⎟ =
⎜
⎟
⎝ ∂T ⎠Q γ − 1 ⎝ ∂T ⎠V
Demostrado
Cp

20º Demuéstrese que entre las capacidades a presión y volumen constante se cumple, para
todos los sistemas pVT:
⎡
⎛ ∂U ⎞ ⎤ ⎛ ∂V ⎞
C p − CV = ⎢ p + ⎜
⎟ ⎥⎜
⎟
⎝ ∂V ⎠T ⎦ ⎝ ∂T ⎠ p
⎣
Solución: Partiendo del primer principio de la termodinámica:
dU = δ Q − pdV
y de las relaciones diferenciales:
⎛ ∂U ⎞
⎛ ∂U ⎞
dU = ⎜
⎟ dT + ⎜
⎟ dV
⎝ ∂T ⎠V
⎝ ∂V ⎠T
Sustituyendo en el primer principio:
⎛ ∂U ⎞
⎛ ∂U ⎞
⎜
⎟ dT + ⎜
⎟ dV = δ Q − pdV
⎝ ∂T ⎠V
⎝ ∂V ⎠T
dV
⎛ ∂U ⎞ ⎛ ∂U ⎞ dV δ Q
=
−p
⎜
⎟ +⎜
⎟
dT
⎝ ∂T ⎠V ⎝ ∂V ⎠T dT dT
Si ahora consideramos constante la presión:
⎛ ∂U ⎞ ⎛ ∂U ⎞ ⎛ ∂V ⎞ ⎛ δ Q ⎞
⎛ dV ⎞
⎜
⎟ +⎜
⎟ ⎜
⎟ =⎜
⎟ − p⎜
⎟
⎝ ∂T ⎠V ⎝ ∂V ⎠T ⎝ ∂T ⎠ p ⎝ dT ⎠ p
⎝ dT ⎠ p
⎛ ∂U ⎞ ⎛ ∂V ⎞
⎛ ∂V ⎞ ⎛ δ Q ⎞ ⎛ ∂U ⎞
⎜
⎟ ⎜
⎟ + p⎜
⎟ =⎜
⎟ −⎜
⎟
⎝ ∂V ⎠T ⎝ ∂T ⎠ p
⎝ ∂T ⎠ p ⎝ dT ⎠ p ⎝ ∂T ⎠V
⎤
⎛ ∂V ⎞ ⎡⎛ ∂U ⎞
⎜
⎟ ⎢⎜
⎟ + p ⎥ = C p − CV
⎝ ∂T ⎠ p ⎣⎝ ∂V ⎠T
⎦

Demostrado.
21º Dos moles de un gas ideal (cV = 2.5R) realizan un proceso adiabático y cuasiestático
desde las condiciones p1 = 12 atm y V1 = 1 L hasta que su presión se reduce al valor
ambiental p2 = 1 atm. Hallar el trabajo intercambiado y los incrementos de energía
interna y de entalpía experimentados.
Solución: Al ser cV = 2.5R, esto implica que cp = 3.5R y γ = 1.4. El volumen final,
temperatura inicial y final es:
p
pV γ = cte → p1V1γ = p2V2γ → V2 = γ 1 V1 = 5.9 L
p2

36
Julián Moreno Mestre

p1V1
pV
= 73.17 K
T2 = 2 2 = 35.98 K
nR
nR
Como es adiabático, esto implica que el calor es nulo, y por tanto el trabajo es igual al
cambio de la energía interna. El trabajo y la energía interna es:
p V − p2V2 12·1 − 1·5.9
∆U = −W = − 1 1
=
= −15.25 atm·L = −1544.8 J
0.4
γ −1
El cambio en la entalpía es:
∆H = C p ∆T = −2163.34 J
T1 =

22º Una máquina realiza un ciclo triangular cuyos vértices en el plano (V, p) son A(1,1),
B(2,1) y C(1,2), expresándose la presión en atmósferas y el volumen en litros.
Determínese en rendimiento de este ciclo, admitiendo que la sustancia que lo recorre
es un gas ideal diatómico.
Solución: Representando gráficamente en un diagrama,
podemos ver que el trabajo total realizado equivale a
menos el área del triángulo.
1·1
W = = 0.5 atm/L = 50.65 J
2

Se trata de un gas ideal diatómico, esto implica que:
5
3
γ = 1.4
Cp = R
CV = R
2
2
Por el gráfico se puede fácilmente deducir que:
TC = TB = 2TA
En la etapa BA (isobara) del ciclo el calor liberado es igual al incremento de entalpía:
7
7
7
QBA = ∆H = nC p ∆T = nR(TA − TB ) = − nRTA = − pAVA = −3.5 atm·L = −354.55 J
2
2
2
En la etapa CB, las temperaturas final e inicial son iguales, el cambio de energía
interna es nulo, por el primer principio de la termodinámica establecemos que el calor
es igual al trabajo realizado en esta etapa. La trayectoria seguida es una línea recta:
p −1 V − 2
=
→ p −1 = 2 −V → p = 3 − V
2 −1 1− 2
El trabajo, y por tanto el calor, realizado en la trayectoria es:
Q =W =

VB

∫

VC

2

⎡
V2⎤
pdV = ∫ (3 − V )dV = ⎢3V − ⎥ = 1.5 atm·L = 151.95 J
2 ⎦1
⎣
1
2

En la etapa AC, el trabajo es cero al no haber incremento de volumen. El calor es igual
al cambio de energía interna:
5
5
5
QAC = ∆U AC = nCV ∆T = nR(TC − TA ) = nRTA = pAVA = 2.5 atm·L = 253.25 J
2
2
2
El calor absorbido o recibido por el ciclo es la suma de los calores positivos, siendo el
calor liberado el negativo. La diferencia entre el recibido y el desprendido como se
observa es igual al trabajo total. El rendimiento del ciclo es:
W
50.65
η=
=
= 0.125
Qabsorbido 151.95 + 253.25

37
Ejercicios y problemas de Termodinámica I
23º Un litro de aire se calienta a la presión atmosférica isobáricamente hasta duplicar su
volumen. Dedúzcase la variación de energía interna y el rendimiento de la
transformación. Se supone que el gas es perfecto y γ = 1.4.
Solución: El trabajo realizado es:
W = p∆V = p (V2 − V1 ) = 1 atm·L = 101.3 J

El incremento de energía interna y de entalpía es:
∆U = CV ∆T

∆H = C p ∆T

Por tanto y dado que:

γ=

Cp

= 1.4
CV
nC p ∆T = nCV ∆T + p∆V → 1.4nCV ∆T = nCV ∆T + p∆V → 0.4nCV ∆T = p∆V
∆H = ∆U + p∆V

0.4∆U = p∆V → ∆U =

p∆V
= 253.25 J
0.4

El calor es igual al incremento de entalpía:
nC p
1 ⎞
⎛
∆H = ∆U + p∆V → nC p ∆T =
∆ T + p ∆V → ⎜ 1 −
⎟ nC p ∆T = p∆V
1.4
⎝ 1.4 ⎠
p∆V
∆H = Q =
= 354.55 J
1
1−
1.4
El rendimiento es:
W 101.3
η= =
= 0.286
Q 354.55
24º Dos litros de un líquido que cumple la ecuación de estado:
V = V0 (1 − k ( p − p0 ) )

con k = 1.82·10–6 atm–1 se comprime desde un estado de equilibrio con p0 = 1 atm y
T0 = 273.2 K aplicándole adiabáticamente la presión constante p1 = 570.0 atm, hasta
que se iguala a la presión del líquido en el estado de equilibrio final. Determinar el
trabajo realizado y los incrementos de energía interna y de entalpía. (Carranza)
Solución: Partiendo de la expresión del trabajo para un proceso isobárico:
W = p1 (V0 − V1 )
De la ecuación de estado:
V1 = V0 (1 − k ( p1 − p0 ) ) = 1.99793 L

Por tanto el trabajo:

W = 570 ( 2 − 1.99793) = 1.18 atm·L = 119.6 J

Y al ser adiabático, el cambio de energía interna es:
∆U = −W = −119.6 J
La entalpía es:
H = U + pV
El cambio de la entalpía es:
∆H = H1 − H 0 = U 1 −U 0 + p1V1 − p0V0 = ∆U + p1V1 − p0V0

38
Julián Moreno Mestre
25º Dos moles de un gas ideal se encuentran en un cilindro adiabático cerrado por un
pistón libre, sin masa, también adiabático, que soporta inicialmente la presión
atmosférica y que tiene una superficie de 5.10 cm2. Una vez alcanzado el equilibrio en
esas condiciones, se deposita sobre el pistón un cuerpo de masa 10 kg, y se comprueba
que en el estado final de equilibrio el volumen del gas se reduce a las dos terceras
partes del inicial. Determinar el calor a volumen constante del gas.
Solución: Al depositarse una masa de 10 kg, esta contribuye a generar una presión
adicional de:
F mg
pm = =
= 1.923·104 pas = 1.899 atm
S
S
La presión total es la suma de la atmosférica y la ejercida por la masa, ósea 190.9 atm.
Partiendo de la expresión del trabajo (a presión constante) será igual al cambio de
energía interna:
∆U = nCv (T f − Ti ) = −W = − p f (V f − Vi )

Para la temperatura final e inicial, despejamos:
piVi = nRTi
Ti =

2
V f = Vi
3
pV
2 p f Vi
Tf = f f =
3 nR
nR

p f V f = nRT f
piVi
nR

Por tanto:
nCv (T f − Ti ) = − p f (V f − Vi ) → Cv =

Cv = R

(2 p

pf
f

− p f (V f − Vi )
n (T f − Ti )

− 3 pi )

⎛2
⎞
− p f ⎜ Vi − Vi ⎟
⎝3
⎠
=
⎛ 2 p f Vi piVi ⎞
n⎜
−
⎟
nR ⎠
⎝ 3 nR

= 2.38 R

26º Demuestre que en un proceso adiabático de un gas perfecto se cumple:
γ −1
⎡
⎤
⎛ p2 ⎞ γ
γ
⎢
H 2 − H1 =
RT1 ⎜ ⎟ − 1⎥
⎢⎝ p1 ⎠
⎥
γ −1
⎢
⎥
⎣
⎦
siendo T1 y p1 las coordenadas iniciales y T2 y p2 las finales.
Solución: Partiendo de la expresión del cambio de la entalpía con la temperatura:
∆H = H 2 − H1 = C p (T2 − T1 )

Y que:
C p − CV = R ⎫
CV
R
γR
γR
⎪
Cp
=
=
⎬ ⇒ C p = γ CV = γ CV = γ R
=γ ⎪
R
C p − CV C p
γ −1
−1
CV
⎭
CV
Conseguimos:
γR
H 2 − H1 =
(T − T )
γ −1 2 1

39
Ejercicios y problemas de Termodinámica I

Y siguiendo la adiabática:
1−γ

1−γ

⎛p ⎞
⎛p ⎞γ
1
p1−γ T γ = cte → p1 −γ T1γ = p1−γ T2γ → T2 = γ ⎜ 1 ⎟ T1γ = ⎜ 1 ⎟ T1
2
⎝ p2 ⎠
⎝ p2 ⎠
Sustituyendo conseguimos demostrar la igualdad:
1−γ
1−γ
⎡
⎤
⎛
⎞
⎛ p1 ⎞ γ
γ
γ R ⎜ ⎛ p1 ⎞ γ
⎟→ H −H =
⎢
RT1 ⎜ ⎟ − 1⎥
H 2 − H1 =
T1 − T1
2
1
⎜ ⎜ p2 ⎟
⎟
⎢⎝ p2 ⎠
⎥
γ −1
γ −1 ⎜ ⎝ ⎠
⎟
⎢
⎥
⎣
⎦
⎝
⎠

40
Julián Moreno Mestre

41
Ejercicios y problemas de Termodinámica I

CAPÍTULO 3º
Segundo principio de la termodinámica. Temperatura termodinámica y entropía.
Principio de aumento de entropía. Ecuación fundamental de la termodinámica.
Ecuaciones TdS.

Resumen de Teoría:
Rendimiento de cualquier máquina térmica:
W Q − Q2
η= = 1
Q1
Q1
Segundo principio de la termodinámica:
Teorema de Carnot: Ninguna máquina operando entre dos temperaturas dadas posee
un rendimiento superior al de una máquina de Carnot que funcionase entre las mismas
temperaturas.
Ciclo de Carnot: (T1>T2)
1º Expansión isoterma a T1, Q1>0
2º Expansión adiabática. Q = 0
3º Compresión isoterma a T2, Q2<0
4º Compresión adiabática Q = 0.
Rendimiento de un ciclo de Carnot:

η = 1−

T2
T1

Enunciado de Kelvin-Planck: No es posible ninguna transformación termodinámica
cuyo único resultado sea la absorción de calor de un solo foco y la producción de una
cantidad equivalente de trabajo.
Enunciado de Clausius: No es posible ningún proceso espontáneo cuyo único
resultado sea el paso de calor de un recinto a otro de mayor temperatura.
Teorema de Clausius (Entropía): Sea un sistema que verifica una transformación
cíclica durante la cual intercambia calor con una serie de recintos a las temperaturas
T1,…,Tn. Llamemos Q1,…,Qn las cantidades respectivas de calor intercambiadas por el
sistema, se verifica entonces que:
n
δQ
Q
∆S = ∫ i
∆S = ∑ i
Ti
i Ti
Principio de Caratheodory: Si un sistema se encuentra en un estado equilibrio
térmico, siempre existen otros estados próximos a aquel que no pueden alcanzarse
mediante procesos adiabáticos.

Entropía de mezcla:

∆S M = − nR ∑ xi ln xi

42
Julián Moreno Mestre

Ecuaciones de estado de la termodinámica:
⎛ ∂U ⎞
⎛ ∂p ⎞
⎛ ∂U ⎞
⎛ ∂S ⎞
dU = TdS − pdV → ⎜
⎟ =T⎜
⎟ − p → ⎜ ∂V ⎟ = T ⎜ ∂T ⎟ − p
⎝
⎠T
⎝
⎠V
⎝ ∂V ⎠T
⎝ ∂V ⎠T
⎛ ∂H ⎞
⎛ ∂H ⎞
⎛ ∂S ⎞
⎛ ∂V ⎞
dH = TdS + Vdp → ⎜
⎟ = T ⎜ ⎟ + V → ⎜ ∂p ⎟ = −T ⎜ ∂T ⎟ + V
⎝
⎠p
⎝
⎠T
⎝ ∂p ⎠T
⎝ ∂p ⎠T
Ecuaciones T·dS:
⎛ ∂V ⎞
TdS = C p dT − T ⎜
⎟ dp → TdS = C p dT − T β Vdp
⎝ ∂T ⎠ p
Tβ
⎛ ∂p ⎞
TdS = CV dT + T ⎜
dV
⎟ dV → TdS = CV dT −
κT
⎝ ∂T ⎠V
C
C ·κ
TdS = λ dV + µ dp → TdS = p dV + V T dp
βV
β
Aplicación a los gases ideales:
dT
dV
dT
dp
dS = CV
+R
dS = C p
−R
T
V
T
p

43
Ejercicios y problemas de Termodinámica I

Problemas:
1º

Para mantener un edificio la temperatura media de 18 ºC, su sistema frigorífico se ve
obligado a extraer de su interior 600.0 cal·s–1, mientras consume un trabajo eléctrico de
1.00 kW. Determinar el incremento de entropía por segundo que sufre el universo debido
al acondicionamiento del edificio sabiendo que el ambiente externo se encuentra a 35 ºC.
Solución: Consideraremos el edificio y el medio como dos focos cuyas temperaturas
permanecen inalterables todo el tiempo, y que sobre ellos trabaja una máquina frigorífica.
Llamemos Q1 al calor por unidad de tiempo extraído por la máquina del edificio, W al

trabajo consumido por unidad de tiempo, y Q2 al calor total producido por unidad de
tiempo. Por tanto:
Q2 = Q1 + W = 600 cal/s + 1000 J/s = 839.2 cal/s
La entropía por unidad de tiempo perdida por el edificio y cedida al medio es:
Q −600
Q 839.2
∆S1 = 1 =
= −2.06 cal/K·s
∆S2 = 2 =
= 2.72 cal/K·s
T1
291
T2
308
La producción total de entropía por unidad de tiempo es:
∆S = ∆S1 + ∆S2 = 0.66 cal/K·s
2º

En un calorímetro adiabático se mezclan 100.0 g de mercurio a 100.0 ºC con 50.0 g de
hielo a 0.0 ºC. Determinar el incremento de entropía del mercurio, del agua, y del universo
sabiendo que ambos líquidos son perfectamente inmiscibles.
Datos: Calor específico de mercurio ce,Hg = 0.033 cal·g –1·K –1.
Calor específico del agua c1 = 1.00 cal·g –1·K –1.
Calor latente de fusión del hielo Lfus = 80.0 cal· g –1.
Solución: Antes de empezar, verificaremos o descartaremos que la temperatura de
equilibrio no sea de 0 ºC (=273 K) dado que el calor latente de fusión del hielo es bastante
más elevado que el calor específico del mercurio. El calor necesario para fundir todo el
hielo y para cambiar 100 grados la temperatura del mercurio es:
Qfus = Lfus ·mhielo = 80·50 = 4000 cal
QHg,100 ºC→0 ºC = ce,Hg ·mHg ·∆T = 0.033·100·100 = −330 cal

Dado que el calor necesario para fundir el hielo es demasiado elevado y no es superado por
el de pasar mercurio de 100 ºC (=373 K) a 0 ºC (=273 K), esto implica que la temperatura
final es de 0 ºC (=273 K).
La masa de hielo y agua (dado que parte del hielo se derretirá calentando el mercurio)
permanece a temperatura constante, la entropía perdida es:
−QHg,100 ºC→0 ºC 330
∆Shielo+agua =
=
= 1.21 cal/K
T
273
El cambio de entropía del mercurio viene dado por:
f
δ QHg,100 ºC→0 ºC 273 ce,Hg mHg dT
⎛ 273 ⎞
∆S Hg = ∫
= ∫
= 0.033·100 ln ⎜
⎟ = −1.03 cal/K
T
T
⎝ 373 ⎠
i
373
La entropía total es:
∆S = ∆S hielo+agua + ∆S Hg = 0.18 cal/K

44
Julián Moreno Mestre
3º

Una viga metálica de 125 kg se encuentra en un ambiente a 12ºC y dispuesta para ser
colocada en un edificio en construcción. Por un descuido, la viga cae al suelo desde una
altura de 24 m sin sufrir daños. Posteriormente la viga se pone en su lugar, a 24 m de
altura, usando un motor que consume una potencia de 0.5 kW durante minuto y medio.
Calcular el incremento de entropía que ha experimentado el universo.
Solución: Dado que es constante la temperatura, el incremento de entropía es proporcional
a la energía perdida en el problema. La energía que se pierde es la potencial que tiene la
viga y aquella del motor que no es íntegramente empleada en subir la viga. Procedemos a
calcularlas comenzando por la potencial.
E p = mgh = 29400 J
Esta es la energía potencial que perderá la viga. El trabajo que produce el motor para subir
la viga es:
W = P·t = 45000 J
Teniendo en cuenta que de ese trabajo solo 29400 J se usarán para subir otra vez la viga,
quedando en forma de energía reutilizable (mientras no se vuelva a caer otra vez), el resto
(15600 J) se perderá seguramente en forma de calor. Por tanto la energía total perdida tras
la caída de la viga y su posterior colocación es:
E = 29400 + 15600 = 45000 J
La entropía generada (incrementada en el universo) es por tanto:
E 45000
∆S = =
= 157.9 J/K
T
285

4º

Una máquina térmica funciona entre un depósito que contiene 1·103 m3 de agua y un río a
temperatura constante de 10 ºC. Si la temperatura inicial del depósito es 100 ºC, ¿Cuál es
la cantidad máxima de trabajo que puede realizar la máquina térmica?
Solución: Consideraremos el río como un foco térmico cuya temperatura no cambiará en
todo el proceso. En cambio el depósito de agua va a descender su temperatura hasta
igualarla con la temperatura del río. Dado que se nos pide calcular el trabajo máximo que
puede realizar la máquina térmica que acoplaríamos entre el río y el depósito, esto significa
que la maquina realizaría ciclos de Carnot para obtener el máximo trabajo posible, lo cual
implica que el incremento de entropía del universo es nulo:
∆Suniverso = ∆S rio + ∆S deposito = 0 → ∆S rio = −∆S deposito

La entropía generada por el depósito es:
Trio
283
δ Qdeposito Trio ce mdT
dT
⎛ 283 ⎞
9
∆Sdeposito = ∫
= ∫
= 4.18·109 ∫
= 4.18·109 ln ⎜
⎟ = −1.154·10 J/K
373 ⎠
T
T
T
⎝
373
Tdepo
Tdep
El calor desprendido por el depósito es igual al cambio de energía interna experimentado
por este:
Qdeposito = ∆U deposito = mce ∆T = −106 ·4180·90 = −3.76·106 J
La entropía recibida por el río, y por tanto el calor recibido, es:
∆S rio = −∆Sdeposito = 1.154·109 J/K → Qrio = Trio ·∆S rio = 3.27·1011 J
El trabajo máximo que puede realizar la máquina térmica es:
Wmax = Qdeposito − Qrio = 4.92·1010 J

45
Ejercicios y problemas de Termodinámica I
5º

Una masa de 10 g de hielo a 0 ºC se ponen en contacto con una fuente térmica a 350.0 K,
hasta que alcanza un nuevo estado de equilibrio. Determinar la variación de entropía del
agua y del universo sabiendo el calor específico del agua líquida a presión constante
1.00 cal·g –1·K –1 y su calor latente de fusión de 80.0 cal· g –1.
Solución: El estado de equilibrio es agua líquida a 350 K. El cambio de entropía del agua
hasta ese nuevo estado de equilibrio lo determinaremos en dos etapas:

Primera etapa: El hielo se funde a temperatura constante recibiendo calor de la fuente
térmica. El cambio de entropía y el calor cedido por la fuente es:
Q
L ·m 10·80
Qfus = Lfus ·m = 10·80 = 800 cal
∆S1 = fus = fus =
= 2.9 cal/K
T
T
273
Segunda etapa: El agua liquida sube su temperatura desde los 273 K hasta los 350 K
recibiendo calor de la fuente de calor. El cambio de entropía y el calor cedido por la fuente
es:
Q273 K →373 K = m·ce ∆T = 10·1·77 = 770 cal
f

δQ

m·ce dT
⎛ 350 ⎞
= 10·1·ln ⎜
⎟ = 2.48 cal/K
T
T
⎝ 273 ⎠
273
i
El cambio total de entropía del agua es:
∆S H2O = ∆S1 + ∆S 2 = 5.38 cal/K
∆S 2 = ∫

350

=

∫

El calor total perdido por la fuente es equivalente al ganado por el agua, luego:
Qfuente = −QH2O = − ( Q fus + Q273 K →373 K ) = −1570 cal
y su cambio de entropía (recordando que su temperatura es constante) es:
Q
−1570
∆Sfuente = fuente =
= −4.48 cal/K
Tfuente
350
El cambio de entropía del universo es:
∆S Universo = ∆Sfuente + ∆S H2O = 0.9 cal/K
6º

Hallar el cambio de entropía del universo al transferirse 500 J de energía desde un foco de
400 K hasta un foco de 300 K.
Si hay una máquina térmica reversible entre los focos que recibe los 500 J del foco de
temperatura 400 K, hallar el trabajo producido.
Solución: En el primer apartado se transfieren 500 J desde el foco a 400 K al foco a 300 K.
La entropía generada en cada foco es:
Q −500
Q 500
∆S1 = 1 =
= −1.25 J/K
∆S2 = 2 =
= 1.6 J/K
T1
400
T2 300
El cambio de entropía del universo es:
∆Su = ∆S1 + ∆S 2 = 0.416 J/K

En el segundo apartado se instala una máquina térmica reversible. Al ser reversible el
cambio de entropía total del universo será cero, por tanto:
Q Q
∆Su = ∆S1 + ∆S 2 = 1 + 2 = 0
T1 T2

46
Julián Moreno Mestre

Sabemos que se transfieren 500 J a la máquina desde el foco a 400 K, por tanto el calor
transferido por la máquina al foco a 300 K será:
Q1 Q2 −500 Q2
+
=
+
= 0 → Q2 = 375 J
400 300
T1 T2
El trabajo realizado por la máquina es igual a la diferencia de los valores absolutos de los
calores cedido o ganado por los focos:
W = Q1 − Q2 = 125 J
7º

Un mol de gas ideal se comprime isotérmicamente aplicándole una presión constante de 15
atm, desde un volumen de 18.2 L hasta un volumen de 9.85 L a la temperatura de 13.2 ºC.
Determinar el incremento de entropía que experimenta el gas.
Solución: Es un proceso isotermo. Eso significa según el primer principio que:
δ Q = −δ W = pdV → Q = p(V2 − V1 ) = 125.25 atm·L = 12693 J
Por tanto el incremento de entropía del sistema es:
Q 12693
∆S = =
= 44.3 J/K
T 286.36

8º

Una máquina térmica funciona reversiblemente (ideal) entre 2 focos térmicos, uno de ellos
formado por 103 Kg de vapor de agua a 100ºC, y otro, por 103 Kg de hielo a 0ºC a la
presión de una atmósfera. ¿Cuál es el rendimiento total de la máquina? ¿Qué trabajo podrá
producir hasta que se funda todo el hielo? ¿Cuál es el incremento de entropía del universo
y de los focos?
Datos: Calor latente de fusión del hielo: Lfus = 80 cal·g-1.
Calor latente de vaporización del agua: LVap = 540 cal·g-1.
Solución: La máquina funcionará hasta que se haya producido un equilibrio de
temperatura entre los dos focos. Dado que la las masas de agua son iguales es de esperar
por el elevado calor latente de vaporización/licuación (LVap) del agua a 100 ºC, mucho mas
que el de fusión/solidificación (Lfus) del agua a 0 ºC, todo el hielo se derrita y la masa de
agua suba hasta los 100 ºC que sería la temperatura de equilibrio, licuándose en ese
proceso parte del gas. Hagamos unos cálculos para verificar esta posibilidad:
Qfus = Lfus ·m = 8·107 cal
Q0 ºC→100 ºC = mce ∆T = 10·107 cal
Qvap = Lvap ·m = 54·107 cal

Siendo tan elevado el calor de vaporización, superior a la suma de los otros dos, es pues
deducible que la temperatura de equilibrio es de 100 ºC. El calor transferido a la máquina
por el foco helado es por tanto:
Q1 = Qfus + Q0 ºC→100 ºC = 18·107 cal
Al saber que la maquina es reversible o ideal, es fácil suponer que el incremento de
entropía del universo es nulo. Esto supone que la entropía ganada por un foco es igual a la
entropía perdida por el otro. El incremento de entropía del foco 1 (el del hielo) lo
calcularemos en dos etapas:
1ª etapa: Es un proceso isotermo ya que el hielo se está fundiendo mientras se le suministra
calor, su incremento de entropía es:
Q
8·107
∆SI = fus =
= 2.93·105 cal/K
T1
273

47
Ejercicios y problemas de Termodinámica I

2ª etapa: Proceso a temperatura variable, desde 0 ºC (= 273 K) hasta los 100 ºC (= 373 K).
f
373
δQ
dT
⎛ 373 ⎞
5
∆S II = ∫
= mce ∫
= 106 ·1·ln ⎜
⎟ = 3.12·10 cal/K
T
T
⎝ 273 ⎠
273
i
La entropía total del foco 1 y del foco 2 es por tanto:
∆S1 = ∆SI + ∆S II = 6.05·105 cal/K
∆S2 = −∆S1 = −6.05·105 cal/K
Dado que el foco 2 permanece en todo momento a temperatura constante, es posible
conocer el calor total que se le transfirió:
Q
∆S2 = 2 → Q2 = ∆S2 ·T2 = (−6.05·105 )·373 = −22.56·107 cal
T2
Por tanto el trabajo realizado por la maquina es:
W = Q2 − Q1 = 4.57·107 cal
El rendimiento de la máquina es por tanto:
W
4.57·107
η=
=
= 0.203
Q2 22.56·107
Rendimiento del 20.3 %.
9º

Calcular la entropía molar de un gas B a 300K y una atmósfera de presión, sabiendo que
ese compuesto es sólido por debajo de 100 K, y que sublima a esa temperatura cuando la
presión es de 1 bar, con una entalpía de sublimación de 2000 cal/mol.
Datos: c p ( s ) = 2.5·10−4 T 3 − 2.4·10−10 T 6 cal/K·mol
c p ( g ) = 6 cal/K·mol
Solución: Dado que conocemos como crece su calor específico (a presión constante) en
estado sólido. Para calcular su entropía lo describiremos tres etapas tal y como indica la
figura:

La entropía molar es por tanto:
s = s0 + ∆s1 + ∆s2 + ∆s3
Calculemos la producción de cada etapa:
Primera etapa: Evolución del solido desde 0 K hasta 100 K.
−4 3
−10 6
f
δ q 100 c p ( s )dT 100 ( 2.5·10 T − 2.4·10 T ) dT
∆s1 = ∫
= ∫
= ∫
=…
T
T
T
0
0
i
100

⎡ 2.5·10−4 T 3 2.4·10−10 T 6 ⎤
2.5·10−4 ·1003 2.4·10−10 ·1006
−
−
= 43.3 cal/K·mol
…= ⎢
⎥ =
3
6
3
6
⎣
⎦0
Segunda etapa: Proceso isotermo, el sólido sublima y se convierte en el gas B.
q 2000
∆s2 = =
= 20 cal/K·mol
T2 100
Tercera etapa: Evolución del sólido desde 100 K hasta 200 K.
f
δ q 300 c p ( g )dT 300 6·dT
⎛ 300 ⎞
∆s1 = ∫
= ∫
= ∫
= 6 ln ⎜
⎟ = 6.6 cal/k·mol
T 100
T
T
⎝ 100 ⎠
100
i

48
Julián Moreno Mestre

Por tanto:
s = s0 + ∆s1 + ∆s2 + ∆s3 = s0 + 69.9
Dado que el gas convertido en sólido es puro, y tal vez forme sólidos cristalinos perfectos,
entonces el valor de s0 es con seguridad cero, por esto la entropía molar del gas a 300 K es:
s = 69.9 cal/K·mol
10º Un mol de gas ideal monoatómico a la temperatura de 399 K y 4 bar de presión, se
encuentra encerrado en un cilindro de paredes adiabáticas provisto de un pistón no
conductor del calor. Determinar la variación de entropía del gas al expansionarse
bruscamente contra una presión exterior de 1 bar.
Solución: Convertimos en atmósferas las presiones.
pi = 3.949 atm
p f = 0.987 atm

Como es una expansión brusca, es una expansión irreversible, y además cambian las tres
variables p,V y T. Para poder determinar el nuevos estado de equilibrio, recurrimos al
primer principio de la termodinámica y a la ecuación del gas ideal (con n = 1 moles):
dU = δ Q + δ W
pV = RT
Sabiendo que no es posible el intercambio de calor con el exterior, y que es una expansión
a presión constante pf:
dU = δ W → Cv ∆T = − p f (V f − Vi ) → Cv (T f − Ti ) = − p f (V f − Vi )
RTi
T
3
3
R(T f − Ti ) = − p f (V f − Vi ) = − RT f + p f
→ (T f − Ti ) = −T f + p f i
2
2
pi
pi
p
2 pf
3
2 0.987
3
5T f − 3Ti = 2 f Ti → T f =
·399 + ·399 = 279.29 K
Ti + Ti = ·
5 pi
5
5 3.949
5
pi
Una vez conocida la temperatura final del gas, determinamos el valor del volumen final e
inicial:
RT f 0.082·279.29
RT 0.082·399
Vi = i =
= 8.285 L
Vf =
=
= 23.203 L
3.949
pi
0.987
pf
Ahora ya podemos conocer el cambio de entropía del sistema:
279.29
23.203
δ Q dU − δ W
dT pdV
dT
dV
dT
dV
dS =
=
= Cv
−
= Cv
+R
→ ∆S = Cv ∫
+R ∫
T
T
T
T
T
V
T
V
399
8.285
3
⎛ 279.29 ⎞
⎛ 23.203 ⎞
∆S = 8.31·ln ⎜
⎟ + 8.31·ln ⎜
⎟ = −4.446 + 8.558 = 4.112 J/K
2
⎝ 399 ⎠
⎝ 8.285 ⎠

49
Ejercicios y problemas de Termodinámica I
11º Un número n de moles de un gas ideal experimentan una expansión libre y adiabática en el
vacío (experimento de Joule). Expresar en términos de la temperatura y volumen inicial y
final el incremento de entropía del sistema. Calcular cuanto vale la entropía molar si el
volumen final es doble que el volumen inicial.
Solución: Al ser una expansión en vacío no se produce ningún cambio en la energía
interna del gas, luego es un proceso a temperatura constante. El primer principio de la
termodinámica nos establece entonces que:
δ Q = −δ W = pdV
Y por tanto:
V
V
Vf
Vf
f
⎛V ⎞
δ Q f pdV 1 f
1 nRT
dV
dV = nR ∫
∆S = ∫
=∫
= ∫ pdV = ∫
= nR ln ⎜ f ⎟
T
T
T Vi
T Vi V
V
⎝ Vi ⎠
i
Vi
Vi

Si el volumen final es doble que el inicial, entonces la entropía molar es:
⎛ 2V ⎞
∆S
∆s =
= R ln ⎜ i ⎟ = 8.31·ln 2 = 5.76 J/K·mol
n
⎝ Vi ⎠
12º Un cilindro vertical de paredes adiabáticas y provisto de un pistón no conductor del calor,
está dividido por una pared fija y diatérmica. La parte superior contiene 10 moles de un gas
ideal y la inferior una mezcla en equilibrio a 1.013 bar de 100 g de agua y 100 g de hielo.
Se introduce lentamente el pistón hacia dentro hasta que la presión del gas se duplica.
¿Cuál es la variación de entropía del gas, de la mezcla y del universo en este proceso?
Solución: La situación de equilibrio inicial nos indica que la
temperatura inicial es de 0 ºC ya que una mezcla de agua + hielo solo
coexiste a esa temperatura a la presión de 1.013 bar = (1 atm). Antes de
abordar la resolución del problema, comprobaremos si el calor necesario
para fundir el hielo es mayor que el proceso isotermo (con T = 273 K)
de comprimir el gas de la parte de arriba:
V /2
V /2
dV
W = − ∫ pdV = − nRT ∫
= nRT ln 2 = 15725 J
V
V
V

El calor necesario para fundir el hielo es igual a:
Q f = L f ·m = 33440 J
Esto significa que todo el proceso es isotermo. La entropía perdida por el gas por el
traspaso en forma de calor a la masa de agua + hielo proveniente del trabajo suministrado
por el exterior es:
−Q −W
15725
∆Sgas =
=
=−
= −57.6 J/K
T
T
273
La entropía ganada por el agua + hielo es:
Q 15725
∆Sagua+hielo = =
= 57.6 J/K
T
273
La entropía del universo es:
∆Suniverso = ∆Sgas + ∆Sagua+hielo = 0

50
Julián Moreno Mestre
13º Un mol de gas ideal monoatómico recorre un ciclo de Carnot
reversible, como el indicado en la figura, con
V1 = 20 L,
V2 = 40 L, T1 = 300 K y T3 = 200 K.
Calcúlense ∆p, ∆V, ∆T, ∆U, ∆H y ∆S en cada etapa del ciclo.

Solución: Mediante la ecuación de estado del gas ideal y de las adiabáticas, determinamos
los valores de las variables p, V y T:
pV = RT
VT γ −1 = cte
Al ser gas monoatómico γ = 1.67.
El estado 1 es el estado inicial, luego los valores p, V y T son:
RT
T1 = 300 K
V1 = 20 L
p1 = 1 = 1.23 atm
V1
Al estado 2 se llega mediante un proceso isotermo desde el estado 1:
RT
T1 = T2 = 300 K
V2 = 40 L
p2 = 2 = 0.615 atm
V2

Al estado 3 se llega mediante un proceso adiabático desde el estado 2:
RT
T3 = 200 K
V2T2γ −1 = V3T3γ −1 → V3 = 52.49 L
p3 = 3 = 0.312 atm
V3
Al estado 4 se llega mediante un proceso isotermo desde el estado 3 y está conectado con
el 1 mediante un proceso adiabático:
RT
T4 = 200 K
p4 = 4 = 0.625 atm
V4T4γ −1 = V1T1γ −1 → V4 = 26.24 L
V4
Tras estos cálculos podemos calcular los cambios de cada una de las variables en las
respectivas etapas.
Etapa 1 a 2:
∆T1→ 2 = 0

∆V1→2 = 20 L

∆p1→2 = −0.615 atm

3
5
R∆T1→2 = 0 J
∆H1→2 = C p ∆T1→2 = R∆T1→2 = 0 J
2
2
V2
V2
⎛V ⎞
W
1
1 RT
dV = R ln ⎜ 2 ⎟ = 5.76 J/K
= 1→2 = ∫ pdV = ∫
T
T V1
T V1 V
⎝ V1 ⎠

∆U1→2 = Cv ∆T1→2 =
∆S1→2 =

Q1→2
T

Etapa 2 a 3:
∆T2→3 = −100 K

∆V2→3 = 12.49 L

∆p2→3 = −0.303 atm

3
R∆T2→3 = −1246.5 J
2
5
∆H 2→3 = C p ∆T2→3 = R∆T2→3 = −2077.5 J
2
Por ser un proceso adiabático reversible ∆S2→3 = 0
∆U 2→3 = Cv ∆T2→3 =

51
Ejercicios y problemas de Termodinámica I

Etapa 3 a 4:
∆T3→4 = 0

∆V3→4 = −26.25 L

∆p3→4 = 0.313 atm

3
5
R∆T3→4 = 0 J
∆H 3→4 = C p ∆T3→4 = R∆T3→4 = 0 J
2
2
V3
V3
⎛V ⎞
W
1
1 RT
= 3→4 = ∫ pdV = ∫
dV = R ln ⎜ 3 ⎟ = −5.76 J/K
T
T V4
T V4 V
⎝ V4 ⎠

∆U 3→4 = Cv ∆T3→4 =
∆S3→4 =

Q3→4
T

Etapa 4 a 1:
∆T4→1 = 100 K

∆V4→1 = 12.49 L

∆p4→1 = −0.303 atm

3
R∆T4→1 = 1246.5 J
2
5
∆H 4→1 = C p ∆T4→1 = R∆T4→1 = 2077.5 J
2
Por ser un proceso adiabático reversible ∆S4→1 = 0
∆U 4→1 = Cv ∆T4→1 =

14º Un mol de gas ideal se expansiona isotérmicamente, a 27 ºC, desde 20 a 40 litros en tres
procesos diferentes:
a) El primero se efectúa, de forma reversible, reduciendo lentamente la presión sobre
el pistón hasta que se alcanza el valor final pext.
b) En el segundo, la presión disminuye bruscamente hasta su valor final pext.
c) En el tercero, el gas ocupa inicialmente un volumen de 20 litros; repentinamente y
por rotura de una fina membrana, se deja que se expansione en el vacío hasta los 20
litros restantes.
Calcúlense en cada proceso Q, W, ∆U y ∆S del gas, del foco y del sistema total aislado
adiabáticamente (gas+foco).
Solución: Primero determinamos las presiones de los estados inicial y final de equilibrio:
RT
RT
pi =
= 1.23 atm
pf =
= 0.615 atm
Vi
Vf
a) Reducción lenta de la presión (isoterma reversible). El trabajo es:
Vf
Vf
⎛V ⎞
dV
W = ∫ pdV = RT ∫
= RT ln ⎜ f ⎟ = 1728 J
V
⎝ Vi ⎠
Vi
Vi

Al ser expansión isoterma de un gas ideal, esto significa que el incremento de energía
interna es cero. Por el primer principio estableceremos por tanto:
Q = W = 1728 J
El cambio de entropía de este proceso es:
∆S =

Q
= 5.76 J/K
T

b) El gas se expande en este caso a presión exterior constante, el trabajo es por tanto:
W = p f (V f − Vi ) = 0.615·(40 − 20) = 1240 J
El cambio de energía interna vuelve a ser cero ya que las temperaturas iniciales y finales
son idénticas. Por tanto el calor es:
Q = W = 1240 J

52
Julián Moreno Mestre

La entropía es:
f

∆S = ∫
i

δQ
T

f

= −∫
i

δW
T

Vf

=

∫

Vi

f
⎛V ⎞
pdV
dV
= nR ∫
= nR ln ⎜ f ⎟ = 5.76 J/K
T
V
⎝ Vi ⎠
Vi

V

c) El gas se expande 20 L en vacío. Aquí el trabajo es nulo, nulo también es el cambio de
energía interna al ser constante la temperatura, y el calor también es cero por el primer
principio. Pero el cambio de entropía es:
Vf
Vf
f
f
⎛V ⎞
δQ
δW
pdV
dV
∆S = ∫
= −∫
=∫
= nR ∫
= nR ln ⎜ f ⎟ = 5.76 J/K
T
T
T
V
⎝ Vi ⎠
i
i
Vi
Vi

Aunque Q y W sean cero, los δ Q y δ W no son diferenciales exactas y pueden no ser nulos.
15º Cierto sistema hidrostático tiene isotermas dadas por pV 2 = cte y una energía interna dada
por U = pV/2. Dicho sistema describe un ciclo A → B → C → A en tres etapas, siendo el
proceso A → B adiabático reversible, el proceso B → C adiabático irreversible y el proceso
C → A isotermo reversible. Calcúlese el calor intercambiado por el sistema y el cambio de
entropía de este en cada uno de los procesos, en función de las coordenadas de cada punto.
Solución: En la etapa de A → B al ser adiabática reversible no existe intercambio de calor
y el incremento de entropía es nulo. En la segunda etapa B → C, que es adiabática
irreversible tampoco se intercambia calor con los alrededores, pero la entropía es distinta
de cero. No tenemos forma exacta de calcularla con los datos de la etapa B → C. En la
última etapa C → A estamos ante un proceso isotermo reversible, que devuelve al punto de
partida al sistema hidrostático. En este proceso:
QC→A = ∆U C→A − WC→A
Donde:
pV
pV
∆U C→A = U A − U C = A A − C C
2
2
Teniendo en cuenta la ecuación que describe las isotermas:
2
pV 2 = pAVA = pCVC2 = cte
Determinamos el trabajo en la etapa C → A:
VA
VA
VA
1 ⎞ pCVC2 pCVC2
dV
2
2 ⎡ −1 ⎤
2⎛ 1
WC → A = ∫ pdV = pCVC ∫ 2 = pCVC ⎢ ⎥ = pCVC ⎜ − ⎟ =
−
=…
V
VC
VA
⎣ V ⎦VC
⎝ VC VA ⎠
VC
VC

… = pCVC −

Por tanto:

pCVC2
p V2
= pCVC − A A → WC → A = pCVC − pAVA
VA
VA

pV
p V − pAVA
pAVA pCVC
pV
−
+ pCVC − pAVA = C C − A A = C C
2
2
2
2
2
La entropía de este proceso (isotermo C → A) es:
p V − pAVA
Q
∆SC→A = C→A → ∆SC→A = C C
2TA
TA
Dado que el proceso total A → B → C → A es reversible, el balance total de entropía es
nulo para el sistema hidrostático, y que la entropía perdida en el proceso C → A es
equivalente a la entropía ganada por B → C, por tanto:
p V − pCVC
∆SB→C = −∆SC→A = A A
2TA
QC→A = ∆U C→A + WC→A =

53
Ejercicios y problemas de Termodinámica I
16º Un sistema tiene una capacidad calorífica a volumen constante dada por:
CV = AT 2
donde A = 0.0418 J·K–3. El sistema se encuentra inicialmente a 200 ºC y puede ser
enfriado a 0 ºC mediante uno u otro de los siguientes procesos:
a) Por contacto directo con un foco térmico a esa temperatura.
b) Haciendo funcionar una máquina térmica reversible entre el sistema y el foco
térmico.
Determínese, en cada caso, el trabajo obtenido y los cambios de entropía del sistema, del
foco y del universo.
Solución: En el primer proceso tenemos que por contacto directo el cambio de energía
interna experimentado por el sistema es igual al calor transferido. No se convierte nada en
trabajo, ya que no experimenta un cambio de volumen ni existe una máquina térmica que
convierta calor en trabajo. Luego:
273

⎡T 3 ⎤
⎡ 2733 4733 ⎤
6
∆U = Q = ∫ AT dT = A ⎢ ⎥ = 0.0418 ⎢
−
⎥ = −1.191·10 J
3 ⎦
⎣ 3 ⎦ 473
⎣ 3
473
El cambio de entropía experimentado por el sistema, el foco y el universo es por tanto:
273

2

273

∆S S =

∫

473

δQ

273

273
⎡T 2 T 2 ⎤
AT 2
= ∫
dT = A ∫ TdT = A ⎢ − ⎥ = −3118 J/K
T
T
2 ⎦ 473
⎣2
473
473
6
−QS 1.191·10
Q
∆S F = F =
=
= 4362 J/K
273
TF
TF
∆SU = ∆S F + ∆S S = −2079 + 4369 = 1244 J/K
273

En el segundo proceso nos encontramos con que parte del calor transferido a la máquina
térmica es convertido en trabajo y el resto cedido en forma de calor. Por tanto el calor
cedido por el sistema menos el calor cedido al foco es igual al trabajo:
QS − QF = W
Dado que es una máquina reversible la entropía del universo es cero, por tanto la entropía
ganada al absorber calor del sistema es igual a la entropía perdida al desprender el calor al
foco (de igual magnitud pero signo contrario según los convenios de signos):
273
273
δQ
AT 2
dT = 3118 J/K
∆SU = ∆S S + ∆S F = 0 → ∆S F = −∆S S → ∆S F = − ∫
=−∫
T
T
473
473
Por tanto el calor cedido al foco es:
QF = T ·∆S F = 8.51·105 J
El trabajo es por tanto:
W = QS − QF = 1.191·106 − 8.51·105 = 3.4·105 J

54
Termodinámica problemas resueltos
Termodinámica problemas resueltos
Termodinámica problemas resueltos
Termodinámica problemas resueltos
Termodinámica problemas resueltos
Termodinámica problemas resueltos
Termodinámica problemas resueltos
Termodinámica problemas resueltos
Termodinámica problemas resueltos
Termodinámica problemas resueltos
Termodinámica problemas resueltos
Termodinámica problemas resueltos
Termodinámica problemas resueltos
Termodinámica problemas resueltos
Termodinámica problemas resueltos
Termodinámica problemas resueltos
Termodinámica problemas resueltos
Termodinámica problemas resueltos
Termodinámica problemas resueltos
Termodinámica problemas resueltos
Termodinámica problemas resueltos
Termodinámica problemas resueltos
Termodinámica problemas resueltos
Termodinámica problemas resueltos
Termodinámica problemas resueltos
Termodinámica problemas resueltos
Termodinámica problemas resueltos
Termodinámica problemas resueltos
Termodinámica problemas resueltos
Termodinámica problemas resueltos
Termodinámica problemas resueltos
Termodinámica problemas resueltos
Termodinámica problemas resueltos
Termodinámica problemas resueltos
Termodinámica problemas resueltos
Termodinámica problemas resueltos
Termodinámica problemas resueltos
Termodinámica problemas resueltos
Termodinámica problemas resueltos
Termodinámica problemas resueltos
Termodinámica problemas resueltos
Termodinámica problemas resueltos
Termodinámica problemas resueltos
Termodinámica problemas resueltos
Termodinámica problemas resueltos
Termodinámica problemas resueltos
Termodinámica problemas resueltos
Termodinámica problemas resueltos
Termodinámica problemas resueltos
Termodinámica problemas resueltos
Termodinámica problemas resueltos
Termodinámica problemas resueltos
Termodinámica problemas resueltos
Termodinámica problemas resueltos
Termodinámica problemas resueltos
Termodinámica problemas resueltos
Termodinámica problemas resueltos
Termodinámica problemas resueltos

More Related Content

What's hot

Resolucion problemas equilibrio quimico
Resolucion problemas equilibrio quimicoResolucion problemas equilibrio quimico
Resolucion problemas equilibrio quimicoJosé Miranda
 
Ejercicios resultos transporte de calor
Ejercicios resultos transporte de calorEjercicios resultos transporte de calor
Ejercicios resultos transporte de caloralvaro gómez
 
Fisica 1 Laboratorio - ley de hooke pdf
Fisica 1 Laboratorio - ley de hooke pdfFisica 1 Laboratorio - ley de hooke pdf
Fisica 1 Laboratorio - ley de hooke pdfJoe Arroyo Suárez
 
Ciencia materiales. Ejercicios introducción y enlaces.
Ciencia materiales. Ejercicios introducción y enlaces.Ciencia materiales. Ejercicios introducción y enlaces.
Ciencia materiales. Ejercicios introducción y enlaces.Ignacio Roldán Nogueras
 
Practica 1. determinacion de la capacidad calorifica
Practica 1. determinacion de la capacidad calorificaPractica 1. determinacion de la capacidad calorifica
Practica 1. determinacion de la capacidad calorificavalida51
 
Capacidad calorifica de gases
Capacidad calorifica de gasesCapacidad calorifica de gases
Capacidad calorifica de gasesdaszemog
 
Problemas resueltos de Cinetica Quimica
Problemas resueltos de Cinetica QuimicaProblemas resueltos de Cinetica Quimica
Problemas resueltos de Cinetica QuimicaJosé Miranda
 
191019853 solucionario-2520-2520 wark-2520termodinamica-2520oficial-5b1-5d
191019853 solucionario-2520-2520 wark-2520termodinamica-2520oficial-5b1-5d191019853 solucionario-2520-2520 wark-2520termodinamica-2520oficial-5b1-5d
191019853 solucionario-2520-2520 wark-2520termodinamica-2520oficial-5b1-5dTirado Vilca
 
Balances de energia
Balances de energiaBalances de energia
Balances de energiagerardito8
 
Ecuaciones Diferenciales - La Transformada de Laplace
Ecuaciones Diferenciales - La Transformada de LaplaceEcuaciones Diferenciales - La Transformada de Laplace
Ecuaciones Diferenciales - La Transformada de LaplaceKike Prieto
 
EJERCICIOS RESUELTOS TERMOQUIMICA
EJERCICIOS RESUELTOS TERMOQUIMICAEJERCICIOS RESUELTOS TERMOQUIMICA
EJERCICIOS RESUELTOS TERMOQUIMICAQuo Vadis
 
Problemas resueltos mecanica_de_fluidos
Problemas resueltos mecanica_de_fluidosProblemas resueltos mecanica_de_fluidos
Problemas resueltos mecanica_de_fluidosChristian Arias Vega
 
Energia potencial electrica problemas resueltos-gonzalo revelo pabon
Energia potencial electrica  problemas resueltos-gonzalo revelo pabonEnergia potencial electrica  problemas resueltos-gonzalo revelo pabon
Energia potencial electrica problemas resueltos-gonzalo revelo pabonGONZALO REVELO PABON . GORETTI
 
Capitulo iv. fisica ii. tensión superficial y capilaridad
Capitulo iv. fisica ii. tensión superficial y capilaridadCapitulo iv. fisica ii. tensión superficial y capilaridad
Capitulo iv. fisica ii. tensión superficial y capilaridadVictor Rojas Lopez
 
Ejemplos resueltos Balances de masa y energía
Ejemplos resueltos Balances de masa y energíaEjemplos resueltos Balances de masa y energía
Ejemplos resueltos Balances de masa y energíadave
 
Ejercicios resueltos cinetica quimica
Ejercicios resueltos cinetica quimicaEjercicios resueltos cinetica quimica
Ejercicios resueltos cinetica quimicaraulasencio
 

What's hot (20)

Resolucion problemas equilibrio quimico
Resolucion problemas equilibrio quimicoResolucion problemas equilibrio quimico
Resolucion problemas equilibrio quimico
 
Ejercicios resultos transporte de calor
Ejercicios resultos transporte de calorEjercicios resultos transporte de calor
Ejercicios resultos transporte de calor
 
Fisica 1 Laboratorio - ley de hooke pdf
Fisica 1 Laboratorio - ley de hooke pdfFisica 1 Laboratorio - ley de hooke pdf
Fisica 1 Laboratorio - ley de hooke pdf
 
Ciencia materiales. Ejercicios introducción y enlaces.
Ciencia materiales. Ejercicios introducción y enlaces.Ciencia materiales. Ejercicios introducción y enlaces.
Ciencia materiales. Ejercicios introducción y enlaces.
 
Practica 1. determinacion de la capacidad calorifica
Practica 1. determinacion de la capacidad calorificaPractica 1. determinacion de la capacidad calorifica
Practica 1. determinacion de la capacidad calorifica
 
Capacidad calorifica de gases
Capacidad calorifica de gasesCapacidad calorifica de gases
Capacidad calorifica de gases
 
Problemas resueltos de Cinetica Quimica
Problemas resueltos de Cinetica QuimicaProblemas resueltos de Cinetica Quimica
Problemas resueltos de Cinetica Quimica
 
191019853 solucionario-2520-2520 wark-2520termodinamica-2520oficial-5b1-5d
191019853 solucionario-2520-2520 wark-2520termodinamica-2520oficial-5b1-5d191019853 solucionario-2520-2520 wark-2520termodinamica-2520oficial-5b1-5d
191019853 solucionario-2520-2520 wark-2520termodinamica-2520oficial-5b1-5d
 
Balances de energia
Balances de energiaBalances de energia
Balances de energia
 
INFORME DE LA CAPACIDAD CALIRÍFICA DE UN CALORÍMETRO
INFORME DE LA CAPACIDAD CALIRÍFICA DE UN CALORÍMETROINFORME DE LA CAPACIDAD CALIRÍFICA DE UN CALORÍMETRO
INFORME DE LA CAPACIDAD CALIRÍFICA DE UN CALORÍMETRO
 
Ecuaciones Diferenciales - La Transformada de Laplace
Ecuaciones Diferenciales - La Transformada de LaplaceEcuaciones Diferenciales - La Transformada de Laplace
Ecuaciones Diferenciales - La Transformada de Laplace
 
EJERCICIOS RESUELTOS TERMOQUIMICA
EJERCICIOS RESUELTOS TERMOQUIMICAEJERCICIOS RESUELTOS TERMOQUIMICA
EJERCICIOS RESUELTOS TERMOQUIMICA
 
Problemas resueltos mecanica_de_fluidos
Problemas resueltos mecanica_de_fluidosProblemas resueltos mecanica_de_fluidos
Problemas resueltos mecanica_de_fluidos
 
Problemas resueltos. castellan
Problemas resueltos. castellanProblemas resueltos. castellan
Problemas resueltos. castellan
 
Energia potencial electrica problemas resueltos-gonzalo revelo pabon
Energia potencial electrica  problemas resueltos-gonzalo revelo pabonEnergia potencial electrica  problemas resueltos-gonzalo revelo pabon
Energia potencial electrica problemas resueltos-gonzalo revelo pabon
 
Ejercicios carnot
Ejercicios carnotEjercicios carnot
Ejercicios carnot
 
Ejercicios tema 3 2 Estructura Cristalina
Ejercicios tema 3 2 Estructura CristalinaEjercicios tema 3 2 Estructura Cristalina
Ejercicios tema 3 2 Estructura Cristalina
 
Capitulo iv. fisica ii. tensión superficial y capilaridad
Capitulo iv. fisica ii. tensión superficial y capilaridadCapitulo iv. fisica ii. tensión superficial y capilaridad
Capitulo iv. fisica ii. tensión superficial y capilaridad
 
Ejemplos resueltos Balances de masa y energía
Ejemplos resueltos Balances de masa y energíaEjemplos resueltos Balances de masa y energía
Ejemplos resueltos Balances de masa y energía
 
Ejercicios resueltos cinetica quimica
Ejercicios resueltos cinetica quimicaEjercicios resueltos cinetica quimica
Ejercicios resueltos cinetica quimica
 

Viewers also liked

Problemas resueltos termodinmica
Problemas resueltos termodinmicaProblemas resueltos termodinmica
Problemas resueltos termodinmicachocolatin
 
Problemas de qumica termodinamica
Problemas de qumica termodinamicaProblemas de qumica termodinamica
Problemas de qumica termodinamicahenrycava
 
Problema 14 70 del libro “termodinámica yunus a. cengel, michael a. boles qu...
Problema 14 70 del libro “termodinámica yunus a. cengel, michael a. boles  qu...Problema 14 70 del libro “termodinámica yunus a. cengel, michael a. boles  qu...
Problema 14 70 del libro “termodinámica yunus a. cengel, michael a. boles qu...Noris Guzman
 
Thermodynamics of solutions. Solved problems (Spanish)
Thermodynamics of solutions. Solved problems (Spanish)Thermodynamics of solutions. Solved problems (Spanish)
Thermodynamics of solutions. Solved problems (Spanish)Angel Darío González-Delgado
 
"Tensión superficial en el agua y otros fluidos”
"Tensión superficial en el agua y otros fluidos”"Tensión superficial en el agua y otros fluidos”
"Tensión superficial en el agua y otros fluidos”Paola Rey
 
Laboratorio de tension superficial y capilaridad
Laboratorio de tension superficial y capilaridadLaboratorio de tension superficial y capilaridad
Laboratorio de tension superficial y capilaridadDamián Solís
 
Ejercicios Calorías - Julios y Centígrados - Kelvin
Ejercicios Calorías - Julios y Centígrados - KelvinEjercicios Calorías - Julios y Centígrados - Kelvin
Ejercicios Calorías - Julios y Centígrados - KelvinGloria BioGeo
 
126696901 000049-ejercicios-resueltos-de-fisica-calorimetria
126696901 000049-ejercicios-resueltos-de-fisica-calorimetria126696901 000049-ejercicios-resueltos-de-fisica-calorimetria
126696901 000049-ejercicios-resueltos-de-fisica-calorimetriaedwin mario córdova paz
 
Carta psicometrica
Carta psicometricaCarta psicometrica
Carta psicometricajoel_f
 
fluidos en reposo fisica
fluidos en reposo fisicafluidos en reposo fisica
fluidos en reposo fisicaCarlos Saldaña
 
Aplicación de Integrales Definidas
Aplicación de Integrales DefinidasAplicación de Integrales Definidas
Aplicación de Integrales DefinidasEmma
 
Tablas termodinámicas del agua yunus a. cengel
Tablas termodinámicas del agua  yunus a. cengel Tablas termodinámicas del agua  yunus a. cengel
Tablas termodinámicas del agua yunus a. cengel Itamar Bernal
 
Fisica ecuacion de continuidad
Fisica ecuacion de continuidadFisica ecuacion de continuidad
Fisica ecuacion de continuidadDiana Salazar
 
S1C2: Fluidos en reposo
S1C2: Fluidos en reposoS1C2: Fluidos en reposo
S1C2: Fluidos en reposoTareas 911
 

Viewers also liked (20)

Problemas resueltos termodinmica
Problemas resueltos termodinmicaProblemas resueltos termodinmica
Problemas resueltos termodinmica
 
Problemas de qumica termodinamica
Problemas de qumica termodinamicaProblemas de qumica termodinamica
Problemas de qumica termodinamica
 
Problema 14 70 del libro “termodinámica yunus a. cengel, michael a. boles qu...
Problema 14 70 del libro “termodinámica yunus a. cengel, michael a. boles  qu...Problema 14 70 del libro “termodinámica yunus a. cengel, michael a. boles  qu...
Problema 14 70 del libro “termodinámica yunus a. cengel, michael a. boles qu...
 
Ejercicio de hysys
Ejercicio de hysysEjercicio de hysys
Ejercicio de hysys
 
Thermodynamics of solutions. Solved problems (Spanish)
Thermodynamics of solutions. Solved problems (Spanish)Thermodynamics of solutions. Solved problems (Spanish)
Thermodynamics of solutions. Solved problems (Spanish)
 
"Tensión superficial en el agua y otros fluidos”
"Tensión superficial en el agua y otros fluidos”"Tensión superficial en el agua y otros fluidos”
"Tensión superficial en el agua y otros fluidos”
 
Laboratorio de tension superficial y capilaridad
Laboratorio de tension superficial y capilaridadLaboratorio de tension superficial y capilaridad
Laboratorio de tension superficial y capilaridad
 
Problemario de termodinamica
Problemario de termodinamicaProblemario de termodinamica
Problemario de termodinamica
 
Unidad 5
Unidad 5Unidad 5
Unidad 5
 
Ejercicios 4
Ejercicios 4Ejercicios 4
Ejercicios 4
 
Ejercicios Calorías - Julios y Centígrados - Kelvin
Ejercicios Calorías - Julios y Centígrados - KelvinEjercicios Calorías - Julios y Centígrados - Kelvin
Ejercicios Calorías - Julios y Centígrados - Kelvin
 
126696901 000049-ejercicios-resueltos-de-fisica-calorimetria
126696901 000049-ejercicios-resueltos-de-fisica-calorimetria126696901 000049-ejercicios-resueltos-de-fisica-calorimetria
126696901 000049-ejercicios-resueltos-de-fisica-calorimetria
 
Carta psicometrica
Carta psicometricaCarta psicometrica
Carta psicometrica
 
fluidos en reposo fisica
fluidos en reposo fisicafluidos en reposo fisica
fluidos en reposo fisica
 
5.disoluciones.
5.disoluciones.5.disoluciones.
5.disoluciones.
 
Aplicación de Integrales Definidas
Aplicación de Integrales DefinidasAplicación de Integrales Definidas
Aplicación de Integrales Definidas
 
Guía 6 ecuación general de energía
Guía 6  ecuación general de energíaGuía 6  ecuación general de energía
Guía 6 ecuación general de energía
 
Tablas termodinámicas del agua yunus a. cengel
Tablas termodinámicas del agua  yunus a. cengel Tablas termodinámicas del agua  yunus a. cengel
Tablas termodinámicas del agua yunus a. cengel
 
Fisica ecuacion de continuidad
Fisica ecuacion de continuidadFisica ecuacion de continuidad
Fisica ecuacion de continuidad
 
S1C2: Fluidos en reposo
S1C2: Fluidos en reposoS1C2: Fluidos en reposo
S1C2: Fluidos en reposo
 

Similar to Termodinámica problemas resueltos

Ensayo termodinamica 2016
Ensayo termodinamica 2016Ensayo termodinamica 2016
Ensayo termodinamica 2016Crisbel93
 
TEMAS DE FISICO QUIMICA I PARA ESTUDIANTES DE INGENIERIA.ppt
TEMAS DE FISICO QUIMICA I PARA ESTUDIANTES DE INGENIERIA.pptTEMAS DE FISICO QUIMICA I PARA ESTUDIANTES DE INGENIERIA.ppt
TEMAS DE FISICO QUIMICA I PARA ESTUDIANTES DE INGENIERIA.pptAndrsDanielBenitesRe
 
Conceptos basicos y gases ideales primera clase
Conceptos basicos y gases ideales primera claseConceptos basicos y gases ideales primera clase
Conceptos basicos y gases ideales primera clasehuberney1850
 
termodinamica-graton.pdf
termodinamica-graton.pdftermodinamica-graton.pdf
termodinamica-graton.pdfvon Pereira
 
SEMINARIOS PRIMERA PARTE-2015 (1).pdf
SEMINARIOS PRIMERA PARTE-2015 (1).pdfSEMINARIOS PRIMERA PARTE-2015 (1).pdf
SEMINARIOS PRIMERA PARTE-2015 (1).pdfMsAguilarVanni
 
Practica2 temperatura
Practica2 temperaturaPractica2 temperatura
Practica2 temperaturaOscar Monroy
 
Cap 4 gases-3662
Cap 4 gases-3662Cap 4 gases-3662
Cap 4 gases-3662opulento22
 
Balancecuestionario
BalancecuestionarioBalancecuestionario
BalancecuestionarioGloria Maria
 
Lectura 1 fisicoquimica
Lectura 1 fisicoquimicaLectura 1 fisicoquimica
Lectura 1 fisicoquimicaYesZz Vc
 
Diapositivas gas ideal+reales
Diapositivas gas ideal+realesDiapositivas gas ideal+reales
Diapositivas gas ideal+realesoriel arancibia
 
Tratatimiento numerico de ecuaciones diferenciales (2)
Tratatimiento numerico de ecuaciones diferenciales (2)Tratatimiento numerico de ecuaciones diferenciales (2)
Tratatimiento numerico de ecuaciones diferenciales (2)daferro
 
Ejercicios termometria dilatacion_pp1-28
Ejercicios termometria dilatacion_pp1-28Ejercicios termometria dilatacion_pp1-28
Ejercicios termometria dilatacion_pp1-28perlacc
 
Estequiometria matricial con modelo termodinamico de correlaciones de Lee-Kesler
Estequiometria matricial con modelo termodinamico de correlaciones de Lee-KeslerEstequiometria matricial con modelo termodinamico de correlaciones de Lee-Kesler
Estequiometria matricial con modelo termodinamico de correlaciones de Lee-KeslerDavid Soler Camargo
 
Instituto universitario politécnico termodinamica 8
Instituto universitario politécnico termodinamica 8Instituto universitario politécnico termodinamica 8
Instituto universitario politécnico termodinamica 8Solmaira Tablante
 
1. ECUACIONES NO LINEALES
1. ECUACIONES NO LINEALES1. ECUACIONES NO LINEALES
1. ECUACIONES NO LINEALESedvinogo
 

Similar to Termodinámica problemas resueltos (20)

Libro termodinamica
Libro termodinamicaLibro termodinamica
Libro termodinamica
 
Ensayo termodinamica 2016
Ensayo termodinamica 2016Ensayo termodinamica 2016
Ensayo termodinamica 2016
 
TEMAS DE FISICO QUIMICA I PARA ESTUDIANTES DE INGENIERIA.ppt
TEMAS DE FISICO QUIMICA I PARA ESTUDIANTES DE INGENIERIA.pptTEMAS DE FISICO QUIMICA I PARA ESTUDIANTES DE INGENIERIA.ppt
TEMAS DE FISICO QUIMICA I PARA ESTUDIANTES DE INGENIERIA.ppt
 
Conceptos basicos y gases ideales primera clase
Conceptos basicos y gases ideales primera claseConceptos basicos y gases ideales primera clase
Conceptos basicos y gases ideales primera clase
 
termodinamica-graton.pdf
termodinamica-graton.pdftermodinamica-graton.pdf
termodinamica-graton.pdf
 
SEMINARIOS PRIMERA PARTE-2015 (1).pdf
SEMINARIOS PRIMERA PARTE-2015 (1).pdfSEMINARIOS PRIMERA PARTE-2015 (1).pdf
SEMINARIOS PRIMERA PARTE-2015 (1).pdf
 
Cap 1 gases
Cap 1 gasesCap 1 gases
Cap 1 gases
 
Practica2 temperatura
Practica2 temperaturaPractica2 temperatura
Practica2 temperatura
 
Cap 4 gases-3662
Cap 4 gases-3662Cap 4 gases-3662
Cap 4 gases-3662
 
Balancecuestionario
BalancecuestionarioBalancecuestionario
Balancecuestionario
 
Lectura 1 fisicoquimica
Lectura 1 fisicoquimicaLectura 1 fisicoquimica
Lectura 1 fisicoquimica
 
7.a gases ideales
7.a gases ideales7.a gases ideales
7.a gases ideales
 
Diapositivas gas ideal+reales
Diapositivas gas ideal+realesDiapositivas gas ideal+reales
Diapositivas gas ideal+reales
 
Tratatimiento numerico de ecuaciones diferenciales (2)
Tratatimiento numerico de ecuaciones diferenciales (2)Tratatimiento numerico de ecuaciones diferenciales (2)
Tratatimiento numerico de ecuaciones diferenciales (2)
 
Ejercicios termometria dilatacion_pp1-28
Ejercicios termometria dilatacion_pp1-28Ejercicios termometria dilatacion_pp1-28
Ejercicios termometria dilatacion_pp1-28
 
Estequiometria matricial con modelo termodinamico de correlaciones de Lee-Kesler
Estequiometria matricial con modelo termodinamico de correlaciones de Lee-KeslerEstequiometria matricial con modelo termodinamico de correlaciones de Lee-Kesler
Estequiometria matricial con modelo termodinamico de correlaciones de Lee-Kesler
 
Instituto universitario politécnico termodinamica 8
Instituto universitario politécnico termodinamica 8Instituto universitario politécnico termodinamica 8
Instituto universitario politécnico termodinamica 8
 
1. ECUACIONES NO LINEALES
1. ECUACIONES NO LINEALES1. ECUACIONES NO LINEALES
1. ECUACIONES NO LINEALES
 
LABORATORIO FISICOQUIMICA 1
LABORATORIO FISICOQUIMICA 1LABORATORIO FISICOQUIMICA 1
LABORATORIO FISICOQUIMICA 1
 
Termodinámica de Soluciones
Termodinámica de SolucionesTermodinámica de Soluciones
Termodinámica de Soluciones
 

Recently uploaded

cuadernillo de lectoescritura para niños de básica
cuadernillo de lectoescritura para niños de básicacuadernillo de lectoescritura para niños de básica
cuadernillo de lectoescritura para niños de básicaGianninaValeskaContr
 
DETALLES EN EL DISEÑO DE INTERIOR
DETALLES EN EL DISEÑO DE INTERIORDETALLES EN EL DISEÑO DE INTERIOR
DETALLES EN EL DISEÑO DE INTERIORGonella
 
BIOLOGIA_banco de preguntas_editorial icfes examen de estado .pdf
BIOLOGIA_banco de preguntas_editorial icfes examen de estado .pdfBIOLOGIA_banco de preguntas_editorial icfes examen de estado .pdf
BIOLOGIA_banco de preguntas_editorial icfes examen de estado .pdfCESARMALAGA4
 
LA OVEJITA QUE VINO A CENAR CUENTO INFANTIL.pdf
LA OVEJITA QUE VINO A CENAR CUENTO INFANTIL.pdfLA OVEJITA QUE VINO A CENAR CUENTO INFANTIL.pdf
LA OVEJITA QUE VINO A CENAR CUENTO INFANTIL.pdfNataliaMalky1
 
SISTEMA INMUNE FISIOLOGIA MEDICA UNSL 2024
SISTEMA INMUNE FISIOLOGIA MEDICA UNSL 2024SISTEMA INMUNE FISIOLOGIA MEDICA UNSL 2024
SISTEMA INMUNE FISIOLOGIA MEDICA UNSL 2024gharce
 
FICHA DE MONITOREO Y ACOMPAÑAMIENTO 2024 MINEDU
FICHA DE MONITOREO Y ACOMPAÑAMIENTO  2024 MINEDUFICHA DE MONITOREO Y ACOMPAÑAMIENTO  2024 MINEDU
FICHA DE MONITOREO Y ACOMPAÑAMIENTO 2024 MINEDUgustavorojas179704
 
IV SES LUN 15 TUTO CUIDO MI MENTE CUIDANDO MI CUERPO YESSENIA 933623393 NUEV...
IV SES LUN 15 TUTO CUIDO MI MENTE CUIDANDO MI CUERPO  YESSENIA 933623393 NUEV...IV SES LUN 15 TUTO CUIDO MI MENTE CUIDANDO MI CUERPO  YESSENIA 933623393 NUEV...
IV SES LUN 15 TUTO CUIDO MI MENTE CUIDANDO MI CUERPO YESSENIA 933623393 NUEV...YobanaZevallosSantil1
 
Los Nueve Principios del Desempeño de la Sostenibilidad
Los Nueve Principios del Desempeño de la SostenibilidadLos Nueve Principios del Desempeño de la Sostenibilidad
Los Nueve Principios del Desempeño de la SostenibilidadJonathanCovena1
 
Tema 8.- Gestion de la imagen a traves de la comunicacion de crisis.pdf
Tema 8.- Gestion de la imagen a traves de la comunicacion de crisis.pdfTema 8.- Gestion de la imagen a traves de la comunicacion de crisis.pdf
Tema 8.- Gestion de la imagen a traves de la comunicacion de crisis.pdfDaniel Ángel Corral de la Mata, Ph.D.
 
PPT_Formación integral y educación CRESE (1).pdf
PPT_Formación integral y educación CRESE (1).pdfPPT_Formación integral y educación CRESE (1).pdf
PPT_Formación integral y educación CRESE (1).pdfEDILIAGAMBOA
 
Fundamentos y Principios de Psicopedagogía..pdf
Fundamentos y Principios de Psicopedagogía..pdfFundamentos y Principios de Psicopedagogía..pdf
Fundamentos y Principios de Psicopedagogía..pdfsamyarrocha1
 
c3.hu3.p1.p2.El ser humano y el sentido de su existencia.pptx
c3.hu3.p1.p2.El ser humano y el sentido de su existencia.pptxc3.hu3.p1.p2.El ser humano y el sentido de su existencia.pptx
c3.hu3.p1.p2.El ser humano y el sentido de su existencia.pptxMartín Ramírez
 
libro para colorear de Peppa pig, ideal para educación inicial
libro para colorear de Peppa pig, ideal para educación iniciallibro para colorear de Peppa pig, ideal para educación inicial
libro para colorear de Peppa pig, ideal para educación inicialLorenaSanchez350426
 
PINTURA ITALIANA DEL CINQUECENTO (SIGLO XVI).ppt
PINTURA ITALIANA DEL CINQUECENTO (SIGLO XVI).pptPINTURA ITALIANA DEL CINQUECENTO (SIGLO XVI).ppt
PINTURA ITALIANA DEL CINQUECENTO (SIGLO XVI).pptAlberto Rubio
 
Estrategia de Enseñanza y Aprendizaje.pdf
Estrategia de Enseñanza y Aprendizaje.pdfEstrategia de Enseñanza y Aprendizaje.pdf
Estrategia de Enseñanza y Aprendizaje.pdfromanmillans
 

Recently uploaded (20)

cuadernillo de lectoescritura para niños de básica
cuadernillo de lectoescritura para niños de básicacuadernillo de lectoescritura para niños de básica
cuadernillo de lectoescritura para niños de básica
 
DETALLES EN EL DISEÑO DE INTERIOR
DETALLES EN EL DISEÑO DE INTERIORDETALLES EN EL DISEÑO DE INTERIOR
DETALLES EN EL DISEÑO DE INTERIOR
 
BIOLOGIA_banco de preguntas_editorial icfes examen de estado .pdf
BIOLOGIA_banco de preguntas_editorial icfes examen de estado .pdfBIOLOGIA_banco de preguntas_editorial icfes examen de estado .pdf
BIOLOGIA_banco de preguntas_editorial icfes examen de estado .pdf
 
LA OVEJITA QUE VINO A CENAR CUENTO INFANTIL.pdf
LA OVEJITA QUE VINO A CENAR CUENTO INFANTIL.pdfLA OVEJITA QUE VINO A CENAR CUENTO INFANTIL.pdf
LA OVEJITA QUE VINO A CENAR CUENTO INFANTIL.pdf
 
SISTEMA INMUNE FISIOLOGIA MEDICA UNSL 2024
SISTEMA INMUNE FISIOLOGIA MEDICA UNSL 2024SISTEMA INMUNE FISIOLOGIA MEDICA UNSL 2024
SISTEMA INMUNE FISIOLOGIA MEDICA UNSL 2024
 
Aedes aegypti + Intro to Coquies EE.pptx
Aedes aegypti + Intro to Coquies EE.pptxAedes aegypti + Intro to Coquies EE.pptx
Aedes aegypti + Intro to Coquies EE.pptx
 
Sesión La luz brilla en la oscuridad.pdf
Sesión  La luz brilla en la oscuridad.pdfSesión  La luz brilla en la oscuridad.pdf
Sesión La luz brilla en la oscuridad.pdf
 
FICHA DE MONITOREO Y ACOMPAÑAMIENTO 2024 MINEDU
FICHA DE MONITOREO Y ACOMPAÑAMIENTO  2024 MINEDUFICHA DE MONITOREO Y ACOMPAÑAMIENTO  2024 MINEDU
FICHA DE MONITOREO Y ACOMPAÑAMIENTO 2024 MINEDU
 
IV SES LUN 15 TUTO CUIDO MI MENTE CUIDANDO MI CUERPO YESSENIA 933623393 NUEV...
IV SES LUN 15 TUTO CUIDO MI MENTE CUIDANDO MI CUERPO  YESSENIA 933623393 NUEV...IV SES LUN 15 TUTO CUIDO MI MENTE CUIDANDO MI CUERPO  YESSENIA 933623393 NUEV...
IV SES LUN 15 TUTO CUIDO MI MENTE CUIDANDO MI CUERPO YESSENIA 933623393 NUEV...
 
Los Nueve Principios del Desempeño de la Sostenibilidad
Los Nueve Principios del Desempeño de la SostenibilidadLos Nueve Principios del Desempeño de la Sostenibilidad
Los Nueve Principios del Desempeño de la Sostenibilidad
 
Tema 8.- Gestion de la imagen a traves de la comunicacion de crisis.pdf
Tema 8.- Gestion de la imagen a traves de la comunicacion de crisis.pdfTema 8.- Gestion de la imagen a traves de la comunicacion de crisis.pdf
Tema 8.- Gestion de la imagen a traves de la comunicacion de crisis.pdf
 
PPT_Formación integral y educación CRESE (1).pdf
PPT_Formación integral y educación CRESE (1).pdfPPT_Formación integral y educación CRESE (1).pdf
PPT_Formación integral y educación CRESE (1).pdf
 
La luz brilla en la oscuridad. Necesitamos luz
La luz brilla en la oscuridad. Necesitamos luzLa luz brilla en la oscuridad. Necesitamos luz
La luz brilla en la oscuridad. Necesitamos luz
 
Aedes aegypti + Intro to Coquies EE.pptx
Aedes aegypti + Intro to Coquies EE.pptxAedes aegypti + Intro to Coquies EE.pptx
Aedes aegypti + Intro to Coquies EE.pptx
 
Fundamentos y Principios de Psicopedagogía..pdf
Fundamentos y Principios de Psicopedagogía..pdfFundamentos y Principios de Psicopedagogía..pdf
Fundamentos y Principios de Psicopedagogía..pdf
 
TL/CNL – 2.ª FASE .
TL/CNL – 2.ª FASE                       .TL/CNL – 2.ª FASE                       .
TL/CNL – 2.ª FASE .
 
c3.hu3.p1.p2.El ser humano y el sentido de su existencia.pptx
c3.hu3.p1.p2.El ser humano y el sentido de su existencia.pptxc3.hu3.p1.p2.El ser humano y el sentido de su existencia.pptx
c3.hu3.p1.p2.El ser humano y el sentido de su existencia.pptx
 
libro para colorear de Peppa pig, ideal para educación inicial
libro para colorear de Peppa pig, ideal para educación iniciallibro para colorear de Peppa pig, ideal para educación inicial
libro para colorear de Peppa pig, ideal para educación inicial
 
PINTURA ITALIANA DEL CINQUECENTO (SIGLO XVI).ppt
PINTURA ITALIANA DEL CINQUECENTO (SIGLO XVI).pptPINTURA ITALIANA DEL CINQUECENTO (SIGLO XVI).ppt
PINTURA ITALIANA DEL CINQUECENTO (SIGLO XVI).ppt
 
Estrategia de Enseñanza y Aprendizaje.pdf
Estrategia de Enseñanza y Aprendizaje.pdfEstrategia de Enseñanza y Aprendizaje.pdf
Estrategia de Enseñanza y Aprendizaje.pdf
 

Termodinámica problemas resueltos

  • 1. Problemas y ejercicios resueltos de Termodinámica I Julián Moreno Mestre Estudiante de Ciencias Físicas Universidad Complutense de Madrid "Lo que hacemos por nosotros mismos, muere con nosotros. Lo que hacemos por los demás y por el mundo, permanece y es inmortal." Albert Payne
  • 3. Ejercicios y problemas de Termodinámica I Índice: PRÓLOGO ………………………………………………………………………. 4 BIBLIOGRAFÍA CONSULTADA ……………………………………………… 5 CAPÍTULO 1º. Principio cero de la termodinámica y temperatura. Ecuaciones de estado. Coeficientes termodinámicos importantes. Relaciones diferenciales en termodinámica. …………………………………………………………….…….. 6 CAPÍTULO 2º. Trabajo en termodinámica. Relaciones entre las derivadas parciales. Primer principio de la termodinámica. Coeficientes calorimétricos …. 22 CAPÍTULO 3º. Segundo principio de la termodinámica. Temperatura termodinámica y entropía. Principio de aumento de entropía. Ecuación fundamental de la termodinámica. Ecuaciones TdS. …………………………….. 42 CAPITULO 4º. Potenciales termodinámicos. Relaciones de Maxwell…………... 78 CAPÍTULO 5º. Transiciones de fase. Regla de las fases. ……………………….. 102 2
  • 5. Ejercicios y problemas de Termodinámica I PRÓLOGO: Tras muchos años cursando como alumno la asignatura de Termodinámica I en la facultad de ciencias físicas de la Universidad Complutense de Madrid, decidí hacer un recopilatorio de parte de los problemas que he estudiado y resuelto en la asignatura. Mi experiencia positiva en las asignaturas de Mecánica y Ondas II y Electromagnetismo I, en las que contribuí a escribir librillos de problemas con la academia INTEC con Ramón Fernández Villegas, me ha llevado a hacer lo mismo con Termodinámica I. El resultado en termodinámica fue un notable y conocer mejor la termodinámica, y fruto del trabajo en la asignatura ha quedado además esta colección de problemas resueltos que espero les sirva de ayuda. Muchos de los problemas son ejercicios de clase de la asignatura. La mayoría de los problemas provienen de libros de termodinámica. En cada uno he procurado contrastar la solución final que daban los libros o en clase. No obstante, prefiero advertir que al ser una publicación sin revisión puede ser posible encontrar no pocos errores. Si cualquiera que lea este documento encuentra un error comuníquemelo al mail para su posterior corrección y mejora de este texto. No es un texto completo en problemas para la asignatura. La ausencia de colecciones de problemas de sistemas abiertos, así como la ausencia de problemas y numerosos casos prácticos en lo referente al equilibrio termodinámico y al tercer principio, dejan incompleto este texto. No obstante estará abierta la posibilidad de un sexto capitulo o ampliación de algunos que incluya y corrija la ausencia de estos temas que se imparten en Termodinámica I. Ideal sería dotar a este texto de un Capítulo 0 que trate sobre los métodos matemáticos utilizados en Termodinámica I, como es la integración de ecuaciones diferenciales en una y varias variables. Quiero expresar mi agradecimiento a la profesora del departamento de Física Aplicada I Vicenta Maria Barragán por la gran cantidad de dudas que me resolvió con los problemas. Sin duda sus orientaciones me han servido para resolver ciertos problemas que puedo presentar en el presente texto. También mi agradecimiento al fallecido profesor José Aguilar Peris, del cual es imposible no considerarse alumno suyo cuando he aprendido mucho de su libro Curso de Termodinámica. No son pocos los problemas que he sacado de su libro. Es mi deseo dedicarle este texto a todos los compañeros de la facultad de ciencias físicas de la complutense, espero que les sirva de mucha ayuda, y espero me disculpen por algún posible error que puedan encontrar. Julián Moreno Mestre Madrid, 25 de Marzo de 2008 julianmorenomestre@hotmail.com 4
  • 6. Julián Moreno Mestre BIBLIOGRAFÍA: Para la elaboración de esta colección de problemas así de cómo los resúmenes de teoría se ha consultado varios libros. Curso de termodinámica. José Aguilar Péris. Ed. Alhambra Longman . Termo I y II. Manuel Zamora Carranza. Ed. Universidad de Sevilla. Físicoquímica I y II. Levine, Ed. Mac Graw Hil. Problemas de Física Vol. III Termología. E. Gullón de Senespleda y M. López Rodríguez. Ed. Librería internacional de Romo SL. 100 Problemas de Termodinámica. J. A. Manzanares y J. Pellicer. Ed. Alianza. Calor y Termodinámica. Zemansky y Dittman. Ed. Mac Graw Hill. 5
  • 7. Ejercicios y problemas de Termodinámica I CAPÍTULO 1º Principio cero de la termodinámica y temperatura. Ecuaciones de estado. Coeficientes termodinámicos importantes. Relaciones diferenciales en termodinámica. Resumen de teoría: Principio cero de la termodinámica: - Dos sistemas aislados A y B puestos en contacto prolongado alcanzan el equilibrio térmico. - Si A y B separadamente están en equilibro térmico con C, están también en equilibrio térmico entre si. (Propiedad transitiva) Temperatura empírica: θ ( x) = 273.16 Ecuaciones de estado importantes: Gas ideal: Sólido paramagnético (Ley de Curie): x x PT pV = nRT TM = HC Gas de Van der Waals: ⎛ n2a ⎞ ⎜ p + 2 ⎟(V − nb) = nRT ⎜ V ⎟ ⎠ ⎝ Coeficientes termodinámicos importantes: Dilatación cúbica Compresibilidad isoterma 1 ⎛ ∂V ⎞ 1 ⎛ ∂V ⎞ β= ⎜ κT = − ⎜ ⎟ ⎟ V ⎝ ∂T ⎠ p V ⎝ ∂p ⎠T Piezotérmico 1 ⎛ ∂p ⎞ α= ⎜ ⎟ p ⎝ ∂T ⎠V 6
  • 8. Julián Moreno Mestre Problemas: 1º Los sistemas A y B son sales paramagnéticas con coordenadas (H, M) y (H’, M’) respectivamente, mientras que el sistema C es un gas con coordenadas (p, V). Cuando A y C están en equilibrio térmico se cumple: nRcH − MpV = 0 , y cuando lo están B y C se cumple: EMBED Equation.3 M ' pV − nR(c' H '+ aM ' ) = 0 , siendo los símbolos n, R, a, c y c’ constantes: a) ¿Cuáles son las funciones, del par de variables de cada sistema, iguales entre si en el equilibrio térmico? b) ¿Cuál es la relación que expresa el equilibrio térmico entre los sistemas A y B? Solución: a) Partiendo de las relaciones entre los sistemas en el equilibrio, las funciones del par de variables iguales entre si en el equilibrio son: nRcH ⎫ nRcH − MpV = 0 → pV = ⎪ nRcH nR (c ' H '+ aM ') ⎪ M = ⎬ pV = M M' nR(c ' H '+ aM ') ⎪ M ' pV − nR(c ' H '+ aM ') = 0 → pV = ⎪ ⎭ M' b) La relación en el equilibrio entre los sistemas A y B la extraemos a partir de la relación obtenida en el apartado a): RcH R(c ' H '+ aM ') nRcH nR(c ' H '+ aM ') = = → M M' M M' 2º Los sistemas A y B son gases ideales con coordenadas (p,V) y (p,V), respectivamente, y el sistema C es una sustancia elástica de coordenadas (F,L). Cuando A y C están en equilibrio térmico se cumple: ⎛ L L2 ⎞ 0 kpV ⎜ − 2 ⎟ − FR = 0 L0 L ⎠ ⎝ Cuando están en equilibrio térmico A y B se cumple: pV − p '(V '− b) = 0 siendo b, R, k y L0 constantes. ¿Cuáles son las funciones del par de variables de cada sistema, iguales entre sí en el equilibrio térmico? ¿Cuál es la relación que expresa el equilibrio térmico entre los sistemas B y C? Solución: Partiendo de las relaciones de equilibrio: ⎫ ⎛ L L2 ⎞ FR 0 kpV ⎜ − 2 ⎟ − FR = 0 → pV = −1 ⎪ ⎛ L L2 ⎞ ⎪ FR ⎛ L L2 ⎞ ⎝ L0 L ⎠ 0 0 k ⎜ − 2 ⎟ ⎬ ⇒ pV = p '(V '− b) = ⎜ − ⎟ k ⎝ L0 L2 ⎠ ⎝ L0 L ⎠ ⎪ ⎪ pV − p '(V '− b) = 0 → pV = p '(V '− b) ⎭ Partiendo de la relación entre el par de variables de cada sistema en el equilibrio, podemos deducir la relación que expresa el equilibrio térmico entre B y C: FR ⎛ L L2 ⎞ 0 p '(V '− b) = ⎜ − ⎟ k ⎝ L0 L2 ⎠ 7 −1
  • 9. Ejercicios y problemas de Termodinámica I 3º Un sólido dieléctrico tiene como variables la polarización , P, y el campo eléctrico E, una sal paramagnética la imanación, I, y el campo magnético H y un gas tiene las coordenadas presión, p, y la densidad ρ. Cuando están en equilibrio mutuo, el primero y el segundo: χ d EH = β IP y cuando están en equilibrio el primero y el tercero cumplen la condición: α pP = ρχ d E donde α y β son constantes y χ d es la susceptibilidad dieléctrica del material no conductor. ¿Cuál es la ecuación que expresa el equilibrio térmico entre el segundo y el tercer sistema? Tomando como temperatura empírica la del gas ideal: T = MpV / mR , donde M es la masa molecular del gas, determinar las ecuaciones de estado de los tres sistemas. (Carranza) Solución: Despejamos χ d E en las ecuaciones del equilibrio antes citadas: χ d EH = β IP → χ d E= β IP ⎫ β IP α pP H ⎪ ⎪ = ⎬ → χ d E= α pP ⎪ H ρ α pP = ρχ d E → χ d E= ρ ⎪ ⎭ Por tanto el equilibrio entre el segundo y el tercero es: βI α p = H ρ En el problema la temperatura empírica del gas es: αp T= ρ Partiendo que la temperatura empírica del gas ideal es: MpV Mp V Mp M p = = = T= mR R m Rρ R ρ Esto significa que la constante α es: M α= R Por tanto la temperatura empírica del gas es: M p T= R ρ Por la relación de equilibrio de la sal y el gas sabemos que la temperatura de la sal es: I T =β H Y por ello también sabemos que la temperatura empírica del sólido dieléctrico será: χ E β IP χ d E= = PT → χ d E = PT → T = d P H 8
  • 10. Julián Moreno Mestre 4º Tres sistemas, que llamaremos 1, 2 y 3 y que tienen el volumen y la presión como variables mecánicas, se ponen en contacto térmico por parejas. Cuando el primero y el tercero están en equilibrio térmico se cumplen: p1V1 − p2V2 = bp1 y cuando lo están el segundo y el tercero: V3 ( p2V2 − p3V3 ) = a donde a y b son constantes. ¿Cumplen estos tres gases el principio cero? Si lo hacen, ¿Cuáles son las funciones que se igualan en el equilibrio térmico? (Carranza) Solución: Si lo cumplen, ya que existen dos ecuaciones que relacionan las variables de estado de dos sistemas con uno de ellos, y esto implicará que todos los tres sistemas van a estar ligados en el equilibrio por tres ecuaciones. Por tanto las funciones que se igualan en el equilibro térmico son: p2V2 = p1V1 − bp1 ⎫ p1V1 − p2V2 = bp1 ⎫ a ⎪ a ⎬→ ⎬ → p2V2 = p1V1 − bp1 = + p3V3 p2V2 = + p3V3 ⎪ V3 ( p2V2 − p3V3 ) = a ⎭ V3 V3 ⎭ 5º Tres gases 1, 2 y 3, cumplen las siguientes ecuaciones cuando se encuentran en equilibrio térmico mutuo entre parejas: α p2 + βV2 k p2 + hV2 ln V1 = + ln α ln p3 = + ln β hV3 α p1 donde α , β , k y h son constantes. Determinar las relaciones necesarias entre las constantes para que se cumpla el principio cero y las funciones que definen la temperatura empírica en cada sistema. Solución: Realizamos un par de operaciones algebraicas: k p + hV2 α p2 + βV2 ln p3 = 2 + ln β + ln α ln V1 = hV3 α p1 ln V1 α = α p1 ln α p2 + β V2 α p1 V1 α ln = α p2 + β V2 p3 β = hV3 ln k p2 + hV2 hV3 p3 β = k p2 + hV2 Si las constantes k y h son proporcionales a α y β respectivamente, se cumplirá el principio cero: k = mα h = mβ Donde m es una constante de proporcionalidad. Finalmente por igualación y simplificación obtenemos: p V α p2 + βV2 = α p1 ln 1 = αV3 ln 3 α β Se define la temperatura empírica como θ = θ ( X , Y ) , en nuestro caso θ = θ ( p, V ) . Por tanto las funciones que describen la temperatura empírica en los sistemas descritos son: p V θ1 ( p1 , V1 ) = α p1 ln 1 θ 2 ( p2 ,V2 ) = α p2 + β V2 θ3 ( p3 ,V3 ) = αV3 ln 3 α 9 β
  • 11. Ejercicios y problemas de Termodinámica I 6º Los valores límites a p → 0 del producto pV en un termómetro de gas a volumen constante, en contacto térmico con dos sistemas, θ 1 y θ 2, son 45.423 y 53.394 Pa·m3, respectivamente. Calcúlese la relación de temperaturas T (θ 1)/T(θ 2), de los dos sistemas. Si el sistema θ 1 es una célula de punto triple del agua, ¿cuál es la temperatura T(θ 2)? Solución: Partiendo de la definición de temperatura en nuestro caso: θ = θ ( X , Y ) → θ = θ ( p,V ) = pV Por tanto la relación de temperaturas: T (θ1 ) p1V p 45.423 = = 1 = = 0.851 T (θ 2 ) p2V p2 53.394 7º Supóngase que se define una escala absoluta de temperaturas, T’, tal que la diferencia entre el punto triple del agua y el cero absoluto son 300 grados. ¿Cuál sería la temperatura del punto de ebullición del agua? (La temperatura del punto de ebullición del agua en la escala absoluta habitual es 373.15K). Solución: Considerando la antigua temperatura en Kelvin, la aplicamos para conocer las temperaturas en la nueva escala absoluta. Utilizamos la siguiente expresión: x T ( x) = 273.16 x3 Colocamos la temperatura del punto triple x3 de la nueva escala absoluta de temperaturas y consideraremos x como la temperatura de ebullición del agua en la nueva escala, y T(x) , resultando la ecuación: x 373.15 → x = 300 = 409.81 373.15 = 273.16 300 273.16 8º Un termómetro de gas, a volumen constante, se utiliza para determinar la temperatura absoluta de un sistema. Las lecturas corregidas de la presión en el bulbo cuando, lleno con diversas cantidades de gas, se pone en contacto con agua en el punto triple (p0) y con el sistema (p) son: p0 (mmHg) 1000.00 800.00 500.00 200.00 100.00 p (mmHg) 1296.02 1036.86 648.11 259.30 129.66 Calcúlese la temperatura del sistema en la escala de los gases ideales. Solución: Partiendo de la temperatura del agua en el punto triple y de la expresión: p p T = T0 = 273.16 p0 p0 Determinamos cual es la temperatura calculada del sistema para cada muestra de gas: p0 (mmHg) 1000.00 800.00 500.00 200.00 100.00 p (mmHg) 1296.02 1036.86 648.11 259.30 129.66 T (K) 354.02 354.04 354.08 354.15 354.18 Haciendo una media aritmética de las temperaturas nos da como resultado aproximado de la temperatura del sistema 354.09 K. 10
  • 12. Julián Moreno Mestre 9º Un termistor obedece la ley: R (T ) = R 0 e B / T donde R 0 y B son constantes. Cuando se calibra dicho termistor en el punto triple del agua se encuentra una resistencia R 3 = 938.7 Ω y en el punto de vapor Rv = 1055.2 Ω. Hallar la temperatura que mide ese termistor cuando su lectura es R = 1004,5 Ω. Solución: Planteemos el sistema de ecuaciones exponenciales que se nos plantea para calcular la temperatura que nos piden que mediría el termistor: 938.7 = R 0 e B / 273.16 ⎫ B = −119.26 K ⎬ 1055.2 = R 0 e B / 373.16 ⎭ R0 = 1452.6 Ω Y ahora deducimos la temperatura del termistor sustituyendo las constantes: 1004.5 = 1452.6e −119.26 / T → T = 324.24 K 10º Un termopar se calibra manteniendo una de sus dos soldaduras en el punto de hielo, th = 0.00 ºC, y se obtiene: ε (t ) = α t + β t 2 donde α = 0.04 mV/ºC y β = 4.00·10-6 mV/ºC2. Determinar la fuerza electromotriz que produce dicho termopar cuando una soldadura se encuentra en el punto de vapor, tv = 100.00 ºC y la otra en el punto de fusión del cinc, tZn = 419.53 ºC. (Carranza) Solución: La fuerza electromotriz producida por el termopar entre las dos soldaduras será debida a la diferencia de potencial generada en las dos soldaduras a distintas temperaturas: ε T = ε (tZn ) − ε (tv ) = 17.48 − 4.04 = 13.44 mV 11º La resistencia de un termómetro de platino es de 18.56 Ω en el punto de fusión del estaño y de 9.83 Ω en el punto triple del agua. En estos mismos puntos las presiones de un termómetro de H2 a volumen constante son 17.2 atm y 6.80 atm. Para otra cantidad de gas en el bulbo se obtiene 1.85 atm y 1.00 atm. Además, las fuerzas electromotrices de un termómetro cobre-niquel en los mismos puntos son 9.02 mV y 2.98 mV. Determínese la temperatura de fusión del estaño en las distintas escalas termométricas. Solución: Utilizaremos en los cálculos la expresión: X TSn = 273.16 Sn X3 Calcularemos la temperatura ahora en cada termómetro: 18.56 Termómetro de platino: TSn = 273.16 = 515.76 K 9.83 12.7 = 510.17 K Termómetro de gas H2: TSn = 273.16 6.8 1.85 = 505.35 K Termómetro de otro gas: TSn = 273.16 1 9.02 Termómetro de Cu-Ni: TSn = 273.16 = 826.81 K 2.98 11
  • 13. Ejercicios y problemas de Termodinámica I 12º La ecuación de estado de un gas perfecto es: pV = nRT Calcular el coeficiciente de compresibilidad isoterma y de dilatación cúbica. Solución: Partiendo de las expresiones diferenciales de dichos coeficientes, derivando determinaremos los mismos: 1 1 1 ⎛ ∂V ⎞ nR 1 1 ⎛ ∂V ⎞ nRT 1 = → β= β= ⎜ κT = − ⎜ = → κT = ⎟ = ⎟ = 2 p T V ⎝ ∂T ⎠ p Vp T V ⎝ ∂p ⎠T Vp p 13º Un mol de gas real, a presiones moderadas, cumple con la ecuación: p (V − b ) = RT Donde R y b son constantes. Calcular el coeficiciente de compresibilidad isoterma y de dilatación cúbica. Solución: Partiendo de las expresiones diferenciales de dichos coeficientes, derivando determinaremos los mismos: R RT 1 ⎛ ∂V ⎞ R 1 ⎛ ∂V ⎞ RT β= ⎜ κT = − ⎜ →β= ⎟ = 2 → κ T = Vp 2 ⎟ = Vp V ⎝ ∂T ⎠ p Vp V ⎝ ∂p ⎠T Vp 14º Hallar el volumen final de 200 g de agua que sufre una compresión isoterma a 0 ºC desde 1 bar hasta 300 bar. κT = 0.50885 Gpas–1 Datos: ρ(0 ºC, 1 bar) = 999.84 kg/m3 Solución: Sabiendo que: 1 ⎛ ∂V ⎞ dV κT = − ⎜ = −κ T dp → ln V = −κ T p + f (T ) ⎟ → V ⎝ ∂p ⎠T V Como es a temperatura constante, entonces f (T ) ≡ cte : ln V = −κ T p + k Partiendo de los datos del problema, determinamos el volumen inicial ocupado por los 200 g de agua: m 200 = 0.200032 L Vi = = ρ 999.84 Y conocido el volumen inicial, convirtiendo las presiones y el coeficiente de compresibilidad a pascales, obtendremos el volumen final: ln Vi = −κ T pi + k ⎫ ⎬ ln V f − ln Vi = −κ T ( p f − pi ) → V f = exp ln Vi − κ T ( p f − pi ) ln V f = −κ T p f + k ⎭ ( ) V f = 0.197012 L 12
  • 14. Julián Moreno Mestre 15º Una vasija contiene 8.450 g de agua a 0ºC y el resto de la misma se llena con parafina. Cuando el agua se congela a 0ºC, se expulsan 0.620g de parafina. La densidad de la parafina a 20ºC es 0.800 g/cm3 y su coeficiente de dilatación 9.0·10-4K-1. Calcúlese la densidad del hielo. Considérese la densidad del agua igual a 1g/cm3. Solución: Integramos la ecuación diferencial del coeficiente de dilatación lineal de la parafina: 1 ⎛ ∂V ⎞ ∂V = β ∂T → ln V = β T + f ( p ) → V (T ) = f ( p)·e β T ⎜ ⎟ =β → V ⎝ ∂T ⎠ p V A partir de la expresión de la densidad: m m − βT e ρ= = V f ( p) pensemos que al ser un proceso isobárico entonces f (p) ≡ cte , y que la masa de la parafina es también constante, por tanto la expresión de la densidad de la parafina es: ρ (T ) = ke− β T Conociendo a 20 ºC = 293 K la densidad de la parafina, averiguamos el valor de la constante k: −4 · 293 ρ (293) = ke−9·10 = 0.8 → k = 1.041 kg/L La densidad de la parafina a 273 K es: ρ (273) = 1.041e−9·10 −4 · 273 = 0.814 g/cm3 El volumen de parafina desalojado es: m 0.620 V= = = 0.763 cm3 ρ 0.814 La vasija a 0 ºC tenía inicialmente agua líquida y parafina, pero a 0 ºC el agua se congela y cambia su densidad cambiando por tanto su volumen, el volumen de parafina expulsado de la vasija es equivalente a la ganancia de volumen del agua al convertirse en hielo. Por tanto el volumen del hielo será: VH2O(s) = VH2O(l) + V = 8.450 + 0.763 = 9.212 cm3 Teniendo en cuenta que toda la masa de agua líquida se convierte en hielo, la densidad del hielo es por tanto: mH2O 8.450 ρ H2O(s) = = = 0.917 g/cm3 VH2O(s) 9.212 13
  • 15. Ejercicios y problemas de Termodinámica I 16º Un hilo metálico de 0.0085 cm2 de sección, sometido a una fuerza de 20 N y a la temperatura de 20 ºC, está situado entre dos soportes rígidos separados 1.2 m. a) ¿Cuál es la fuerza recuperadora final si la temperatura se reduce a 8ºC? b) Si además de la anterior disminución de temperatura los soportes se acercan 0.012 cm, ¿cuál será la fuerza recuperadora final? Supóngase que en todo momento el hilo se mantiene rectilíneo y que el coeficiente de dilatación lineal y el módulo de Young isotermo tienen valores constantes e iguales a 1.5·10-5 K-1 y 2·1011 N/m2, respectivamente. Solución: A partir de las correspondientes relaciones diferenciales: 1 ⎛ ∂L ⎞ L ⎛ ∂F ⎞ βL = ⎜ ⎟ ϕ= ⎜ ⎟ L ⎝ ∂T ⎠ F A ⎝ ∂L ⎠T Procedemos a integrar las ecuaciones diferenciales: 1 ⎛ ∂L ⎞ ∂L L ⎛ ∂F ⎞ ∂L ∂F β L = ⎜ ⎟ → β L ∂T = ϕ= ⎜ ⎟ → = L ⎝ ∂T ⎠ F L A ⎝ ∂L ⎠T L Aϕ F F β LT = ln L + g ( F ) ln L = + f (T ) → f (T ) = ln L − Aϕ Aϕ Tras este proceso podemos distinguir la ecuación de estado más un término constante: F β LT = ln L − + cte Aϕ Con un valor inicial, determinamos el valor del término constante. 20 1.5·10−5 ·293 = ln1.2 − + cte → cte = −30227.1 0.85·10−6 ·2·1011 Por tanto: F β LT = ln L − − 0.177808 Aϕ Ahora respondemos a los apartados del ejercicio: a) Tenemos que calcular la fuerza cuando la temperatura es 8 ºC. F 1.5·10−5 ·281 = ln1.2 − − 0.177808 → F = 50.6 N 0.85·10−6 ·2·1011 b) No solo la temperatura es de 8º C, sino que además cambia la longitud: F 1.5·10−5 ·281 = ln(1.19988) − − 0.177808 → F = 33.8 N 0.85·10−6 ·2·1011 14
  • 16. Julián Moreno Mestre 17º Los coeficientes térmicos de dilatación isobárica β y compresibilidad isoterma κ T de cierto gas real vienen dados por las expresiones: Rv 2 (v − b) v 2 (v − b) 2 β= κT = RTv 3 − 2a (v − b) 2 RTv 3 − 2a (v − b) 2 siendo a y b dos constantes características del gas y v el volumen molar. a) Hállese la ecuación de estado de dicho gas, teniendo en cuenta que para volúmenes molares altos el gas se comporta idealmente. b) ¿Qué volumen ocupará un mol de gas ideal a 300 K y 14 atm suponiendo que a = 3.61 atm·L2/mol2 y b = 0.0428 L/mol. Solución: a) Integramos el coeficiente de compresibilidad isoterma: v 2 (v − b ) 2 1 ⎛ ∂v ⎞ RTv 3 − 2a(v − b) 2 ⎛ ∂p ⎞ =− ⎜ ⎟ →⎜ ⎟ =− κT = RTv 3 − 2a(v − b) 2 v ⎝ ∂p ⎠T v 3 (v − b) 2 ⎝ ∂v ⎠T ⎛ RT 2a ⎞ RT a dp = − ⎜ − 3 ⎟ dv → p = + 2 + f (T ) 2 v ⎠ v−b v ⎝ (v − b) Pero el coeficiente de dilatación isobárica presenta dificultades: Rv 2 (v − b) 1 ⎛ ∂v ⎞ RTv 3 − 2a (v − b) 2 ⎛ ∂T ⎞ = ⎜ →⎜ = β= ⎟ ⎟ RTv 3 − 2a (v − b) 2 v ⎝ ∂T ⎠ p Rv 3 (v − b) ⎝ ∂v ⎠ p Hay problemas ya que no es una ecuación de variables separadas, buscaremos con la relaciones diferenciales un recurso que nos permita su integración: ⎛ ∂T ⎞ ⎛ ∂p ⎞ 1 RTv 3 − 2a (v − b) 2 v − b ⎛ ∂T ⎞ ⎛ ∂p ⎞ v − b = −⎜ = · = −⎜ ⎟ ⎟ ⎜ ⎟ ⎜ ⎟ 3 v (v − b) v−b ⎝ ∂v ⎠ p ⎝ ∂v ⎠T R ⎝ ∂p ⎠v ⎝ ∂v ⎠T R ⎛ ∂T ⎞ ⎛ ∂p ⎞ R RT ⎛ ∂p ⎞ v − b ⎛ ∂T ⎞ v − b −⎜ →⎜ → dp = dT = + g (v ) ⎟ ⎜ ⎟ = −⎜ ⎟ ⎟ = R v−b v−b ⎝ ∂v ⎠T R ⎝ ∂p ⎠v ⎝ ∂v ⎠T ⎝ ∂p ⎠v Por tanto la función de estado es: RT a p= + 2 +k v −b v Según las condiciones del problema, si el volumen molar es muy grande, entonces la ecuación de estado debe desembocar en la de un gas ideal, eso significa que: RT a + 2 + k ⎧v −b ≈ v p= RT RT k =0 →⎨ ⇒ p= + k ⎯⎯⎯⎯ p = → v −b v 2 gas ideal v v ⎩a / v ≈ 0 v ≡ grande Por tanto la ecuación de estado buscada es esta: RT a p= + 2 v −b v b) Si n = 1 entonces v = V , por tanto la ecuación de estado: RT a p= + 2 V −b V Sustituyendo llegamos a la ecuación (que desemboca en ecuación de tercer grado): 0.082·300 3.61 14 = + → V = 1.65 L. V − 0.0428 V 2 Para resolver la ecuación anterior necesitamos ayuda informática o métodos numéricos. 15
  • 17. Ejercicios y problemas de Termodinámica I 18º Los coeficientes de dilatación cúbica y de compresibilidad isoterma de cierta sustancia vienen dados por: 3aT 3 b β= κT = V V siendo a y b constantes. Determínese la ecuación de estado que relaciona p, V y T. Solución: Procedemos a integrar las ecuaciones diferenciales de los coeficientes de dilatación cúbica y de compresibilidad isoterma: 3aT 3 1 ⎛ ∂V ⎞ b 1 ⎛ ∂V ⎞ = ⎜ β= κT = = − ⎜ ⎟ ⎟ V V ⎝ ∂T ⎠ p V V ⎝ ∂p ⎠T ⎛ ∂V ⎞ 3aT 3 = ⎜ ⎟ ⎝ ∂T ⎠ p ⎛ ∂V ⎞ b = −⎜ ⎟ ⎝ ∂p ⎠T b∂p = −∂V 3aT 3∂T = ∂V 3aT 4 = V + f ( p) bp = −V + g (T ) 4 3aT 4 V = g (T ) − bp V= + f ( p) 4 Tras este proceso de integración, es ahora fácil determinar la ecuación de estado resultante más un término constante fruto del proceso de integración: 3aT 4 V= − bp + cte 4 19º El coeficiente de dilatación de un gas vale: 4T 3 T 4 −θ 4 donde θ es una constante, y el coeficiente de compresibilidad κ T = p −1 . Hallar la ecuación de estado del gas. β= Solución: Solución: Procedemos a integrar las ecuaciones diferenciales de los coeficientes de dilatación cúbica y de compresibilidad isoterma: 4T 3 1 ⎛ ∂V ⎞ 1 1 ⎛ ∂V ⎞ β= 4 4= ⎜ κT = = − ⎜ ⎟ ⎟ T −θ V ⎝ ∂T ⎠ p p V ⎝ ∂p ⎠T ∂p ∂V 4T 3 ∂V =− ∂T = 4 4 p V T −θ V 4 4 ln p = − ln V + g (T ) ln(T − θ ) = ln V + f ( p) ln V = − ln p + g (T ) ln V = ln(T 4 − θ 4 ) + f ( p) Tras este proceso de integración, es ahora fácil determinar la ecuación de estado resultante más un término constante fruto del proceso de integración: ln V = ln(T 4 − θ 4 ) − ln p + cte Para simplificarla, reagrupamos los logaritmos en un lado de la igualdad y nos valdremos de sus propiedades: Vp ⎛ Vp ⎞ = cte = cte → 4 ln V − ln(T 4 − θ 4 ) + ln p = cte → ln ⎜ 4 4 ⎟ T −θ 4 ⎝ T −θ ⎠ 16
  • 18. Julián Moreno Mestre 20º La ecuación de estado de una sustancia elástica ideal es: ⎛ L L2 ⎞ 0 F = KT ⎜ − 2 ⎟ ⎝ L0 L ⎠ donde F es la fuerza recuperadora, K es una constante y L0 (valor de la longitud a fuerza recuperadora nula) es función solo de la temperatura. Determínese: a) El coeficiente de dilatación lineal. b) El módulo de Young de la sustancia, así como el valor de este último a fuerza recuperadora nula. Solución: a) Transformamos la ecuación de estado para facilitarnos la labor del cálculo del coeficiente de dilatación lineal. ⎛ L L2 ⎞ ⎛ L3 − L3 ⎞ 0 0 F = KT ⎜ − 2 ⎟ = KT ⎜ → FL0 L2 = KT ( L3 − L3 ) 0 2 ⎟ ⎝ L0 L ⎠ ⎝ L0 L ⎠ Procedemos a derivar considerando F como constante: ∂ ∂L ∂L ( FL0 L2 ) = ∂∂ KT ( L3 − L30 ) → 2 FL0 L ⎛ ∂T ⎞ = K ( L3 − L30 ) + 3L2 KT ⎛ ∂T ⎞ ⎜ ⎟ ⎜ ⎟ T ∂T ⎝ ⎠F ⎝ ⎠F Y sabiendo que el coeficiente de dilatación lineal es: 1 ⎛ ∂L ⎞ βL = ⎜ ⎟ L ⎝ ∂T ⎠ F lo sacaremos de la ecuación: 1 ⎛ ∂L ⎞ 1 ⎛ ∂L ⎞ 3 3 3 2 FL0 L2 ⎜ ⎟ = K ( L − L0 ) + 3L KT ⎜ ⎟ L ⎝ ∂T ⎠ F L ⎝ ∂T ⎠ F ( ) 2 FL0 L2 β L = K ( L3 − L3 ) + 3L3 KT β L → β L L2 ( 2 FL0 − 3LKT ) = K ( L3 − L3 ) 0 0 βL = K ( L3 − L3 ) 0 2 FL0 L2 − 3L3 KT Mediante la ecuación de estado, eliminamos F: K ( L3 − L3 ) ( L3 − L30 ) 0 → βL = βL = 2TL3 − L3T ⎛ L3 − L3 ⎞ 0 2 3 0 L0 L − 3L KT 2 KT ⎜ 2 ⎟ ⎝ L0 L ⎠ b) Derivamos la ecuación inicial de nuestro problema: ⎛ 2 L2 1 ⎞ L ⎛ ∂F ⎞ L ϕ = ⎜ ⎟ = KT ⎜ 30 + ⎟ A ⎝ ∂L ⎠T A L0 ⎠ ⎝ L Y ahora para fuerza recuperadora nula L = L0: ⎛ 2 L2 1 ⎞ 3KT L0 ϕ = KT ⎜ 30 + ⎟ → ϕ = A A ⎝ L0 L0 ⎠ 17
  • 19. Ejercicios y problemas de Termodinámica I 21º El coeficiente de dilatación térmica del mercurio a 273 K es 1.8·10–4·K–1 . El coeficiente de compresibilidad isotermo es 5.3·10–6 bar–1. Si una ampolla completamente llena de mercurio, sellada y sin espacio vapor, se calienta desde 273 a 274 K, ¿cuál sería el aumento de presión desarrollado en el interior de la ampolla? Solución: Partiendo de las relaciones diferenciales de los coeficientes: 1 ⎛ ∂V ⎞ 1 ⎛ ∂V ⎞ β= ⎜ κT = − ⎜ ⎟ ⎟ V ⎝ ∂T ⎠ p V ⎝ ∂p ⎠T Procedemos integrando a buscar la ecuación de estado del mercurio: 1 ⎛ ∂V ⎞ ∂V 1 ⎛ ∂V ⎞ ∂V β= ⎜ κT = − ⎜ ⎟ → β∂T = ⎟ → −κ T ∂p = V ⎝ ∂T ⎠ p V V ⎝ ∂p ⎠T V ln V = β T + f ( p) ln V = −κ T p + g (T ) Podemos deducir ahora la ecuación de estado más un término constante: ln V = β T − κ T p + k Para calcular el incremento de presión despejamos la presión en la ecuación de estado: k + β T − ln V p= κT El incremento de presión es: k + β T f − ln V k + β Ti − ln V β (T f − Ti ) ∆p = p f − pi = − = = 33.96 bar κT κT κT 22º Demuestre que la ecuación: ⎛ ∂p ⎞ ⎛ ∂T ⎞ ⎛ ∂V ⎞ ⎟ ⎜ ⎟ = −1 ⎜ ⎟ ⎜ ⎟ ⎜ ⎝ ∂V ⎠T ⎝ ∂p ⎠V ⎝ ∂T ⎠ p se cumple en un gas ideal y en un gas de Clausius de ecuación p(V − b) = RT . Solución: Sea la ecuación de estado del gas ideal: pV = nRT Sean sus derivadas parciales de la ecuación de arriba: ⎛ ∂T ⎞ V nR nRT ⎛ ∂p ⎞ ⎛ ∂V ⎞ ⎜ ⎟ = ⎜ ⎟ = ⎜ ⎟ =− 2 V p ⎝ ∂V ⎠T ⎝ ∂T ⎠ p ⎝ ∂p ⎠V nR Por tanto, sustituyendo, simplificando y teniendo en cuenta la ecuación del gas ideal: nRT ⎛ nRT ⎞ ⎛ V ⎞ ⎛ nR ⎞ = −1 ⎟=− ⎜ − 2 ⎟⎜ ⎟⎜ pV ⎝ V ⎠ ⎝ nR ⎠ ⎝ p ⎠ Demostrado En el caso de la ecuación de un gas de Clausius: ⎛ ∂T ⎞ RT V −b R ⎛ ∂p ⎞ ⎛ ∂V ⎞ ⎜ ⎟ = ⎜ ⎟ = ⎜ ⎟ =− 2 (V − b) R ⎝ ∂V ⎠T ⎝ ∂T ⎠ p p ⎝ ∂p ⎠V Ahora sustituyendo, simplificando y considerando la ecuación del gas de Clausius: ⎛ RT ⎞ ⎛ V − b ⎞ ⎛ R ⎞ RT = −1 ⎜− ⎟⎜ ⎟ = − 2 ⎟⎜ p (V − b) ⎝ (V − b) ⎠ ⎝ R ⎠ ⎝ p ⎠ Demostrado. 18
  • 20. Julián Moreno Mestre 23º Un metal, cuyos coeficientes de dilatación cúbica y de compresibilidad isotérmica son 5·10–5 K–1 y 1.2·10–11 Pa–1 respectivamente, está a una presión de 105 Pa y a una temperatura de 20 ºC. Si se le recubre con una capa gruesa y muy ajustada de una sustancia de dilatación y compresibilidad despreciables: a) ¿Cuál será su presión final al elevar su temperatura hasta 32 ºC? b) ¿Cuál es la máxima temperatura que puede alcanzar el sistema si la presión más alta que puede resistir la envoltura es 1.2·1018 Pa? Solución: Partiendo de las relaciones diferenciales de los coeficientes: 1 ⎛ ∂V ⎞ 1 ⎛ ∂V ⎞ β= ⎜ κT = − ⎜ ⎟ ⎟ V ⎝ ∂T ⎠ p V ⎝ ∂p ⎠T Procedemos integrando a buscar la ecuación de estado del mercurio: 1 ⎛ ∂V ⎞ ∂V 1 ⎛ ∂V ⎞ ∂V β= ⎜ κT = − ⎜ ⎟ → β∂T = ⎟ → −κ T ∂p = V ⎝ ∂T ⎠ p V V ⎝ ∂p ⎠T V ln V = β T + f ( p) ln V = −κ T p + g (T ) Podemos deducir ahora la ecuación de estado más un término constante: ln V = β T − κ T p + k Al recubrirse de una capa gruesa que le impide dilatarse, esto supone que V es constante y por tanto rescribimos la ecuación de estado como: k = β T − κT p Determinamos ahora el valor de la constante considerando los valores iniciales del problema: k = 5·10−5 ·293 − 1.2·10−11 ·105 = 1.465·10−2 a) Su presión final al subir la temperatura hasta 32 ºC = 305 K será: 1.465·10−2 = 5·10−5 ·305 − 1.2·10−11 p → p = 5.01·107 Pas b) La máxima temperatura que puede alcanzar el sistema con la presión más alta soportable será: 1.465·10−2 = 5·10−5 ·T − 1.2·10−111.2·108 → T = 321.8 K 24º La constante dieléctrica relativa del agua varía con la temperatura de la forma: ε r = a − bT donde a y b son constantes. Sabiendo que: ⎛ ∂P ⎞ χ e = ⎜ ⎟ = ε 0 (ε r − 1) ⎝ ∂E ⎠T Establecer una ecuación de estado para el agua líquida sometida al campo eléctrico. Solución: A partir de la ecuación diferencial que cumple la susceptibilidad dieléctrica: ⎛ ∂P ⎞ χ e = ⎜ ⎟ = ε 0 (ε r − 1) ⎝ ∂E ⎠T Buscaremos integrando la ecuación de estado que nos piden para el agua líquida: ⎛ ∂P ⎞ ⎜ ⎟ = ε 0 (a − bT − 1) → ∂ P = ε 0 (a − bT − 1)∂E → P = ε 0 (a − bT − 1)E + cte ⎝ ∂E ⎠T Tengamos presente que si el campo es cero, entonces también es nula la polarización, luego el término cte es nulo. P = ε 0 (a − bT − 1)E + cte 19
  • 21. Ejercicios y problemas de Termodinámica I 25º Un paramagnético ideal cumple la ley de Curie con la susceptibilidad dependiente solo de la temperatura: 1 ⎛ ∂M ⎞ C χm = ⎜ ⎟ = V ⎝ ∂H ⎠T T donde C es una constante. Suponiendo que se conoce su coeficiente de dilatación, α, que es una función exclusiva de la temperatura, determinar la influencia del campo magnético sobre el coeficiente magnetotérmico que se define como: ⎛ ∂M ⎞ ⎜ ⎟ ⎝ ∂T ⎠ H Solución: Nos valdremos para obtener la relación de las segundas derivadas parciales cruzadas, y nos valdremos además que el momento magnético del material M es una variable de estado: ⎛ ∂ ⎛ ∂M ⎞ ⎞ ⎛ ∂ ⎛ ∂M ⎞ ⎞ ∂χ ∂V ⎛ ∂ ( χ mV ) ⎞ = ⎛ m ⎞ V + χ m ⎛ ⎞ ⎜ ⎜ ⎟ ⎟ =⎜ ⎜ ⎟ ⎟ =⎜ ⎟ ⎜ ⎟ ⎜ ⎟ ⎠ H ⎝ ∂T ⎠ H ⎝ ∂T ⎠ H ⎝ ∂H ⎝ ∂T ⎠ H ⎠T ⎝ ∂T ⎝ ∂H ⎠T ⎠ H ⎝ ∂T ⎛ ∂ ⎛ ∂M ⎞ ⎞ ⎛ ∂χ m ⎞ χm 1⎞ ⎛ V + V χmβ = V χm ⎜ β − ⎟ ⎜ ⎜ ⎟ ⎟ =⎜ ⎟ V + V χmβ = − T T⎠ ⎝ ⎝ ∂H ⎝ ∂T ⎠ H ⎠T ⎝ ∂T ⎠ H Integramos una vez con respecto a H: H ⎛ ∂ ⎛ ∂M ⎞ ⎞ ⎛ 1 ⎞⎞ ⎛ dH = ∫ ⎜ V χ m ⎜ β − ⎟ ⎟ dH ∫ ⎜ ∂H ⎜ ∂T ⎟H ⎟ T ⎠⎠ ⎝ ⎠ ⎠T ⎝ ⎝ 0⎝ Ningún elemento del integrando del segundo miembro depende del campo magnético, luego: 1⎞ ⎛ ∂M ⎞ ⎛ ∂M ⎞ ⎛ ⎜ ⎟ =⎜ ⎟ + V χm ⎜ β − ⎟ T⎠ ⎝ ∂T ⎠ H ⎝ ∂T ⎠ H =0 ⎝ 20
  • 23. Ejercicios y problemas de Termodinámica I CAPÍTULO 2º Trabajo en termodinámica. Relaciones entre las derivadas parciales. Primer principio de la termodinámica. Coeficientes calorimétricos. Resumen de teoría: Trabajo: Alambre metálico: δW = − fdl Cambio de volumen: δ W = pdV Torsión de un alambre: δ W = − µ dθ Pila eléctrica reversible: δW = −ε dq Polarización dieléctrico: δW = − EdP Lámina superficial de un líquido: δ W = −σ ·dA Sistema magnético: δW = − µ0 VHdH − µ0 VHdm − µ0 VHdm Aumento del campo en el vacío. − µ0 VHdH Aumento del momento magnético del material. Relaciones entre las derivadas parciales: Dada una función implícita f ( x, y, z ) = 0 entre tres variables termodinámicas, de las cuales dos pueden seleccionarse como independientes, pudiéndose escribir esto de tres formas alternativas: x = x( y, z ) y = y ( x, z ) z = z ( x, y ) Según el teorema fundamental de la diferenciación: ⎛ ∂x ⎞ ⎛ ∂x ⎞ ⎛ ∂y ⎞ ⎛ ∂y ⎞ dx = ⎜ ⎟ dy + ⎜ ⎟ dz dy = ⎜ ⎟ dx + ⎜ ⎟ dz ⎝ ∂x ⎠ z ⎝ ∂z ⎠ x ⎝ ∂z ⎠ y ⎝ ∂y ⎠ z ⎛ ∂z ⎞ ⎛ ∂z ⎞ dz = ⎜ ⎟ dy + ⎜ ⎟ dx ⎝ ∂x ⎠ y ⎝ ∂y ⎠ x De estas tres ecuaciones, despejando, es posible deducir expresiones diferenciales del tipo: −1 ⎛ ∂x ⎞ ⎛ ∂y ⎞ ⎜ ⎟ =⎜ ⎟ ⎝ ∂y ⎠ z ⎝ ∂x ⎠ z ⎛ ∂x ⎞ ⎛ ∂y ⎞ ⎜ ⎟ ⎜ ⎟ =1 ⎝ ∂y ⎠ z ⎝ ∂x ⎠ z ⎛ ∂x ⎞ ⎛ ∂x ⎞ ⎛ ∂z ⎞ ⎛ ∂y ⎞ ⎛ ∂x ⎞ ⎛ ∂z ⎞ ⎜ ⎟ ⎜ ⎟ ⎜ ⎟ = −1 ⎜ ⎟ = −⎜ ⎟ ⎜ ⎟ ⎝ ∂z ⎠ y ⎝ ∂y ⎠ x ⎝ ∂y ⎠ z ⎝ ∂x ⎠ y ⎝ ∂z ⎠ x ⎝ ∂y ⎠ z Imaginemos que dividimos por dw las tres relaciones diferenciales: dx ⎛ ∂x ⎞ dy ⎛ ∂x ⎞ dz dy ⎛ ∂y ⎞ dx ⎛ ∂y ⎞ dz =⎜ ⎟ +⎜ ⎟ =⎜ ⎟ +⎜ ⎟ dw ⎝ ∂x ⎠ z dw ⎝ ∂z ⎠ x dw dw ⎝ ∂y ⎠ z dw ⎝ ∂z ⎠ y dw dz ⎛ ∂z ⎞ dy ⎛ ∂z ⎞ dx =⎜ ⎟ +⎜ ⎟ dw ⎝ ∂y ⎠ x dw ⎝ ∂x ⎠ y dw Y considerásemos como constante una de variables x, y, z, entonces obtendríamos relaciones del tipo: ⎛ ∂x ⎞ ⎛ ∂x ⎞ ⎛ ∂w ⎞ ⎛ ∂x ⎞ ⎛ ∂x ⎞ ⎛ ∂x ⎞ ⎛ ∂w ⎞ ⎜ ⎟ =⎜ ⎟ ⎜ ⎟ =⎜ ⎟ +⎜ ⎟ ⎟ ⎜ ⎟ ⎜ ⎝ ∂y ⎠ z ⎝ ∂w ⎠ z ⎝ ∂y ⎠ z ⎝ ∂y ⎠ z ⎝ ∂y ⎠ w ⎝ ∂w ⎠ y ⎝ ∂y ⎠ z 22
  • 24. Julián Moreno Mestre Primer principio de la termodinámica: dU = δ Q − δ W Capacidades caloríficas en sistemas hidrostáticos y sus valores: ⎛ ∂H ⎞ ⎛ ∂U ⎞ Cp = ⎜ CV = ⎜ ⎟ ⎟ ⎝ ∂T ⎠V ⎝ ∂T ⎠ p - Monoatómicos: CV = 1.5nR y C P = 2.5nR - Biatómicos: CV = 2.5nR y C P = 3.5nR Relación de Mayer: α 2TV ⎛ ∂p ⎞ ⎛ ∂V ⎞ C p − CV = T ⎜ = = nR ⎟ ⎜ ⎟ κT ⎝ ∂T ⎠V ⎝ ∂T ⎠ p Coeficiente adiabático y trabajo adiabático, transformaciones adiabáticas: C piVi − p f V f γ= p Wad = γ −1 CV TV γ −1 = cte pV γ = cte Coeficientes calorimétricos de un sistema p, V, T: δ Q = C p dT + hdp δ Q = CV dT + ldV p1−γ T γ = cte δ Q = λ dV + µ dp C p − CV ⎛ ∂T ⎞ l = ( C p − CV ) ⎜ ⎟ = βV ⎝ ∂V ⎠ p Cp ⎛ ∂T ⎞ ⎟ = ⎝ ∂V ⎠ p βV λ = Cp ⎜ ⎛ ∂T ⎞ CV κ = CV T ⎟ = β ⎝ ∂p ⎠V α p µ = CV ⎜ CV − C p ⎛ ∂T ⎞ ⎛ ∂V ⎞ ⎛ ∂V ⎞ κ = ( CV − C p ) T = l ⎜ h = ( CV − C p ) ⎜ ⎟ = ⎟ = λ⎜ ⎟ +µ αp β ⎝ ∂p ⎠V ⎝ ∂p ⎠T ⎝ ∂p ⎠T 23
  • 25. Ejercicios y problemas de Termodinámica I Problemas: 1º Determine el trabajo realizado en la expansión isoterma de un mol de gas ideal a la temperatura de 300 K cuando: a) El gas se expansiona en una etapa, desde 10 atm a 1 atm, contra una presión exterior constante de 1 atm. b) La expansión se realiza en dos etapas. En la primera el gas se expansiona desde 10 a 5 atm, contra una presión exterior constante de 5 atm. En la segunda el gas se expansiona desde 5 a 1 atm, contra una presión exterior constante de 5 atm. En la segunda el gas se expansiona desde 5 atm a 1 atm, contra una presión exterior constante de 1 atm. c) La expansión se realiza en tres etapas: 1) desde 10 a 5 atm a presión exterior constante de 5atm, 2) desde 5 a 2 atm a presión exterior constante de 2 atm, 3) desde 2 a 1 atm a presión exterior constante de 1 atm. d) La expansión se realiza en 9 etapas, desde 10 a 1 atm, reduciendo progresivamente presión exterior en incrementos de 1 atm. e) La expansión se realiza en un número infinito de etapas, haciendo que la presión externa sea un infinitésimo inferior de la presión interna en cada etapa sucesiva. Compárese entre sí los resultados obtenidos en cada uno de los apartados anteriores. Solución: Dado que trabajamos con un solo mol de gas ideal. Resumimos la ecuación de estado en: pV = RT Así calcularemos los volúmenes finales e iniciales en las etapas sugeridas por los apartados según: RT V= p Siendo los volúmenes ocupados por el gas en función de la presión: p(atm) 10 9 8 7 6 5 4 3 2 1 V (L) 2.46 2.73 3.07 3.51 4.1 4.92 6.15 8.20 12.3 24.6 Ahora realizaremos fácilmente los cálculos para los cuatro primeros apartados del problema, observando como tomando diferentes caminos para llegar desde el estado inicial al estado final los trabajos son distintos, lo cual es conforme con la teoría que el trabajo en termodinámica no es función de estado. a) Expansión en una etapa a presión exterior constante de 1 atm: W= V2 ∫ p dV = p (V 2 2 2 − V1 ) = 22.14 atm·L = 2243.7 J V1 b) Expansión en dos etapas: Etapa 1: W1 = V2 ∫ p dV = p (V 2 2 2 − V1 ) = 1246.5 J V1 Etapa 2: W2 = V3 ∫ p dV = p (V 3 3 3 − V2 ) = 1994.4 J V2 El trabajo total es: W = W1 + W2 = 3240.9 J 24
  • 26. Julián Moreno Mestre c) Expansión en tres etapas: Etapa 1: W1 = V2 ∫ p dV = p (V 2 2 2 − V1 ) = 1246.5 J V1 V3 ∫ p dV = p (V Etapa 2: W2 = 3 3 3 − V2 ) = 1495.8 J V2 Etapa 3: W3 = V4 ∫ p dV = p (V 4 4 4 − V3 ) = 1246.5 J V3 El trabajo total es: W = W1 + W2 + W3 = 3988.8 J d) Expansión en 9 etapas, calculamos los trabajos como en los procedimientos anteriores: W1 = p2 (V2 − V1 ) = 246.2 J W2 = p3 (V3 − V2 ) = 275.5 J W3 = p4 (V4 − V3 ) = 312.0 J W4 = p5 (V5 − V4 ) = 358.6 J W5 = p6 (V6 − V5 ) = 415.3 J W6 = p7 (V7 − V6 ) = 498.4 J W7 = p8 (V8 − V7 ) = 623.0 J W8 = p9 (V9 − V8 ) = 830.7 J W9 = p10 (V10 − V9 ) = 1246.0 J 9 El trabajo total es por tanto: W = ∑ Wi =4805.7 J i =1 e) En este caso se van a ir produciendo cambios infinitesimales de volumen y de presión. Hay que hacer por tanto el cálculo del trabajo como un proceso isotermo y reversible: V2 V2 ⎡V ⎤ dV W = ∫ pdV = RT ∫ = RT ln ⎢ 2 ⎥ = 5738.0 J V ⎣ V1 ⎦ V1 V1 2º Un mol de gas perfecto se expande isotérmica e irreversiblemente desde la presión inicial de 10 atm contra una presión exterior de 6 atm, y una vez alcanzado el equilibrio vuelve a expandirse bruscamente de modo isotérmico contra la presión exterior constante de 3 atm hasta alcanzar de nuevo el equilibrio. Calcúlese en julios el trabajo total realizado por el gas si la temperatura es en todo momento de 300 K. Solución: Se trata de un proceso en dos etapas. Comenzamos aplicando la ecuación de estado del gas ideal (para n = 1): pV = RT Para determinar los volúmenes de los estados inicial, intermedio y final p(atm) 10 6 1 V (L) 2.46 4.1 24.6 Calculamos los trabajos para cada etapa: Etapa 1: W1 = V2 ∫ p dV = p (V 2 2 2 − V1 ) = 996.8 J V1 Etapa 2: W2 = V3 ∫ p dV = p (V 3 V2 25 3 3 − V2 ) = 2076.6 J
  • 27. Ejercicios y problemas de Termodinámica I El trabajo total es: W = W1 + W2 = 3073.4 J 3º Considérense tres estados de equilibrio; A(3 bar, 1 L), B(1 bar, 3 L) y C(1 bar, 1 L), de cierto sistema hidrostático. Calcule el trabajo realizado por el sistema cuando pasa del estado A al B a través de cada una de las siguientes trayectorias del plano pV: a) arco de circunferencia, con centro en C, que une A con B. b) línea recta uniendo A con B. c) Proceso isócoro (AC), seguido de un proceso isóbaro (CB). Solución: En los tres apartados emplearemos la expresión integral del trabajo: Vf W= ∫ pdV Vi imponiendo los respectivos caminos de cada apartado: a) Se trata de un camino de arco de circunferencia, en especial medio arco de una circunferencia tal y como podemos observar en el diagrama pV. El trabajo es el área sombreada. Para calcular la relación entre la presión y el volumen siguiendo el arco de circunferencia utilizamos la expresión matemática de la circunferencia de radio R centrada en un punto de coordenadas (xc, yc): ( x − xc ) 2 + ( y − yc ) 2 = R 2 Sustituimos las variables x e y por p y V, y además podemos comprobar que se trata de una circunferencia de R = 2. Consideraremos además que esta ecuación que describe la trayectoria pV de arco de circunferencia es además adimensional. 2 2 (V − Vc ) + ( p − pc ) = R 2 Despejando la presión p hasta describir un semiarco del primer cuadrante: p = R 2 − (V − Vc ) + pc 2 y sustituyendo en la integral: W= ∫( VB ) R 2 − (V − Vc ) + pc dV 2 VA El cálculo de esta integral requiere de utilizar bastantes herramientas de cálculo, por ello recurriendo a un libro de tablas de integrales podemos ver que las integrales del tipo: x R 2 − x2 R 2 ⎛x⎞ arcsin ⎜ ⎟ + ∫ 2 2 ⎝R⎠ y así realizar el cálculo. O bien recurrimos a Derive planteándole la integral: R 2 − x 2 dx = 3 W =∫ 1 ( ) 22 − (V − 1) + 1 dV = π + 2 bar·L = 520.2 J 2 b) La trayectoria la determinamos con la expresión de la ecuación de la recta que pasa por dos puntos: x − x0 y − y0 = x1 − x0 y1 − y0 Sustituyendo por nuestras variables x e y por nuestras variables p y V: 26
  • 28. Julián Moreno Mestre V − VA p − pA V − VA V −1 = → p= ( pB − pA ) + pA = (1 − 3) + 3 = −V + 4 VB − VA pB − pA VB − VA 3 −1 Integrando ahora: 3 VA W= VB 1 ∫ pdV = ∫ −V + 4dV = 4 bar·L = 400 J c) Viendo la gráfica pV planteada, obviamente solo hay que calcular el trabajo en el proceso isóbaro (CB) y olvidarse de calcularlo en el isócoro (AC), ya que en el es evidente que no se produce trabajo. W= VB ∫ pdV = p (V B − VC ) = 1(3 − 1) = 2 bar·L = 200 J VC 4º Calcular el trabajo efectuado por un mol de gas que obedece a la ecuación de Van der Waals al expansionarse desde el volumen V1 al volumen V2 en los siguientes casos: a) Por vía reversible e isoterma. b) Por vía irreversible a presión constante. Determínese las expresiones del trabajo en ambos casos si el gas fuese ideal. Solución: La ecuación de Van der Waals es: a ⎞ RT a ⎛ − 2 ⎜ p + 2 ⎟ (V − b) = RT → p = V ⎠ V −b V ⎝ a) Expansión isoterma reversible, es decir, que no es brusca, luego: W= V2 ∫ V1 V V2 2 a ⎞ V ⎛ RT ⎡a⎤ pdV = ∫ ⎜ − 2 ⎟ dV = RT [ ln(V − b) ]V2 + ⎢ ⎥ 1 V −b V ⎠ ⎣ V ⎦V1 V1 ⎝ ⎛V −b ⎞ a a W = RT ln ⎜ 1 ⎟+ − ⎝ V2 − b ⎠ V1 V2 b) Se trata de una expansión a presión exterior constante: W= V2 ∫ pdV → W = p(V 2 − V1 ) V1 Si el gas hubiese sido ideal, entonces la ecuación de estado sería: RT pV = RT → V = p y ambos trabajos serían (recontestando a los apartados): a) Expansión isoterma reversible: W= V2 ∫ V1 pdV = V2 ∫ V1 ⎛V ⎞ RT V dV = RT [ ln(V )]V2 → W = RT ln ⎜ 2 ⎟ 1 V ⎝ V1 ⎠ b) Expansión isobara: W= V2 ∫ pdV → W = p(V 2 V1 27 − V1 )
  • 29. Ejercicios y problemas de Termodinámica I 5º Un mol de gas ideal evoluciona cuasiestáticamente desde el estado A hasta el estado C. Determínese el trabajo realizado cuando la evolución tiene lugar a lo largo de los caminos I y II indicados en el diagrama pT de la figura. Solución: Empezaremos por el camino I. En este camino determinaremos la ecuación de la recta que pasa por los puntos que marca A y C: p − p1 T − T1 T −T = → T = ( p − p1 ) 2 1 + T1 p2 − p1 T2 − T1 p2 − p1 Sustituimos en la ecuación de estado del gas ideal (para n = 1): T −T R T − T RT pV = RT → pV = R( p − p1 ) 2 1 + RT1 → V = ( p − p1 ) 2 1 + 1 p2 − p1 p p2 − p1 p Determinamos el diferencial de volumen: ⎡ ⎤ dp T −T dV = ⎢ Rp1 2 1 − RT1 ⎥ 2 p2 − p1 ⎣ ⎦p y procedemos a calcular ahora el trabajo realizado por el camino I: V2 p2 ⎡ ⎤ dp ⎡ ⎤ ⎛p ⎞ T −T T −T WI = ∫ p2 dV = ∫ p ⎢ Rp1 2 1 − RT1 ⎥ 2 = ⎢ Rp1 2 1 − RT1 ⎥ ln ⎜ 2 ⎟ p2 − p1 p2 − p1 ⎣ ⎦p ⎣ ⎦ ⎝ p1 ⎠ V1 p1 ⎡ ⎤ ⎛p ⎞ T −T WI = ⎢ Rp1 2 1 − RT1 ⎥ ln ⎜ 2 ⎟ p2 − p1 ⎣ ⎦ ⎝ p1 ⎠ Por el camino 2 el trabajo pasa por un proceso isotermo primero y por un proceso isóbaro después. El trabajo realizado en el proceso isotermo (de A a B) es: VB VB ⎛V ⎞ dV WIIIsotermo = − ∫ pdV = − RT1 ∫ = RT1 ln ⎜ 1 ⎟ V ⎝ VB ⎠ V1 V1 En el proceso isóbaro es: V2 WIIIsóbaro = − ∫ pdV = − p (V2 − VB ) VB El trabajo total en el camino 2 es: ⎛V ⎞ WII = WIIIsotermo + WIIIsóbaro = RT1 ln ⎜ 1 ⎟ − p (V2 − VB ) ⎝ VB ⎠ Cambiando los volúmenes por los datos dados por el problema: RT RT RT V1 = 1 VB = 1 V2 = 2 p1 p2 p2 ⎛p ⎞ WII = RT1 ln ⎜ 2 ⎟ − R(T 2 −T1 ) ⎝ p1 ⎠ 28
  • 30. Julián Moreno Mestre 6º La presión de un gas en un cilindro dotado de un pistón desplazable varia con el volumen según la expresión p = C/V 2. Donde C es una constante. Si la presión inicial es 500 kPa, el volumen inicial es 0.05 m3, y la presión final, 200 kPa, determínese el trabajo realizado en el sistema. Solución: Pasamos a atmósferas las presiones y los volúmenes a litros: pi = 4.936 atm Vi = 50 L p f = 1.974 atm A partir de la ecuación de estado determinamos el volumen final: C p = 2 → C = piVi 2 = p f V f2 → V f = 79.057 L V Siendo la constante de la ecuación de estado: C = piVi 2 = 12339.6 atm·L2 Calculamos ahora el trabajo: Vf W= ∫ Vf pdV = Vi 7º V f C ⎡C ⎤ dV = − ⎢ ⎥ = 9188.6 J ∫ V2 ⎣ V ⎦Vi Vi Un mol de agua se comprime reversiblemente en una prensa hidráulica, a la temperatura constante de 20 ºC, desde la presión inicial de una atmósfera hasta la presión final de cien atmósferas. Calcúlese el trabajo realizado en el proceso sabiendo que, a esta temperatura, el coeficiente de compresibilidad isotermo y la densidad valen 45.30·10-6 atm-1 y 0.9982 g·cm-3, respectivamente. Soluciones: Lo primero es determinar la ecuación de estado que rige este proceso: 1 ⎛ ∂V ⎞ ∂V κT = − ⎜ → κ T p = − ln V + K → V = Ke−κT p → K = VeκT p ⎟ → κ T ∂p = − V ⎝ ∂p ⎠T V Determinamos el volumen de agua a 1 atm de presión, con la densidad y partiendo del número de moles y la masa molar del agua (18 g/mol): m 18·n V= = = 1.80324·10−2 L ρ ρ Determinamos ahora el volumen final con la ecuación de estado: K = 1.80324·10−2 ·e 45.3·10 −6 = V f ·e45.3·10 −4 → V f = 1.79517·10−2 L Determinamos también el valor de la constante de la ecuación de estado: K = 1.80324·10−2 ·e45.3·10 Diferenciamos la ecuación de estado: −6 = 1.80332·10−2 L K = VeκT p → 0 = eκT p dV + V κ T eκT p dp → dV = −V κ T dp = K κT p e κ T dp Y con la expresión del trabajo: V2 100 V1 1 W = − ∫ pdV = − ∫ 29 K p κ p κ T dp = − K κ T e T 100 ∫ eκ 1 100 p T ⎡ e −κ p (κ T p + 1) ⎤ dp = − Kκ T ⎢ − ⎥ = 0.412 J κ T2 ⎣ ⎦1 T p
  • 31. Ejercicios y problemas de Termodinámica I 8º El nitrógeno líquido es un material dieléctrico. En un cierto intervalo de temperaturas, la polarización eléctrica o momento bipolar por unidad de volumen, P, está relacionado con el campo E por: P = ε 0 (23.8 − 4.16·10−4 E)E donde P está expresado C·m–2, E en V·m–1 y ε 0 es la permitividad del vacío igual a 8.85·10–12 C2·N–1·m–2. Calcúlese el trabajo realizado por cm3 de nitrobenceno cuando el campo eléctrico aumenta de 0 a 104 V·m-1 e indíquese si se da o se recibe trabajo del sistema. Solución: A partir de la expresión del trabajo para este tipo de sistemas dieléctricos: P2 W = − ∫ EdP P1 Determinamos el diferencial de polarización con la expresión del ejercicio: P = ε 0 (23.8 − 4.16·10−4 E)E → P = ε 0 (23.8 − 8.32·10−4 E)dE Por tanto el trabajo: 104 ⎛ 108 1012 ⎞ −9 W = − ∫ Eε 0 (23.8 − 8.32·10−4 E)dE = ε 0 ⎜ −23.8 + 8.32·10−4 ⎟ = 8.08·10 J 2 2 ⎠ ⎝ 0 9º Una muestra de bismuto de masa 27.3 g, cuya susceptibilidad magnética a la temperatura ambiente es χm = – 1.32·10– 5, se introduce en un solenoide que crea un campo de inducción magnética uniforme de diez teslas. Determinar la imanación final de la muestra y el trabajo que realiza. Datos: µ0 = 4π·10– 7 T·m·A–1; densidad del bismuto ρ = 9.80 g·cm–3. Solución: Sabiendo que la susceptibilidad magnética se define como: M χm = H Donde M es la imanación, y H el campo magnético en el material. Sabiendo que: B B H= = µ µ0 (1 + χ m ) Determinamos la imanación final: χmB M= = −105.04 A·m −1 µ0 (1 + χ m ) El volumen de bismuto que tenemos es: m V = = 2.78·10−6 m3 ρ La expresión del trabajo es: M2 M2 V µ0 ⎡ M ⎤ 2 −3 W = − ∫ µ0 HVd M = −V ∫ µ0 dM = ⎢ 2 ⎥ = 1.46·10 J χm χm ⎣ ⎦M 1 M1 M1 M M 30
  • 32. Julián Moreno Mestre 10º Un sólido paramagnético ideal de volumen V adquiere, cuasiestáticamente, una imanación m en presencia de un campo magnético externo variable. La temperatura del sólido varía durante la imanación según la ley: 2 T = T0 e K m siendo K una constante. Hállese el trabajo realizado en la imanación del sólido en función de la temperatura. Solución: En sistemas magnéticos la expresión del trabajo es: m2 W = − ∫ µ0V Hdm m1 A partir de la ecuación dada por el problema, determinamos el dm: 2 ⎛T ⎞ dT 1 dT T = T0 e K m → ln ⎜ ⎟ = K m 2 → 2Kmdm = → 2mdm = T K T ⎝ T0 ⎠ Y con la ley de Curie: H m=C T Procedemos a aplicarla a nuestro diferencial: 1 dT H 1 dT dT 2mdm = → 2C dm = → dm = K T K T 2CKH T Sustituimos en la expresión del trabajo: m2 T µV µV T dT W = − ∫ µ0V Hdm = − ∫ µ0V H = − 0 ∫ dT → W = − 0 (T − T0 ) 2CK 2CKH 2CK T m T 1 0 0 11º Un mol de gas ideal está en equilibrio a 6 atm de presión y volumen 10 L. Se le enfría isócoramente hasta alcanzar una presión igual a la mitad de su presión inicial. A continuación se calienta a presión constante hasta que alcanza un volumen, Vf, tal que en una compresión isoterma final regresa a su estado inicial. a) Dibuje el proceso en un diagrama de Clapeyron (p,V) b) Calcule el trabajo neto realizado en el ciclo. Solución: Exponemos la información que tenemos en el problema: pV TA = TC = = 731.7 K pA = 6 atm VA = VB = 10 L R RT pV TB = B B = 365.85 K VC = C = 20 L pB = pC = 3 atm R pC Solo realizan trabajo la etapa isoterma y la etapa isóbara. Por tanto: W = Wisóbara + Wisócora = pB (VC − VB ) + VA ∫ VC ⎛V W = pB (VC − VB ) + RTC ln ⎜ A ⎝ VC 31 VA pdV = pB (VC − VB ) + RTC ∫ ⎞ ⎟ = −11.59 atm/L = −1174 J ⎠ VC dV V
  • 33. Ejercicios y problemas de Termodinámica I 12º Para comprimir adiabáticamente 2 moles de un gas ideal biatómico cuya temperatura inicial es de 300 K, ha sido necesario suministrarle 800 J en forma de trabajo. ¿Cuál es la temperatura final del gas? Solución: En un proceso adiabático el incremento de energía interna es igual al trabajo suministrado. Por tanto la temperatura final la determinamos mediante el primer principio: 5 ∆U = −W → nCV (T f − Ti ) = 2 8.31(T f − 300 ) = 800 → T f = 319.25 K 2 13º Un mol de gas ideal monoatómico se encuentra en un cilindro provisto de un pistón móvil. Si la presión externa sobre el pistón se mantiene constante en 1 atm, ¿Qué cantidad de calor debe aportarse al gas para aumentar su volumen de 20 a 50 litros? Solución: El gas realiza una expansión isobara. La temperatura inicial y final será de: pV f pV Tf = = 609.8 K Ti = i = 243.9 K nR nR Determinamos el calor que es necesario aportarle mediante el primer principio: 3 Q = ∆U + W = CV ∆T + p∆V = R∆T + p∆V = 7600 J 2 14º Un recipiente de 15 m3 contiene radiación electromagnética en equilibrio con sus paredes a la temperatura de 300 K. Dicha radiación se comporta como un sistema (gas de fotones) con ecuaciones térmica de estado y energética dadas por las expresiones: aT 4 p= U = aVT 4 3 donde a = 7.56·10-16 J·m-3·K-4. a) Calcúlese el calor absorbido por el sistema de radiación en un proceso isotermo reversible en el que su volumen se duplica. b) Obténgase, por otra parte, la ecuación que rige, en coordenadas (p,V ), los procesos adiabáticos reversibles experimentados por dicho sistema. Solución: a) Partiendo del primer principio de la termodinámica: ∆U = Q − W → Q = ∆U + W Determinamos ahora el calor transferido a partir del cambio de energía interna y el trabajo. Comenzamos por el trabajo, el cual es realizado por el sistema sobre los alrededores, luego es positivo: Vf Vi W= Vf Vi ∫ pdV = ∫ 4aT 4 4aT 4 4aT 4 4aT 4 (V f − Vi ) = (2Vi − Vi ) = dV = Vi 3 3 3 3 El cambio de energía interna depende en este caso del volumen: ∆U = U f − U i = aV f T 4 − aViT 4 = 2aViT 4 − aViT 4 = aViT 4 Por tanto el calor: aV T 4 4aViT 4 Q = ∆U + W = aViT 4 + i = = 1.225·10−4 J 3 3 b) Partiendo del primer principio con Q = 0 y diferenciando: aT 4 dU = − dW → aT dV + 4aVT dT = − pdV → aT dV + 4aVT dT = − dV 3 4 3 4 3 32
  • 34. Julián Moreno Mestre T 4T dT 4dV dT 4 dV dV → 4VdT = − dV → 4 =− → 4∫ =− ∫ 3 3 3V 3 V T T 4 4 ln T = − ln V + cte → cte = ln (T 4V 4 / 3 ) → cte = T 4V 4 / 3 3 Y ahora a partir de la ecuación de estado que relaciona presión y temperatura: aT 4 3p 3 p 4/3 p= →T4 = cte = T 4V 1/ 3 → cte = V → cte = pV 4 / 3 3 a a 15º Un mol de agua a 24 ºC y 1.013 bar se calienta a presión constante hasta 100 ºC. El coeficiente de dilatación cúbica del agua vale 4·10–4 K–1. Calcúlese el cambio de energía interna experimentado por el agua, considerando que la densidad media del líquido en dicho intervalo de temperaturas es de 1 g·cm–3 y el calor específico medio a presión constante vale 4.18·103 J·kg–1 ·K–1. TdV + 4VdT = − Solución: En el presente problema tenemos un exceso de datos para calcular el cambio de energía interna. Nos basta con conocer el calos específico del agua, el incremento de temperatura y la masa de agua (1 mol de agua = 18 g de agua) que tenemos: ∆U = mce ∆T = 5718.24 J 16º Un mol de gas biatómico experimenta cambios reversibles desde pi = 10 atm y V1 = 10 L a pf = 1 atm, de acuerdo con los siguientes procesos: a) V = cte b) T = cte, c) Adiabáticamente Calcúlense W, Q, ∆U y ∆H en cada proceso. Represéntese los procesos en un diagrama pV. Solución: Determinamos primero la temperatura inicial con la ecuación de estado del gas ideal: pV pV = RT → T = = 1219.5 K R Como se trata de un proceso de expansión o contracción de un gas ideal biatómico, entonces las funciones de estado entalpía y energía interna obedecerán a las expresiones (con n = 1): ∆U = CV ∆T ∆H = C p ∆T Donde: 5 7 R Cp = R 2 2 a) Como es a V = cte, determinaremos la temperatura final: pV p f V f = RTi = piVi → V f = i i = 100 L pf como es un proceso isócoro, el trabajo es nulo. La entalpía y la energía interna es: ∆U = CV ∆T = −22792.3 J ∆H = C p ∆T = −31909.5 J CV = El calor es igual al incremento de energía interna según el primer principio. 33
  • 35. Ejercicios y problemas de Termodinámica I b) Se trata de un proceso isotermo. Dado que no existen transiciones de fase en el gas, el cambio de energía interna y el de entalpía son nulos. Por el primer principio, el calor es igual al trabajo. Determinaremos primero el volumen final: pV pV = RT → T = f i = 121.95 K R y ahora ya podemos calcular el calor y el trabajo: Vf Vf ⎛V ⎞ RT Q = W = − ∫ pdV = − ∫ dV = − RT ln ⎜ f ⎟ = −23325.0 J V ⎝ Vi ⎠ Vi Vi c) Al tratarse de un proceso adiabático, el gas se va regir (en el proceso) por la ecuación de estado siguiente: p1−γ T γ = cte al tratarse de un gas ideal biatómico (cumple con la ecuación de estado del gas ideal), el coeficiente adiabático γ = 1.4 . Por tanto determinamos la temperatura final y el volumen final del gas: cte = pi1−γ Ti1−γ = p1f−γ T f1−γ → T f = 631.6 K p f V f = RT f → V f = 51.79 L Determinamos ahora el cambio de energía interna y la entalpía del proceso: ∆U = CV ∆T = −12208.6 J ∆H = C p ∆T = −17092.1 J El trabajo adiabático obedece a la expresión: p V − p2V2 W= 1 1 = −12209.2 J γ −1 Como es un proceso adiabático, el calor es cero. 17º Un cilindro vertical de paredes adiabáticas y 100 cm de altura está dividido en dos partes por una membrana impermeable que se encuentra a 50 cm de la base. La parte superior del cilindro está cerrada por un pistón adiabático sobre el que se ejerce una presión exterior constante. Inicialmente la parte inferior está vacía, mientras que la parte superior contiene un mol de gas ideal monoatómico a 300 K, encontrándose el pistón a 100 cm de altura. En un momento determinado se rompe la membrana y, consecuencia, el pistón desciende. Determínese la altura a la que se detiene el pistón una vez que se ha alcanzado el equilibrio y el trabajo realizado sobre el gas. Solución: Antes de la rotura de la membrana, las variables de estado del sistema valen: RT V1 = 1 = 24.6 L T1 = 300 K p1 = 1 atm p1 Después de la rotura de la membrana, la energía interna no cambia, la temperatura es la misma, el volumen se duplica y la presión es por tanto la mitad: T2 = 300 K p2 = 0.5 atm V2 = 29.2 L Pero al poco de romperse la membrana el pistón comienza a contraer el gas, y lo hace de forma adiabática. Dado que es un gas monoatómico, el coeficiente adiabático del gas es γ = 1.67 . El gas va a tener una presión final igual a la atmosférica (p3 = 1 atm). Siguiendo las relaciones entre dos estados unidos por una adiabática determinaremos el volumen final del gas: p3V3γ = p2V2γ → V3 = 32.5 L 34
  • 36. Julián Moreno Mestre La altura que adquiere el pistón la calculamos mediante una regla de tres: V1 h1 24.6 L 60 cm = → = → h3 = 66 cm V3 h3 h3 32.5 L El trabajo (adiabático) realizado sobre el gas es: p V − p3V3 W= 2 2 = −11.77 atm·L= −1192.9 J 1− γ 18º Un gas ideal biatómico, encerrado en un cilindro de paredes adiabáticas, se calienta a la presión constante de 2 bar mediante una resistencia de capacidad calorífica despreciable. El volumen ocupado por el gas aumenta de 25 a 42 L en 6 minutos. Calcúlese: a) El cambio de energía interna experimentado por el gas. b) El trabajo eléctrico suministrado a la resistencia. c) La intensidad que circula por la resistencia de valor 100 Ω. Solución: No es posible calcular ni el número de moles ni la temperatura de los diferentes de estados. No obstante como es un proceso isóbaro: pV1 = nRT1 pV2 = nRT2 Pasamos a atmósferas los 2 bares de presión, por tanto la presión es de 1.9738 atm. Ahora respondemos a los apartados: a) Utilizaremos en este apartado las relaciones anteriores de las variables de estado para calcular el cambio de energía interna: 5 5 5 ∆U = CV ∆T = nR(T2 − T1 ) = nRT2 − nRT1 = 8498 J 2 2 2 b) El trabajo realizado es: W= V2 ∫ pdV = p(V 2 − V1 ) = 3399 J V1 c) Calculamos el trabajo eléctrico realizado sobre el sistema por la resistencia: ∆U = We − W → We = ∆U + W = 11897 J Por la Ley de Joule del calentamiento eléctrico de las resistencias: We We = I 2 Rt → I = = 0.575 A Rt 19º Partiendo de la ecuación calorimétrica δ Q = C p dT + hdp demuéstrese que en un proceso adiabático se cumple: γ ⎛ ∂p ⎞ ⎛ ∂p ⎞ ⎜ ⎟ = ⎜ ⎟ ⎝ ∂T ⎠Q γ − 1 ⎝ ∂T ⎠V Solución: Al ser un proceso adiabático δ Q = 0 , luego: Cp ⎛ ∂p ⎞ 0 = C p dT + hdp → C p dT = −hdp → ⎜ ⎟ = ⎝ ∂T ⎠Q − h Dado que el coeficiente calorimétrico es: ⎛ ∂T ⎞ h = −(C p − CV ) ⎜ ⎟ ⎝ ∂p ⎠V 35
  • 37. Ejercicios y problemas de Termodinámica I por tanto: Cp ⎛ ∂p ⎞ ⎜ ⎟ = ⎝ ∂T ⎠Q C p ⎛ ∂p ⎞ CV ⎛ ∂p ⎞ ⎛ ∂p ⎞ ⎛ ∂p ⎞ →⎜ ⎟ = ⎜ ⎟ →⎜ ⎟ =C ⎟ C ⎜ ∂T ⎠V ⎛ ∂T ⎞ p ⎝ ∂T ⎠Q C p − CV ⎝ ∂T ⎠V ⎝ ∂T ⎠Q − V ⎝ (C p − CV ) ⎜ ⎟ CV CV ⎝ ∂p ⎠V γ ⎛ ∂p ⎞ ⎛ ∂p ⎞ ⎜ ⎟ = ⎜ ⎟ ⎝ ∂T ⎠Q γ − 1 ⎝ ∂T ⎠V Demostrado Cp 20º Demuéstrese que entre las capacidades a presión y volumen constante se cumple, para todos los sistemas pVT: ⎡ ⎛ ∂U ⎞ ⎤ ⎛ ∂V ⎞ C p − CV = ⎢ p + ⎜ ⎟ ⎥⎜ ⎟ ⎝ ∂V ⎠T ⎦ ⎝ ∂T ⎠ p ⎣ Solución: Partiendo del primer principio de la termodinámica: dU = δ Q − pdV y de las relaciones diferenciales: ⎛ ∂U ⎞ ⎛ ∂U ⎞ dU = ⎜ ⎟ dT + ⎜ ⎟ dV ⎝ ∂T ⎠V ⎝ ∂V ⎠T Sustituyendo en el primer principio: ⎛ ∂U ⎞ ⎛ ∂U ⎞ ⎜ ⎟ dT + ⎜ ⎟ dV = δ Q − pdV ⎝ ∂T ⎠V ⎝ ∂V ⎠T dV ⎛ ∂U ⎞ ⎛ ∂U ⎞ dV δ Q = −p ⎜ ⎟ +⎜ ⎟ dT ⎝ ∂T ⎠V ⎝ ∂V ⎠T dT dT Si ahora consideramos constante la presión: ⎛ ∂U ⎞ ⎛ ∂U ⎞ ⎛ ∂V ⎞ ⎛ δ Q ⎞ ⎛ dV ⎞ ⎜ ⎟ +⎜ ⎟ ⎜ ⎟ =⎜ ⎟ − p⎜ ⎟ ⎝ ∂T ⎠V ⎝ ∂V ⎠T ⎝ ∂T ⎠ p ⎝ dT ⎠ p ⎝ dT ⎠ p ⎛ ∂U ⎞ ⎛ ∂V ⎞ ⎛ ∂V ⎞ ⎛ δ Q ⎞ ⎛ ∂U ⎞ ⎜ ⎟ ⎜ ⎟ + p⎜ ⎟ =⎜ ⎟ −⎜ ⎟ ⎝ ∂V ⎠T ⎝ ∂T ⎠ p ⎝ ∂T ⎠ p ⎝ dT ⎠ p ⎝ ∂T ⎠V ⎤ ⎛ ∂V ⎞ ⎡⎛ ∂U ⎞ ⎜ ⎟ ⎢⎜ ⎟ + p ⎥ = C p − CV ⎝ ∂T ⎠ p ⎣⎝ ∂V ⎠T ⎦ Demostrado. 21º Dos moles de un gas ideal (cV = 2.5R) realizan un proceso adiabático y cuasiestático desde las condiciones p1 = 12 atm y V1 = 1 L hasta que su presión se reduce al valor ambiental p2 = 1 atm. Hallar el trabajo intercambiado y los incrementos de energía interna y de entalpía experimentados. Solución: Al ser cV = 2.5R, esto implica que cp = 3.5R y γ = 1.4. El volumen final, temperatura inicial y final es: p pV γ = cte → p1V1γ = p2V2γ → V2 = γ 1 V1 = 5.9 L p2 36
  • 38. Julián Moreno Mestre p1V1 pV = 73.17 K T2 = 2 2 = 35.98 K nR nR Como es adiabático, esto implica que el calor es nulo, y por tanto el trabajo es igual al cambio de la energía interna. El trabajo y la energía interna es: p V − p2V2 12·1 − 1·5.9 ∆U = −W = − 1 1 = = −15.25 atm·L = −1544.8 J 0.4 γ −1 El cambio en la entalpía es: ∆H = C p ∆T = −2163.34 J T1 = 22º Una máquina realiza un ciclo triangular cuyos vértices en el plano (V, p) son A(1,1), B(2,1) y C(1,2), expresándose la presión en atmósferas y el volumen en litros. Determínese en rendimiento de este ciclo, admitiendo que la sustancia que lo recorre es un gas ideal diatómico. Solución: Representando gráficamente en un diagrama, podemos ver que el trabajo total realizado equivale a menos el área del triángulo. 1·1 W = = 0.5 atm/L = 50.65 J 2 Se trata de un gas ideal diatómico, esto implica que: 5 3 γ = 1.4 Cp = R CV = R 2 2 Por el gráfico se puede fácilmente deducir que: TC = TB = 2TA En la etapa BA (isobara) del ciclo el calor liberado es igual al incremento de entalpía: 7 7 7 QBA = ∆H = nC p ∆T = nR(TA − TB ) = − nRTA = − pAVA = −3.5 atm·L = −354.55 J 2 2 2 En la etapa CB, las temperaturas final e inicial son iguales, el cambio de energía interna es nulo, por el primer principio de la termodinámica establecemos que el calor es igual al trabajo realizado en esta etapa. La trayectoria seguida es una línea recta: p −1 V − 2 = → p −1 = 2 −V → p = 3 − V 2 −1 1− 2 El trabajo, y por tanto el calor, realizado en la trayectoria es: Q =W = VB ∫ VC 2 ⎡ V2⎤ pdV = ∫ (3 − V )dV = ⎢3V − ⎥ = 1.5 atm·L = 151.95 J 2 ⎦1 ⎣ 1 2 En la etapa AC, el trabajo es cero al no haber incremento de volumen. El calor es igual al cambio de energía interna: 5 5 5 QAC = ∆U AC = nCV ∆T = nR(TC − TA ) = nRTA = pAVA = 2.5 atm·L = 253.25 J 2 2 2 El calor absorbido o recibido por el ciclo es la suma de los calores positivos, siendo el calor liberado el negativo. La diferencia entre el recibido y el desprendido como se observa es igual al trabajo total. El rendimiento del ciclo es: W 50.65 η= = = 0.125 Qabsorbido 151.95 + 253.25 37
  • 39. Ejercicios y problemas de Termodinámica I 23º Un litro de aire se calienta a la presión atmosférica isobáricamente hasta duplicar su volumen. Dedúzcase la variación de energía interna y el rendimiento de la transformación. Se supone que el gas es perfecto y γ = 1.4. Solución: El trabajo realizado es: W = p∆V = p (V2 − V1 ) = 1 atm·L = 101.3 J El incremento de energía interna y de entalpía es: ∆U = CV ∆T ∆H = C p ∆T Por tanto y dado que: γ= Cp = 1.4 CV nC p ∆T = nCV ∆T + p∆V → 1.4nCV ∆T = nCV ∆T + p∆V → 0.4nCV ∆T = p∆V ∆H = ∆U + p∆V 0.4∆U = p∆V → ∆U = p∆V = 253.25 J 0.4 El calor es igual al incremento de entalpía: nC p 1 ⎞ ⎛ ∆H = ∆U + p∆V → nC p ∆T = ∆ T + p ∆V → ⎜ 1 − ⎟ nC p ∆T = p∆V 1.4 ⎝ 1.4 ⎠ p∆V ∆H = Q = = 354.55 J 1 1− 1.4 El rendimiento es: W 101.3 η= = = 0.286 Q 354.55 24º Dos litros de un líquido que cumple la ecuación de estado: V = V0 (1 − k ( p − p0 ) ) con k = 1.82·10–6 atm–1 se comprime desde un estado de equilibrio con p0 = 1 atm y T0 = 273.2 K aplicándole adiabáticamente la presión constante p1 = 570.0 atm, hasta que se iguala a la presión del líquido en el estado de equilibrio final. Determinar el trabajo realizado y los incrementos de energía interna y de entalpía. (Carranza) Solución: Partiendo de la expresión del trabajo para un proceso isobárico: W = p1 (V0 − V1 ) De la ecuación de estado: V1 = V0 (1 − k ( p1 − p0 ) ) = 1.99793 L Por tanto el trabajo: W = 570 ( 2 − 1.99793) = 1.18 atm·L = 119.6 J Y al ser adiabático, el cambio de energía interna es: ∆U = −W = −119.6 J La entalpía es: H = U + pV El cambio de la entalpía es: ∆H = H1 − H 0 = U 1 −U 0 + p1V1 − p0V0 = ∆U + p1V1 − p0V0 38
  • 40. Julián Moreno Mestre 25º Dos moles de un gas ideal se encuentran en un cilindro adiabático cerrado por un pistón libre, sin masa, también adiabático, que soporta inicialmente la presión atmosférica y que tiene una superficie de 5.10 cm2. Una vez alcanzado el equilibrio en esas condiciones, se deposita sobre el pistón un cuerpo de masa 10 kg, y se comprueba que en el estado final de equilibrio el volumen del gas se reduce a las dos terceras partes del inicial. Determinar el calor a volumen constante del gas. Solución: Al depositarse una masa de 10 kg, esta contribuye a generar una presión adicional de: F mg pm = = = 1.923·104 pas = 1.899 atm S S La presión total es la suma de la atmosférica y la ejercida por la masa, ósea 190.9 atm. Partiendo de la expresión del trabajo (a presión constante) será igual al cambio de energía interna: ∆U = nCv (T f − Ti ) = −W = − p f (V f − Vi ) Para la temperatura final e inicial, despejamos: piVi = nRTi Ti = 2 V f = Vi 3 pV 2 p f Vi Tf = f f = 3 nR nR p f V f = nRT f piVi nR Por tanto: nCv (T f − Ti ) = − p f (V f − Vi ) → Cv = Cv = R (2 p pf f − p f (V f − Vi ) n (T f − Ti ) − 3 pi ) ⎛2 ⎞ − p f ⎜ Vi − Vi ⎟ ⎝3 ⎠ = ⎛ 2 p f Vi piVi ⎞ n⎜ − ⎟ nR ⎠ ⎝ 3 nR = 2.38 R 26º Demuestre que en un proceso adiabático de un gas perfecto se cumple: γ −1 ⎡ ⎤ ⎛ p2 ⎞ γ γ ⎢ H 2 − H1 = RT1 ⎜ ⎟ − 1⎥ ⎢⎝ p1 ⎠ ⎥ γ −1 ⎢ ⎥ ⎣ ⎦ siendo T1 y p1 las coordenadas iniciales y T2 y p2 las finales. Solución: Partiendo de la expresión del cambio de la entalpía con la temperatura: ∆H = H 2 − H1 = C p (T2 − T1 ) Y que: C p − CV = R ⎫ CV R γR γR ⎪ Cp = = ⎬ ⇒ C p = γ CV = γ CV = γ R =γ ⎪ R C p − CV C p γ −1 −1 CV ⎭ CV Conseguimos: γR H 2 − H1 = (T − T ) γ −1 2 1 39
  • 41. Ejercicios y problemas de Termodinámica I Y siguiendo la adiabática: 1−γ 1−γ ⎛p ⎞ ⎛p ⎞γ 1 p1−γ T γ = cte → p1 −γ T1γ = p1−γ T2γ → T2 = γ ⎜ 1 ⎟ T1γ = ⎜ 1 ⎟ T1 2 ⎝ p2 ⎠ ⎝ p2 ⎠ Sustituyendo conseguimos demostrar la igualdad: 1−γ 1−γ ⎡ ⎤ ⎛ ⎞ ⎛ p1 ⎞ γ γ γ R ⎜ ⎛ p1 ⎞ γ ⎟→ H −H = ⎢ RT1 ⎜ ⎟ − 1⎥ H 2 − H1 = T1 − T1 2 1 ⎜ ⎜ p2 ⎟ ⎟ ⎢⎝ p2 ⎠ ⎥ γ −1 γ −1 ⎜ ⎝ ⎠ ⎟ ⎢ ⎥ ⎣ ⎦ ⎝ ⎠ 40
  • 43. Ejercicios y problemas de Termodinámica I CAPÍTULO 3º Segundo principio de la termodinámica. Temperatura termodinámica y entropía. Principio de aumento de entropía. Ecuación fundamental de la termodinámica. Ecuaciones TdS. Resumen de Teoría: Rendimiento de cualquier máquina térmica: W Q − Q2 η= = 1 Q1 Q1 Segundo principio de la termodinámica: Teorema de Carnot: Ninguna máquina operando entre dos temperaturas dadas posee un rendimiento superior al de una máquina de Carnot que funcionase entre las mismas temperaturas. Ciclo de Carnot: (T1>T2) 1º Expansión isoterma a T1, Q1>0 2º Expansión adiabática. Q = 0 3º Compresión isoterma a T2, Q2<0 4º Compresión adiabática Q = 0. Rendimiento de un ciclo de Carnot: η = 1− T2 T1 Enunciado de Kelvin-Planck: No es posible ninguna transformación termodinámica cuyo único resultado sea la absorción de calor de un solo foco y la producción de una cantidad equivalente de trabajo. Enunciado de Clausius: No es posible ningún proceso espontáneo cuyo único resultado sea el paso de calor de un recinto a otro de mayor temperatura. Teorema de Clausius (Entropía): Sea un sistema que verifica una transformación cíclica durante la cual intercambia calor con una serie de recintos a las temperaturas T1,…,Tn. Llamemos Q1,…,Qn las cantidades respectivas de calor intercambiadas por el sistema, se verifica entonces que: n δQ Q ∆S = ∫ i ∆S = ∑ i Ti i Ti Principio de Caratheodory: Si un sistema se encuentra en un estado equilibrio térmico, siempre existen otros estados próximos a aquel que no pueden alcanzarse mediante procesos adiabáticos. Entropía de mezcla: ∆S M = − nR ∑ xi ln xi 42
  • 44. Julián Moreno Mestre Ecuaciones de estado de la termodinámica: ⎛ ∂U ⎞ ⎛ ∂p ⎞ ⎛ ∂U ⎞ ⎛ ∂S ⎞ dU = TdS − pdV → ⎜ ⎟ =T⎜ ⎟ − p → ⎜ ∂V ⎟ = T ⎜ ∂T ⎟ − p ⎝ ⎠T ⎝ ⎠V ⎝ ∂V ⎠T ⎝ ∂V ⎠T ⎛ ∂H ⎞ ⎛ ∂H ⎞ ⎛ ∂S ⎞ ⎛ ∂V ⎞ dH = TdS + Vdp → ⎜ ⎟ = T ⎜ ⎟ + V → ⎜ ∂p ⎟ = −T ⎜ ∂T ⎟ + V ⎝ ⎠p ⎝ ⎠T ⎝ ∂p ⎠T ⎝ ∂p ⎠T Ecuaciones T·dS: ⎛ ∂V ⎞ TdS = C p dT − T ⎜ ⎟ dp → TdS = C p dT − T β Vdp ⎝ ∂T ⎠ p Tβ ⎛ ∂p ⎞ TdS = CV dT + T ⎜ dV ⎟ dV → TdS = CV dT − κT ⎝ ∂T ⎠V C C ·κ TdS = λ dV + µ dp → TdS = p dV + V T dp βV β Aplicación a los gases ideales: dT dV dT dp dS = CV +R dS = C p −R T V T p 43
  • 45. Ejercicios y problemas de Termodinámica I Problemas: 1º Para mantener un edificio la temperatura media de 18 ºC, su sistema frigorífico se ve obligado a extraer de su interior 600.0 cal·s–1, mientras consume un trabajo eléctrico de 1.00 kW. Determinar el incremento de entropía por segundo que sufre el universo debido al acondicionamiento del edificio sabiendo que el ambiente externo se encuentra a 35 ºC. Solución: Consideraremos el edificio y el medio como dos focos cuyas temperaturas permanecen inalterables todo el tiempo, y que sobre ellos trabaja una máquina frigorífica. Llamemos Q1 al calor por unidad de tiempo extraído por la máquina del edificio, W al trabajo consumido por unidad de tiempo, y Q2 al calor total producido por unidad de tiempo. Por tanto: Q2 = Q1 + W = 600 cal/s + 1000 J/s = 839.2 cal/s La entropía por unidad de tiempo perdida por el edificio y cedida al medio es: Q −600 Q 839.2 ∆S1 = 1 = = −2.06 cal/K·s ∆S2 = 2 = = 2.72 cal/K·s T1 291 T2 308 La producción total de entropía por unidad de tiempo es: ∆S = ∆S1 + ∆S2 = 0.66 cal/K·s 2º En un calorímetro adiabático se mezclan 100.0 g de mercurio a 100.0 ºC con 50.0 g de hielo a 0.0 ºC. Determinar el incremento de entropía del mercurio, del agua, y del universo sabiendo que ambos líquidos son perfectamente inmiscibles. Datos: Calor específico de mercurio ce,Hg = 0.033 cal·g –1·K –1. Calor específico del agua c1 = 1.00 cal·g –1·K –1. Calor latente de fusión del hielo Lfus = 80.0 cal· g –1. Solución: Antes de empezar, verificaremos o descartaremos que la temperatura de equilibrio no sea de 0 ºC (=273 K) dado que el calor latente de fusión del hielo es bastante más elevado que el calor específico del mercurio. El calor necesario para fundir todo el hielo y para cambiar 100 grados la temperatura del mercurio es: Qfus = Lfus ·mhielo = 80·50 = 4000 cal QHg,100 ºC→0 ºC = ce,Hg ·mHg ·∆T = 0.033·100·100 = −330 cal Dado que el calor necesario para fundir el hielo es demasiado elevado y no es superado por el de pasar mercurio de 100 ºC (=373 K) a 0 ºC (=273 K), esto implica que la temperatura final es de 0 ºC (=273 K). La masa de hielo y agua (dado que parte del hielo se derretirá calentando el mercurio) permanece a temperatura constante, la entropía perdida es: −QHg,100 ºC→0 ºC 330 ∆Shielo+agua = = = 1.21 cal/K T 273 El cambio de entropía del mercurio viene dado por: f δ QHg,100 ºC→0 ºC 273 ce,Hg mHg dT ⎛ 273 ⎞ ∆S Hg = ∫ = ∫ = 0.033·100 ln ⎜ ⎟ = −1.03 cal/K T T ⎝ 373 ⎠ i 373 La entropía total es: ∆S = ∆S hielo+agua + ∆S Hg = 0.18 cal/K 44
  • 46. Julián Moreno Mestre 3º Una viga metálica de 125 kg se encuentra en un ambiente a 12ºC y dispuesta para ser colocada en un edificio en construcción. Por un descuido, la viga cae al suelo desde una altura de 24 m sin sufrir daños. Posteriormente la viga se pone en su lugar, a 24 m de altura, usando un motor que consume una potencia de 0.5 kW durante minuto y medio. Calcular el incremento de entropía que ha experimentado el universo. Solución: Dado que es constante la temperatura, el incremento de entropía es proporcional a la energía perdida en el problema. La energía que se pierde es la potencial que tiene la viga y aquella del motor que no es íntegramente empleada en subir la viga. Procedemos a calcularlas comenzando por la potencial. E p = mgh = 29400 J Esta es la energía potencial que perderá la viga. El trabajo que produce el motor para subir la viga es: W = P·t = 45000 J Teniendo en cuenta que de ese trabajo solo 29400 J se usarán para subir otra vez la viga, quedando en forma de energía reutilizable (mientras no se vuelva a caer otra vez), el resto (15600 J) se perderá seguramente en forma de calor. Por tanto la energía total perdida tras la caída de la viga y su posterior colocación es: E = 29400 + 15600 = 45000 J La entropía generada (incrementada en el universo) es por tanto: E 45000 ∆S = = = 157.9 J/K T 285 4º Una máquina térmica funciona entre un depósito que contiene 1·103 m3 de agua y un río a temperatura constante de 10 ºC. Si la temperatura inicial del depósito es 100 ºC, ¿Cuál es la cantidad máxima de trabajo que puede realizar la máquina térmica? Solución: Consideraremos el río como un foco térmico cuya temperatura no cambiará en todo el proceso. En cambio el depósito de agua va a descender su temperatura hasta igualarla con la temperatura del río. Dado que se nos pide calcular el trabajo máximo que puede realizar la máquina térmica que acoplaríamos entre el río y el depósito, esto significa que la maquina realizaría ciclos de Carnot para obtener el máximo trabajo posible, lo cual implica que el incremento de entropía del universo es nulo: ∆Suniverso = ∆S rio + ∆S deposito = 0 → ∆S rio = −∆S deposito La entropía generada por el depósito es: Trio 283 δ Qdeposito Trio ce mdT dT ⎛ 283 ⎞ 9 ∆Sdeposito = ∫ = ∫ = 4.18·109 ∫ = 4.18·109 ln ⎜ ⎟ = −1.154·10 J/K 373 ⎠ T T T ⎝ 373 Tdepo Tdep El calor desprendido por el depósito es igual al cambio de energía interna experimentado por este: Qdeposito = ∆U deposito = mce ∆T = −106 ·4180·90 = −3.76·106 J La entropía recibida por el río, y por tanto el calor recibido, es: ∆S rio = −∆Sdeposito = 1.154·109 J/K → Qrio = Trio ·∆S rio = 3.27·1011 J El trabajo máximo que puede realizar la máquina térmica es: Wmax = Qdeposito − Qrio = 4.92·1010 J 45
  • 47. Ejercicios y problemas de Termodinámica I 5º Una masa de 10 g de hielo a 0 ºC se ponen en contacto con una fuente térmica a 350.0 K, hasta que alcanza un nuevo estado de equilibrio. Determinar la variación de entropía del agua y del universo sabiendo el calor específico del agua líquida a presión constante 1.00 cal·g –1·K –1 y su calor latente de fusión de 80.0 cal· g –1. Solución: El estado de equilibrio es agua líquida a 350 K. El cambio de entropía del agua hasta ese nuevo estado de equilibrio lo determinaremos en dos etapas: Primera etapa: El hielo se funde a temperatura constante recibiendo calor de la fuente térmica. El cambio de entropía y el calor cedido por la fuente es: Q L ·m 10·80 Qfus = Lfus ·m = 10·80 = 800 cal ∆S1 = fus = fus = = 2.9 cal/K T T 273 Segunda etapa: El agua liquida sube su temperatura desde los 273 K hasta los 350 K recibiendo calor de la fuente de calor. El cambio de entropía y el calor cedido por la fuente es: Q273 K →373 K = m·ce ∆T = 10·1·77 = 770 cal f δQ m·ce dT ⎛ 350 ⎞ = 10·1·ln ⎜ ⎟ = 2.48 cal/K T T ⎝ 273 ⎠ 273 i El cambio total de entropía del agua es: ∆S H2O = ∆S1 + ∆S 2 = 5.38 cal/K ∆S 2 = ∫ 350 = ∫ El calor total perdido por la fuente es equivalente al ganado por el agua, luego: Qfuente = −QH2O = − ( Q fus + Q273 K →373 K ) = −1570 cal y su cambio de entropía (recordando que su temperatura es constante) es: Q −1570 ∆Sfuente = fuente = = −4.48 cal/K Tfuente 350 El cambio de entropía del universo es: ∆S Universo = ∆Sfuente + ∆S H2O = 0.9 cal/K 6º Hallar el cambio de entropía del universo al transferirse 500 J de energía desde un foco de 400 K hasta un foco de 300 K. Si hay una máquina térmica reversible entre los focos que recibe los 500 J del foco de temperatura 400 K, hallar el trabajo producido. Solución: En el primer apartado se transfieren 500 J desde el foco a 400 K al foco a 300 K. La entropía generada en cada foco es: Q −500 Q 500 ∆S1 = 1 = = −1.25 J/K ∆S2 = 2 = = 1.6 J/K T1 400 T2 300 El cambio de entropía del universo es: ∆Su = ∆S1 + ∆S 2 = 0.416 J/K En el segundo apartado se instala una máquina térmica reversible. Al ser reversible el cambio de entropía total del universo será cero, por tanto: Q Q ∆Su = ∆S1 + ∆S 2 = 1 + 2 = 0 T1 T2 46
  • 48. Julián Moreno Mestre Sabemos que se transfieren 500 J a la máquina desde el foco a 400 K, por tanto el calor transferido por la máquina al foco a 300 K será: Q1 Q2 −500 Q2 + = + = 0 → Q2 = 375 J 400 300 T1 T2 El trabajo realizado por la máquina es igual a la diferencia de los valores absolutos de los calores cedido o ganado por los focos: W = Q1 − Q2 = 125 J 7º Un mol de gas ideal se comprime isotérmicamente aplicándole una presión constante de 15 atm, desde un volumen de 18.2 L hasta un volumen de 9.85 L a la temperatura de 13.2 ºC. Determinar el incremento de entropía que experimenta el gas. Solución: Es un proceso isotermo. Eso significa según el primer principio que: δ Q = −δ W = pdV → Q = p(V2 − V1 ) = 125.25 atm·L = 12693 J Por tanto el incremento de entropía del sistema es: Q 12693 ∆S = = = 44.3 J/K T 286.36 8º Una máquina térmica funciona reversiblemente (ideal) entre 2 focos térmicos, uno de ellos formado por 103 Kg de vapor de agua a 100ºC, y otro, por 103 Kg de hielo a 0ºC a la presión de una atmósfera. ¿Cuál es el rendimiento total de la máquina? ¿Qué trabajo podrá producir hasta que se funda todo el hielo? ¿Cuál es el incremento de entropía del universo y de los focos? Datos: Calor latente de fusión del hielo: Lfus = 80 cal·g-1. Calor latente de vaporización del agua: LVap = 540 cal·g-1. Solución: La máquina funcionará hasta que se haya producido un equilibrio de temperatura entre los dos focos. Dado que la las masas de agua son iguales es de esperar por el elevado calor latente de vaporización/licuación (LVap) del agua a 100 ºC, mucho mas que el de fusión/solidificación (Lfus) del agua a 0 ºC, todo el hielo se derrita y la masa de agua suba hasta los 100 ºC que sería la temperatura de equilibrio, licuándose en ese proceso parte del gas. Hagamos unos cálculos para verificar esta posibilidad: Qfus = Lfus ·m = 8·107 cal Q0 ºC→100 ºC = mce ∆T = 10·107 cal Qvap = Lvap ·m = 54·107 cal Siendo tan elevado el calor de vaporización, superior a la suma de los otros dos, es pues deducible que la temperatura de equilibrio es de 100 ºC. El calor transferido a la máquina por el foco helado es por tanto: Q1 = Qfus + Q0 ºC→100 ºC = 18·107 cal Al saber que la maquina es reversible o ideal, es fácil suponer que el incremento de entropía del universo es nulo. Esto supone que la entropía ganada por un foco es igual a la entropía perdida por el otro. El incremento de entropía del foco 1 (el del hielo) lo calcularemos en dos etapas: 1ª etapa: Es un proceso isotermo ya que el hielo se está fundiendo mientras se le suministra calor, su incremento de entropía es: Q 8·107 ∆SI = fus = = 2.93·105 cal/K T1 273 47
  • 49. Ejercicios y problemas de Termodinámica I 2ª etapa: Proceso a temperatura variable, desde 0 ºC (= 273 K) hasta los 100 ºC (= 373 K). f 373 δQ dT ⎛ 373 ⎞ 5 ∆S II = ∫ = mce ∫ = 106 ·1·ln ⎜ ⎟ = 3.12·10 cal/K T T ⎝ 273 ⎠ 273 i La entropía total del foco 1 y del foco 2 es por tanto: ∆S1 = ∆SI + ∆S II = 6.05·105 cal/K ∆S2 = −∆S1 = −6.05·105 cal/K Dado que el foco 2 permanece en todo momento a temperatura constante, es posible conocer el calor total que se le transfirió: Q ∆S2 = 2 → Q2 = ∆S2 ·T2 = (−6.05·105 )·373 = −22.56·107 cal T2 Por tanto el trabajo realizado por la maquina es: W = Q2 − Q1 = 4.57·107 cal El rendimiento de la máquina es por tanto: W 4.57·107 η= = = 0.203 Q2 22.56·107 Rendimiento del 20.3 %. 9º Calcular la entropía molar de un gas B a 300K y una atmósfera de presión, sabiendo que ese compuesto es sólido por debajo de 100 K, y que sublima a esa temperatura cuando la presión es de 1 bar, con una entalpía de sublimación de 2000 cal/mol. Datos: c p ( s ) = 2.5·10−4 T 3 − 2.4·10−10 T 6 cal/K·mol c p ( g ) = 6 cal/K·mol Solución: Dado que conocemos como crece su calor específico (a presión constante) en estado sólido. Para calcular su entropía lo describiremos tres etapas tal y como indica la figura: La entropía molar es por tanto: s = s0 + ∆s1 + ∆s2 + ∆s3 Calculemos la producción de cada etapa: Primera etapa: Evolución del solido desde 0 K hasta 100 K. −4 3 −10 6 f δ q 100 c p ( s )dT 100 ( 2.5·10 T − 2.4·10 T ) dT ∆s1 = ∫ = ∫ = ∫ =… T T T 0 0 i 100 ⎡ 2.5·10−4 T 3 2.4·10−10 T 6 ⎤ 2.5·10−4 ·1003 2.4·10−10 ·1006 − − = 43.3 cal/K·mol …= ⎢ ⎥ = 3 6 3 6 ⎣ ⎦0 Segunda etapa: Proceso isotermo, el sólido sublima y se convierte en el gas B. q 2000 ∆s2 = = = 20 cal/K·mol T2 100 Tercera etapa: Evolución del sólido desde 100 K hasta 200 K. f δ q 300 c p ( g )dT 300 6·dT ⎛ 300 ⎞ ∆s1 = ∫ = ∫ = ∫ = 6 ln ⎜ ⎟ = 6.6 cal/k·mol T 100 T T ⎝ 100 ⎠ 100 i 48
  • 50. Julián Moreno Mestre Por tanto: s = s0 + ∆s1 + ∆s2 + ∆s3 = s0 + 69.9 Dado que el gas convertido en sólido es puro, y tal vez forme sólidos cristalinos perfectos, entonces el valor de s0 es con seguridad cero, por esto la entropía molar del gas a 300 K es: s = 69.9 cal/K·mol 10º Un mol de gas ideal monoatómico a la temperatura de 399 K y 4 bar de presión, se encuentra encerrado en un cilindro de paredes adiabáticas provisto de un pistón no conductor del calor. Determinar la variación de entropía del gas al expansionarse bruscamente contra una presión exterior de 1 bar. Solución: Convertimos en atmósferas las presiones. pi = 3.949 atm p f = 0.987 atm Como es una expansión brusca, es una expansión irreversible, y además cambian las tres variables p,V y T. Para poder determinar el nuevos estado de equilibrio, recurrimos al primer principio de la termodinámica y a la ecuación del gas ideal (con n = 1 moles): dU = δ Q + δ W pV = RT Sabiendo que no es posible el intercambio de calor con el exterior, y que es una expansión a presión constante pf: dU = δ W → Cv ∆T = − p f (V f − Vi ) → Cv (T f − Ti ) = − p f (V f − Vi ) RTi T 3 3 R(T f − Ti ) = − p f (V f − Vi ) = − RT f + p f → (T f − Ti ) = −T f + p f i 2 2 pi pi p 2 pf 3 2 0.987 3 5T f − 3Ti = 2 f Ti → T f = ·399 + ·399 = 279.29 K Ti + Ti = · 5 pi 5 5 3.949 5 pi Una vez conocida la temperatura final del gas, determinamos el valor del volumen final e inicial: RT f 0.082·279.29 RT 0.082·399 Vi = i = = 8.285 L Vf = = = 23.203 L 3.949 pi 0.987 pf Ahora ya podemos conocer el cambio de entropía del sistema: 279.29 23.203 δ Q dU − δ W dT pdV dT dV dT dV dS = = = Cv − = Cv +R → ∆S = Cv ∫ +R ∫ T T T T T V T V 399 8.285 3 ⎛ 279.29 ⎞ ⎛ 23.203 ⎞ ∆S = 8.31·ln ⎜ ⎟ + 8.31·ln ⎜ ⎟ = −4.446 + 8.558 = 4.112 J/K 2 ⎝ 399 ⎠ ⎝ 8.285 ⎠ 49
  • 51. Ejercicios y problemas de Termodinámica I 11º Un número n de moles de un gas ideal experimentan una expansión libre y adiabática en el vacío (experimento de Joule). Expresar en términos de la temperatura y volumen inicial y final el incremento de entropía del sistema. Calcular cuanto vale la entropía molar si el volumen final es doble que el volumen inicial. Solución: Al ser una expansión en vacío no se produce ningún cambio en la energía interna del gas, luego es un proceso a temperatura constante. El primer principio de la termodinámica nos establece entonces que: δ Q = −δ W = pdV Y por tanto: V V Vf Vf f ⎛V ⎞ δ Q f pdV 1 f 1 nRT dV dV = nR ∫ ∆S = ∫ =∫ = ∫ pdV = ∫ = nR ln ⎜ f ⎟ T T T Vi T Vi V V ⎝ Vi ⎠ i Vi Vi Si el volumen final es doble que el inicial, entonces la entropía molar es: ⎛ 2V ⎞ ∆S ∆s = = R ln ⎜ i ⎟ = 8.31·ln 2 = 5.76 J/K·mol n ⎝ Vi ⎠ 12º Un cilindro vertical de paredes adiabáticas y provisto de un pistón no conductor del calor, está dividido por una pared fija y diatérmica. La parte superior contiene 10 moles de un gas ideal y la inferior una mezcla en equilibrio a 1.013 bar de 100 g de agua y 100 g de hielo. Se introduce lentamente el pistón hacia dentro hasta que la presión del gas se duplica. ¿Cuál es la variación de entropía del gas, de la mezcla y del universo en este proceso? Solución: La situación de equilibrio inicial nos indica que la temperatura inicial es de 0 ºC ya que una mezcla de agua + hielo solo coexiste a esa temperatura a la presión de 1.013 bar = (1 atm). Antes de abordar la resolución del problema, comprobaremos si el calor necesario para fundir el hielo es mayor que el proceso isotermo (con T = 273 K) de comprimir el gas de la parte de arriba: V /2 V /2 dV W = − ∫ pdV = − nRT ∫ = nRT ln 2 = 15725 J V V V El calor necesario para fundir el hielo es igual a: Q f = L f ·m = 33440 J Esto significa que todo el proceso es isotermo. La entropía perdida por el gas por el traspaso en forma de calor a la masa de agua + hielo proveniente del trabajo suministrado por el exterior es: −Q −W 15725 ∆Sgas = = =− = −57.6 J/K T T 273 La entropía ganada por el agua + hielo es: Q 15725 ∆Sagua+hielo = = = 57.6 J/K T 273 La entropía del universo es: ∆Suniverso = ∆Sgas + ∆Sagua+hielo = 0 50
  • 52. Julián Moreno Mestre 13º Un mol de gas ideal monoatómico recorre un ciclo de Carnot reversible, como el indicado en la figura, con V1 = 20 L, V2 = 40 L, T1 = 300 K y T3 = 200 K. Calcúlense ∆p, ∆V, ∆T, ∆U, ∆H y ∆S en cada etapa del ciclo. Solución: Mediante la ecuación de estado del gas ideal y de las adiabáticas, determinamos los valores de las variables p, V y T: pV = RT VT γ −1 = cte Al ser gas monoatómico γ = 1.67. El estado 1 es el estado inicial, luego los valores p, V y T son: RT T1 = 300 K V1 = 20 L p1 = 1 = 1.23 atm V1 Al estado 2 se llega mediante un proceso isotermo desde el estado 1: RT T1 = T2 = 300 K V2 = 40 L p2 = 2 = 0.615 atm V2 Al estado 3 se llega mediante un proceso adiabático desde el estado 2: RT T3 = 200 K V2T2γ −1 = V3T3γ −1 → V3 = 52.49 L p3 = 3 = 0.312 atm V3 Al estado 4 se llega mediante un proceso isotermo desde el estado 3 y está conectado con el 1 mediante un proceso adiabático: RT T4 = 200 K p4 = 4 = 0.625 atm V4T4γ −1 = V1T1γ −1 → V4 = 26.24 L V4 Tras estos cálculos podemos calcular los cambios de cada una de las variables en las respectivas etapas. Etapa 1 a 2: ∆T1→ 2 = 0 ∆V1→2 = 20 L ∆p1→2 = −0.615 atm 3 5 R∆T1→2 = 0 J ∆H1→2 = C p ∆T1→2 = R∆T1→2 = 0 J 2 2 V2 V2 ⎛V ⎞ W 1 1 RT dV = R ln ⎜ 2 ⎟ = 5.76 J/K = 1→2 = ∫ pdV = ∫ T T V1 T V1 V ⎝ V1 ⎠ ∆U1→2 = Cv ∆T1→2 = ∆S1→2 = Q1→2 T Etapa 2 a 3: ∆T2→3 = −100 K ∆V2→3 = 12.49 L ∆p2→3 = −0.303 atm 3 R∆T2→3 = −1246.5 J 2 5 ∆H 2→3 = C p ∆T2→3 = R∆T2→3 = −2077.5 J 2 Por ser un proceso adiabático reversible ∆S2→3 = 0 ∆U 2→3 = Cv ∆T2→3 = 51
  • 53. Ejercicios y problemas de Termodinámica I Etapa 3 a 4: ∆T3→4 = 0 ∆V3→4 = −26.25 L ∆p3→4 = 0.313 atm 3 5 R∆T3→4 = 0 J ∆H 3→4 = C p ∆T3→4 = R∆T3→4 = 0 J 2 2 V3 V3 ⎛V ⎞ W 1 1 RT = 3→4 = ∫ pdV = ∫ dV = R ln ⎜ 3 ⎟ = −5.76 J/K T T V4 T V4 V ⎝ V4 ⎠ ∆U 3→4 = Cv ∆T3→4 = ∆S3→4 = Q3→4 T Etapa 4 a 1: ∆T4→1 = 100 K ∆V4→1 = 12.49 L ∆p4→1 = −0.303 atm 3 R∆T4→1 = 1246.5 J 2 5 ∆H 4→1 = C p ∆T4→1 = R∆T4→1 = 2077.5 J 2 Por ser un proceso adiabático reversible ∆S4→1 = 0 ∆U 4→1 = Cv ∆T4→1 = 14º Un mol de gas ideal se expansiona isotérmicamente, a 27 ºC, desde 20 a 40 litros en tres procesos diferentes: a) El primero se efectúa, de forma reversible, reduciendo lentamente la presión sobre el pistón hasta que se alcanza el valor final pext. b) En el segundo, la presión disminuye bruscamente hasta su valor final pext. c) En el tercero, el gas ocupa inicialmente un volumen de 20 litros; repentinamente y por rotura de una fina membrana, se deja que se expansione en el vacío hasta los 20 litros restantes. Calcúlense en cada proceso Q, W, ∆U y ∆S del gas, del foco y del sistema total aislado adiabáticamente (gas+foco). Solución: Primero determinamos las presiones de los estados inicial y final de equilibrio: RT RT pi = = 1.23 atm pf = = 0.615 atm Vi Vf a) Reducción lenta de la presión (isoterma reversible). El trabajo es: Vf Vf ⎛V ⎞ dV W = ∫ pdV = RT ∫ = RT ln ⎜ f ⎟ = 1728 J V ⎝ Vi ⎠ Vi Vi Al ser expansión isoterma de un gas ideal, esto significa que el incremento de energía interna es cero. Por el primer principio estableceremos por tanto: Q = W = 1728 J El cambio de entropía de este proceso es: ∆S = Q = 5.76 J/K T b) El gas se expande en este caso a presión exterior constante, el trabajo es por tanto: W = p f (V f − Vi ) = 0.615·(40 − 20) = 1240 J El cambio de energía interna vuelve a ser cero ya que las temperaturas iniciales y finales son idénticas. Por tanto el calor es: Q = W = 1240 J 52
  • 54. Julián Moreno Mestre La entropía es: f ∆S = ∫ i δQ T f = −∫ i δW T Vf = ∫ Vi f ⎛V ⎞ pdV dV = nR ∫ = nR ln ⎜ f ⎟ = 5.76 J/K T V ⎝ Vi ⎠ Vi V c) El gas se expande 20 L en vacío. Aquí el trabajo es nulo, nulo también es el cambio de energía interna al ser constante la temperatura, y el calor también es cero por el primer principio. Pero el cambio de entropía es: Vf Vf f f ⎛V ⎞ δQ δW pdV dV ∆S = ∫ = −∫ =∫ = nR ∫ = nR ln ⎜ f ⎟ = 5.76 J/K T T T V ⎝ Vi ⎠ i i Vi Vi Aunque Q y W sean cero, los δ Q y δ W no son diferenciales exactas y pueden no ser nulos. 15º Cierto sistema hidrostático tiene isotermas dadas por pV 2 = cte y una energía interna dada por U = pV/2. Dicho sistema describe un ciclo A → B → C → A en tres etapas, siendo el proceso A → B adiabático reversible, el proceso B → C adiabático irreversible y el proceso C → A isotermo reversible. Calcúlese el calor intercambiado por el sistema y el cambio de entropía de este en cada uno de los procesos, en función de las coordenadas de cada punto. Solución: En la etapa de A → B al ser adiabática reversible no existe intercambio de calor y el incremento de entropía es nulo. En la segunda etapa B → C, que es adiabática irreversible tampoco se intercambia calor con los alrededores, pero la entropía es distinta de cero. No tenemos forma exacta de calcularla con los datos de la etapa B → C. En la última etapa C → A estamos ante un proceso isotermo reversible, que devuelve al punto de partida al sistema hidrostático. En este proceso: QC→A = ∆U C→A − WC→A Donde: pV pV ∆U C→A = U A − U C = A A − C C 2 2 Teniendo en cuenta la ecuación que describe las isotermas: 2 pV 2 = pAVA = pCVC2 = cte Determinamos el trabajo en la etapa C → A: VA VA VA 1 ⎞ pCVC2 pCVC2 dV 2 2 ⎡ −1 ⎤ 2⎛ 1 WC → A = ∫ pdV = pCVC ∫ 2 = pCVC ⎢ ⎥ = pCVC ⎜ − ⎟ = − =… V VC VA ⎣ V ⎦VC ⎝ VC VA ⎠ VC VC … = pCVC − Por tanto: pCVC2 p V2 = pCVC − A A → WC → A = pCVC − pAVA VA VA pV p V − pAVA pAVA pCVC pV − + pCVC − pAVA = C C − A A = C C 2 2 2 2 2 La entropía de este proceso (isotermo C → A) es: p V − pAVA Q ∆SC→A = C→A → ∆SC→A = C C 2TA TA Dado que el proceso total A → B → C → A es reversible, el balance total de entropía es nulo para el sistema hidrostático, y que la entropía perdida en el proceso C → A es equivalente a la entropía ganada por B → C, por tanto: p V − pCVC ∆SB→C = −∆SC→A = A A 2TA QC→A = ∆U C→A + WC→A = 53
  • 55. Ejercicios y problemas de Termodinámica I 16º Un sistema tiene una capacidad calorífica a volumen constante dada por: CV = AT 2 donde A = 0.0418 J·K–3. El sistema se encuentra inicialmente a 200 ºC y puede ser enfriado a 0 ºC mediante uno u otro de los siguientes procesos: a) Por contacto directo con un foco térmico a esa temperatura. b) Haciendo funcionar una máquina térmica reversible entre el sistema y el foco térmico. Determínese, en cada caso, el trabajo obtenido y los cambios de entropía del sistema, del foco y del universo. Solución: En el primer proceso tenemos que por contacto directo el cambio de energía interna experimentado por el sistema es igual al calor transferido. No se convierte nada en trabajo, ya que no experimenta un cambio de volumen ni existe una máquina térmica que convierta calor en trabajo. Luego: 273 ⎡T 3 ⎤ ⎡ 2733 4733 ⎤ 6 ∆U = Q = ∫ AT dT = A ⎢ ⎥ = 0.0418 ⎢ − ⎥ = −1.191·10 J 3 ⎦ ⎣ 3 ⎦ 473 ⎣ 3 473 El cambio de entropía experimentado por el sistema, el foco y el universo es por tanto: 273 2 273 ∆S S = ∫ 473 δQ 273 273 ⎡T 2 T 2 ⎤ AT 2 = ∫ dT = A ∫ TdT = A ⎢ − ⎥ = −3118 J/K T T 2 ⎦ 473 ⎣2 473 473 6 −QS 1.191·10 Q ∆S F = F = = = 4362 J/K 273 TF TF ∆SU = ∆S F + ∆S S = −2079 + 4369 = 1244 J/K 273 En el segundo proceso nos encontramos con que parte del calor transferido a la máquina térmica es convertido en trabajo y el resto cedido en forma de calor. Por tanto el calor cedido por el sistema menos el calor cedido al foco es igual al trabajo: QS − QF = W Dado que es una máquina reversible la entropía del universo es cero, por tanto la entropía ganada al absorber calor del sistema es igual a la entropía perdida al desprender el calor al foco (de igual magnitud pero signo contrario según los convenios de signos): 273 273 δQ AT 2 dT = 3118 J/K ∆SU = ∆S S + ∆S F = 0 → ∆S F = −∆S S → ∆S F = − ∫ =−∫ T T 473 473 Por tanto el calor cedido al foco es: QF = T ·∆S F = 8.51·105 J El trabajo es por tanto: W = QS − QF = 1.191·106 − 8.51·105 = 3.4·105 J 54